0 - MRCP May 2019

You might also like

Download as pdf or txt
Download as pdf or txt
You are on page 1of 305



Question bank
PAST-PAPERS
‫‪‬إ‪‬اء‪‬‬
‫‪ ‬آﯾﺔ ﻣﻦ ﻛﺘﺎب ﷲ ﺣﻔﻈﻨﺎھﺎ وﻟﻜﻞ ﻣﻌﻠﻮﻣﺔ ﻗﺮأﻧﺎھﺎ وﻧﺴﯿﻨﺎھﺎ ﻟﻌﻞ ﷲ ﯾﺤﻤﯿﻨﺎ ﻣﻦ اﻟﻨﺴﯿﺎن وﯾﺮدھﺎ إﻟﯿﻨﺎ‬
‫‪ ‬ﺷﺨﺺ دﻋﻮﻧﺎ ﻟﮫ ﻓﻲ ظﮭﺮاﻟﻐﯿﺐ ﻟﻌﻞ ﷲ أن ﯾﺴﺘﺠﯿﺐ دﻋﺎﺋﻨﺎ‬
‫‪ ‬ﻣﺮﯾﺾ ﻗﺪ آﻟﻤﮫ ﻣﺎ أﻟﻤﮫ ﻟﻌﻞ ﷲ ﯾﺸﻔﯿﮫ وﯾﻌﯿﺪ إﻟﯿﮫ ﺻﺤﺘﮫ وﻋﺎﻓﯿﺘﮫ‬
‫‪ ‬ﻟﺤﻈﺔ ﺗﺂﻟﻢ ﺑﮭﺎ أﺧﻲ وﺟﺪي وأﺣﺒﺘﻲ أﺛﻨﺎء ﻋﻼﺟﮭﻢ ﻟﻌﻞ ﷲ ﻻ ﯾﺮﯾﮭﻢ اﻵﻟﻢ وﻻ ﯾﻀﯿﻤﮭﻢ ﻣﺮة أﺧﺮى‬
‫‪ ‬ﻣﻮﻗﻒ اﺣﺘﺠﺖ ﺑﮫ ﻋﯿﻨﺎي ﻓﻠﻢ ﺗﻠﺒﯿﺎﻧﻲ ﻟﻌﻞ ﷲ ﻻ ﯾﻀﺮﻧﻲ ﺑﮭﻤﺎ ﻓﻲ ﺣﯿﺎﺗﻲ‬
‫‪ ‬ﺣﺎﻟﻢ ﺻﺎﺣﺐ أھﺪاف وﻣﺒﺎدئ ﻟﻌﻞ ﷲ ﯾﺤﻘﻖ ﻟﮫ ﻣﻘﺎﺻﺪه ﺣﺘﻰ ﯾﺮﺿﯿﮫ وﯾﺮﺿﻰ ﻋﻨﮫ‬
‫‪ ‬ﻣﺠﺘﮭﺪ ﻟﻌﻞ ﷲ ﯾﺠﻌﻞ ﻟﮫ ﻣﻦ اﻟﺘﻮﻓﯿﻖ أوﻓﺮ ﺣﻆ و ﻧﺼﯿﺐ وﯾﮭﻮن ﻟﮫ اﻷﺳﺒﺎب ﻛﻠﮭﺎ ﻣﻦ ﺣﯿﺚ ﻻ‬
‫ﯾﺪري ﻣﻦ ﻏﯿﺮ ﺿﺮاء ﻣﻀﺮة وﻻ ﻓﺘﻨﺔ ﻣﻀﻠﮫ‬
‫‪ ‬ﻣﻦ ﻓﻘﺪ ﺷﯿﺌﺎ ﻓﻲ اﻟﺤﯿﺎة ﻟﻌﻞ ﷲ ﯾﻌﯿﺪه إﻟﯿﮫ أو ﯾﻌﻮﺿﮫ ﺑﻤﺎ ﯾﺴﻌﺪه‬
‫‪ ‬ﺷﺨﺺ اﺑﺘﻌﺪﻧﺎ ﻋﻨﮫ ﻟﻌﻞ ﷲ ﯾﺠﻤﻌﮫ ﺑﻤﻦ ھﻢ أﻓﻀﻞ ﻣﻨﺎ‬
‫وأ‪‬ا وﻟﯿﺲ آﺧﺮا ﻷھﻠﻨﺎ ﻟﻌﻞ ﷲ ﯾﺠﻌﻠﮭﺎ ﻟﮭﻢ ﺻﺪﻗﺔ ﺟﺎرﯾﮫ ﺗﺮد إﻟﯿﮭﻢ ﻗﻠﯿﻼ ﻣﻦ ﻓﻀﻠﮭﻢ ﻋﻠﯿﻨﺎ وﺗﻜﻮن ﺳﺒﺒﺎ‬
‫ﯾﻠﻢ ﺷﻤﻠﻨﺎ ﺗﺤﺖ ﺳﻘﻒ واﺣﺪ ﺑﻌﺪ أن ھ ﱠﺠ َﺮﺗﻨﺎ اﻟﺤﺮوب واﻟﻤﺼﺎﺋﺐ‬

‫‪PT Part I 2019 index‬‬


‫‪Branch‬‬ ‫‪No. of Q‬‬
‫‪Cardiology‬‬ ‫)‪(386‬‬
‫‪Clinical haematology/oncology‬‬ ‫)‪(392‬‬
‫‪Clinical‬‬
‫)‪(513‬‬
‫‪pharmacology/therapeutics/toxicology‬‬
‫‪Dermatology‬‬ ‫)‪(138‬‬
‫‪Endocrinology‬‬ ‫)‪(414‬‬
‫‪Gastroenterology‬‬ ‫)‪(403‬‬
‫‪General Revision‬‬ ‫)‪(122‬‬
‫‪Infectious diseases/sexually‬‬
‫)‪(412‬‬
‫‪transmitted diseases‬‬
‫‪Nephrology‬‬ ‫)‪(335‬‬
‫‪Neurology‬‬ ‫)‪(413‬‬
‫‪Ophthalmology‬‬ ‫)‪(81‬‬
‫‪Psychiatry‬‬ ‫)‪(268‬‬
‫‪Respiratory Medicine‬‬ ‫)‪(457‬‬
‫‪Rheumatology‬‬ ‫)‪(380‬‬
Clinical Sciences
Cell/molecular and membrane
(72)
biology
Clinical anatomy (67)
Clinical biochemistry and metabolism (118)
Clinical physiology (92)
Genetics (107)
Immunology (118)
Statistics (158)

Past-Papers index
Passing mark range between 63-66%
Year No. of Q
MRCP 1 May 19 100
MRCP 1 Jan 19 100
MRCP 1 Sept 18 100
MRCP 1 May 18 100
MRCP 1 Jan 18 100
MRCP 1 Sept 17 100
MRCP 1 May 17 100
MRCP 1 Jan 17 100
MRCP 1 Sept 16 100
MRCP 1 May 16 100
MRCP 1 Jan 2016 100
MRCP 1 Sept 2015 100
MRCP 1 May 2015 100
MRCP 1 Jan 2015 100

A. H. Murad
‫ﻻ ﺗﻨﺴﻮﻧﺎ ﻣﻦ ﺻﺎﻟﺢ دﻋﺎﺋﻜﻢ‬
‫ﺗﻢ ﺑﺤﻤﺪ ﷲ وﺗﻮﻓﯿﻘﮫ وﻣﻨﮫ‬
‫ﺟﻌﻞ ﷲ ﻋﻤﻠﻨﺎ ﻣﺘﻘﺒﻼ ﺧﺎﻟﺼﺎ ﻟﻮﺟﮭﮫ اﻟﻜﺮﯾﻢ‬
0:00:17/03:00:00

A 22-year-old man is admitted with an episode of paroxysmal supraventricular tachycardia


(SVT) which required cardioversion after he collapsed at a local night club. This is the second
occasion that he has suffered palpitations over the past six months. He does not drink coffee
and does not use illicit drugs.
His blood pressure is 110/70 mmHg post-cardioversion. His pulse rate is 85 bpm and regular.
His chest is clear, and his body mass index (BMI) is 22 kg/m 2.
Investigations:

Haemoglobin (Hb) 13.0 g/dl (13.5–17.5 g/dl)

White cell count (WCC) 7.0 × 10 9/l (4–11 × 10 9/l)

Platelets (PLT) 203 × 10 9/l (150–400 × 10 9/l)

Sodium (Na +) 142 mmol/l (135–145 mmol/l)

Potassium (K +) 4.4 mmol/l (3.5–5.0 mmol/l)

Creatinine (Cr) 85 µmol/l (50–120 µmol/l)

Sinus rhythm, shortened PR


Electrocardiogram (ECG)
interval with delta wave

Which of the following is the most appropriate long-term intervention?

A Amiodarone

B Atenolol

C Digoxin

D Flecainide

E Radiofrequency ablation

Explanation 

E Radiofrequency ablation
This patient has symptomatic Wolff–Parkinson–White (WPW) syndrome. Given his young age
and the need to commit to long-term anti-arrhythmic therapy, radiofrequency ablation is
usually considered here as the preferred intervention. Success rates in preventing
tachyarrhythmias are 95% or greater. The main risks include treatment failure and a small risk
of inducing heart block.

A Amiodarone

Significant long-term risks exist for amiodarone therapy, including photosensitisation,


pulmonary fibrosis and thyroid dysfunction. Given there are a number of alternative
approaches to treating this patient’s arrhythmia, amiodarone is not a preferred intervention
here.

B Atenolol

Beta-blockers are a common second-line intervention for WPW, although they are not
advised for patients who have or may develop pre-excited atrial fibrillation.

C Digoxin

Digoxin will not maintain this patient in sinus rhythm. It is usually reserved for patients who
do not tolerate beta blockade who have chronic atrial fibrillation and require control of their
ventricular rate. It is also used in the management of end-stage cardiac failure where other
options have been exhausted.

D Flecainide

Flecainide is used in the prevention of atrial fibrillation and paroxysmal supraventricular


tachycardia. It should not be used in patients with structural heart disease or in those with
myocardial ischaemia. It would be a potential intervention in patients who are not interested
in radiofrequency ablation.
51676

Rate this question:

Next Question

Previous Question Tag Question

Feedback End Review

Difficulty: Average

Peer Responses %
Q. Answered Flagged

Q1

Q2

Q3

Q4

Q5

Q6

Q7

Q8

Q9

0:00:17/03:00:00

An 18-year-old man is admitted to the Emergency Department with right upper quadrant
abdominal pain, nausea and vomiting. He tells you he is chronically anaemic and has a
problem with his red blood cells that runs in the family, and that episodes of abdominal pain
have increased over the past few months.
He looks pale and is pyrexial 37.9 °C. His blood pressure is 105/72 mmHg. Abdominal
palpation confirms tenderness in the right upper quadrant, and you also note that he has
splenomegaly.
Investigations:

9.8 g/dl (spherocytes identified


Haemoglobin (Hb) (13.5–17.5 g/dl)
on film)

White cell count (WCC) 15.2 × 10 9/l (4–11 × 10 9/l)

Platelets (PLT) 201 × 10 9/l (150–400 × 10 9/l)

Sodium (Na +) 144 mmol/l (135–145 mmol/l)

Potassium (K +) 4.9 mmol/l (3.5–5.0 mmol/l)

Creatinine (Cr) 112 µmol/l (50–120 µmol/l)

C-reactive protein (CRP) 220 mg/l (0–10 mg/l)

Alanine aminotransferase (ALT) 89 iu/l (5–20 iu/l)

Alkaline phosphatase 322 iu/l (30–130 iu/l)

Bilirubin 22 µmol/l (2–17 µmol/l)

Which of the following is the most likely cause of his right upper quadrant pain?

A Budd–Chiari syndrome

B Cholecystitis

C Epstein–Barr virus infection

D Gilbert syndrome

E Hepatitis A
Explanation 

B Cholecystitis

This patient has hereditary spherocytosis, which is associated with chronic haemolysis,
increased reticulocytes and spherocytes identified on the blood film. The disease is caused
by a defect in red cell membrane proteins, most commonly a combined spectrin and ankyrin
deficiency. It is likely this chronic haemolysis has led to the formation of bile pigment
gallstones, which, in turn, have precipitated an episode of acute cholecystitis. The risk of
gallstones associated with cholecystitis is abolished by splenectomy.

A Budd–Chiari syndrome

Budd–Chiari syndrome is associated with thrombophilia and leads to dull right upper
quadrant pain and the development of progressive portal hypertension and ascites, rather
than the picture of acute infection seen here.

C Epstein–Barr virus infection

This presents with an acute hepatitis picture, rather than the evidence of hepatic obstruction
seen here on liver function testing. It is also associated with acute pharyngitis. In patients
with spherocytosis, Epstein–Barr virus infection may trigger an acute haemolytic crisis.

D Gilbert syndrome

Gilbert syndrome presents with painless jaundice at the time of an intercurrent illness such as
an episode of influenza. Isolated hyperbilirubinaemia is seen on liver function testing. Where
it exists concurrently with hereditary spherocytosis, it can increase the risk of gallstones 4-
fold.

E Hepatitis A

There are no risk factors reported here for transmission of acute viral hepatitis and no
prodromal illness typical of the condition, and there is an obstructive, rather than a hepatitic,
picture seen on liver function testing.
51688

Rate this question:

Next Question

Previous Question Tag Question


Feedback End Review

Difficulty: Average

Peer Responses %

Q. Answered Flagged

Q1

Q2

Q3

Q4

Q5

Q6

Q7

Q8

Q9

0:00:17/03:00:00

An 18-year-old woman who is sitting her end-of-school exams is reviewed in the First Seizure
Clinic. She suffered a generalised tonic–clonic seizure some two to three weeks earlier, while
walking to school. You understand from the patient and her mother that she has suffered
from increasing clumsiness affecting the upper limbs over the past few months, particularly in
the mornings when she seems to suffer from involuntary jerking. They feel that the jerky limb
movements are worse if she is sleep-deprived. She has normal school attainment and is
planning to attend university. Apart from the neurological symptoms, she is otherwise well,
although she has struggled with weight gain.

Neurological examination in the clinic is entirely normal. An electroencephalogram (EEG)


recorded the previous week shows 4- to 6-Hz bilateral polyspike and slow-wave discharges
with frontal predominance.

From which of the following therapies is she most likely to benefit?

A Carbamazepine

B Leviteracetam

C Phenytoin

D Sodium valproate

E Vigabatrin

Explanation 

B Leviteracetam

Leviteracetam is the intervention of choice in this patient who is suffering from juvenile
myoclonic epilepsy (JME), as evidenced by the symptoms of involuntary muscular jerks,
worst in the morning, generalised tonic–clonic seizure, and the typical appearance for JME on
the EEG. In patients with JME, leviteracetam has shown very high rates of symptom
resolution and does not result in weight gain (in contrast to sodium valproate).

A Carbamazepine
Carbamazepine can accentuate absences in patients with JME and is therefore not an
appropriate option here. Carbamazepine is usually considered a treatment option in patients
with complex partial seizures.

C Phenytoin

Like carbamazepine, phenytoin can exacerbate absences in patients with JME, and its long-
term side-effect profile is considered inappropriate for an 18-year-old woman when
alternatives, such as leviteracetam, exist. Significant side-effects associated with phenytoin
therapy include peripheral sensory polyneuropathy, osteomalacia and gum hypertrophy.

D Sodium valproate

Sodium valproate is effective in treating patients with JME, although it leads to weight gain
and symptoms of a polycystic ovarian syndrome (PCOS)-like syndrome. It is therefore
inappropriate in this patient who already suffers from symptoms of weight gain.

E Vigabatrin

Vigabatrin has been reported to cause absence status in patients with JME and is therefore
contraindicated for the condition. Compared with other options, its adverse event profile
would also be considered inappropriate, with weight gain, psychiatric disturbance and optic
atrophy all potential adverse events.
51694

Rate this question:

Next Question

Previous Question Tag Question

Feedback End Review

Difficulty: Average

Peer Responses %
Q. Answered Flagged

Q1

Q2

Q3

Q4

Q5

Q6

Q7

Q8

Q9

0:00:17/03:00:00

A 21-year-old student comes to the Infectious Diseases Clinic a few weeks after returning
from holiday to India. He has been suffering from night sweats three to four times per week,
vague right upper quadrant pains and diarrhoea which is often bloodstained and contains
mucus. He has lost some 4 kg in weight.
Examination reveals a blood pressure of 102/80 mmHg, and a pulse of 70 bpm and regular.
His body mass index is 21.5 kg/m 2. There is mild abdominal distension and pain in the right
upper quadrant on palpation.
Investigations:

Haemoglobin (Hb) 10.9 g/dl (11.5–15.5 g/dl)

White cell count (WCC) 14.5 × 10 9/l (4–11 × 10 9/l)

Platelets (PLT) 203 × 10 9/l (150–400 × 10 9/l)

Sodium (Na +) 142 mmol/l (135–145 mmol/l)

Potassium (K +) 3.5 mmol/l (3.5–5.0 mmol/l)

Creatinine (Cr) 102 µmol/l (50–120 µmol/l)

Alanine aminotransferase (ALT) 104 iu/l (5–30 iu/l)

Alkaline phosphatase 289 iu/l (30–130 iu/l)

Bilirubin 12 µmol/l (2–17 µmol/l)

Albumin 32 g/l (35–55 g/l)

Which of the following is the most likely diagnosis?

A Escherichia coli (EC) infection

B Entamoeba histolytica (EH) infection

C Giardia lamblia (GL) infection

D Salmonella typhi (ST) infection

E Shigella sonnei (SS) infection


Explanation 

B Entamoeba histolytica (EH) infection

The subacute course to this man’s symptoms, accompanied by evidence of active colitis and
possible hepatic infection (abnormal liver function tests), coupled with travel to India, hints at
amoebiasis. Stool antigen testing and polymerase chain reaction (PCR) kits are available to
confirm the diagnosis. Sequential treatment with metronidazole, followed by a course of
diloxanide, to achieve eradication of the parasite is the intervention of choice. Surgical
drainage of any liver abscess is not required.

A Escherichia coli (EC) infection

E. coli infection may lead to diarrhoea shortly after arriving in a foreign country, but
symptoms are usually short-lived, resolving after a few days, with no intervention required.
For patients who travel frequently and suffer repeated travellers’ diarrhoea, antibiotics may
shorten the period of symptoms (both quinolones and azithromycin are used as
interventions).

C Giardia lamblia (GL) infection

GL infection leads to irritable bowel syndrome like symptoms with intermittent abdominal
bloating and diarrhoea. It would not be expected to lead to the abnormal liver function tests
seen here. It can be treated with a single dose of tinidazole or a short course of
metronidazole.

D Salmonella typhi (ST) infection

Salmonella is associated with bloody diarrhoea in some cases, but infection is usually self-
limiting and resolves within four to seven days, rather than the prolonged period of
symptoms seen here. For older patients and those who are immunocompromised,
intervention with antibiotics (quinolones) may be considered. Chronic shedding of Salmonella
is occasionally seen, and where patients with ST infection work in the food industry, they
should provide a negative stool sample before returning to work.

E Shigella sonnei (SS) infection

SS infection leads to bloody diarrhoea and symptoms of dysentery. Outbreaks are usually
seen in institutions, such as day care nurseries, and infection may be acquired with foreign
travel, although a shorter time course of symptoms would be expected vs that seen here.
51735

Rate this question:


Next Question

Previous Question Tag Question

Feedback End Review

Difficulty: Average

Peer Responses %

Q. Answered Flagged

Q1

Q2

Q3

Q4

Q5

Q6

Q7

Q8

Q9

0:00:17/03:00:00

A 37-year-old woman who has primary pulmonary hypertension comes to the clinic for
review. Her symptoms are stable on bosentan and amlodipine, and she is able to walk up to
half a mile on the flat without stopping. She tells you that she is eight weeks pregnant and
plans to continue the pregnancy.
Her blood pressure is 135/84 mmHg, and pulse 72 bpm and regular. Her chest is clear, and
there is minor pitting oedema affecting both ankles. Routine blood tests are unremarkable.
Which of the following is the most appropriate course of action?

A Advise her that a termination of pregnancy is essential

B Stop bosentan and continue amlodipine

C Stop bosentan and amlodipine

D Add sildenafil

E Switch bosentan for sildenafil

Explanation 

E Switch bosentan for sildenafil

In embryo/fetal toxicity studies in rats, endothelin receptor antagonists have been shown to
lead to malformations of the head, mouth, face and large blood vessels. As such, they should
be discontinued as soon as pregnancy is confirmed. In contrast, phosphodiesterase type 5
(PDE-5) inhibitors have not shown evidence of teratogenicity. Rather than risk symptoms
worsening significantly, it is therefore optimal to switch bosentan for sildenafil.

A Advise her that a termination of pregnancy is essential

This patient has stable symptoms and a relatively good functional status and should
therefore be given the option to continue her pregnancy. Given there are data to support the
use of calcium channel antagonists, PDE-5 inhibitors and inhaled prostaglandins in
pregnancy, she has significant alternative options for treatment to bosentan.

B Stop bosentan and continue amlodipine


Switching this patient from two therapies to one runs the risk of destabilising her symptoms
and a permanent worsening in/progression of her primary pulmonary hypertension. As such,
it is more appropriate to substitute the PDE-5 inhibitor in this situation.

C Stop bosentan and amlodipine

Stopping both therapies is inappropriate here. Calcium antagonists have a substantial body
of evidence in pregnant women to support their use, and moving from two therapies for
primary pulmonary hypertension to none risks a significant deterioration in her symptoms.

D Add sildenafil

Although she may require additional therapy during her pregnancy, continuing bosentan is
not appropriate because animal models have confirmed significant risk of teratogenicity.
51767

Rate this question:

Next Question

Previous Question Tag Question

Feedback End Review

Difficulty: Average

Peer Responses %

Q. Answered Flagged

Q1

Q2

Q3
Q. Answered Flagged

Q4

Q5

Q6

Q7

Q8

0:00:17/03:00:00

A 25-year-old woman who has type 1 diabetes comes to the Diabetes Clinic for review. She is
20 weeks pregnant. She had been diagnosed with background diabetic retinopathy but now
presents with blurred vision affecting her left eye and a documented reduction in visual
acuity. Her blood pressure is 110/80 mmHg, and pulse 70 bpm and regular.
Fundoscopy reveals macular oedema affecting the left eye, with a circinate of hard exudates
and haemorrhages close to the macula. Her HbA1c is 6.2%, and she is using an insulin pump.
Which of the following is the most appropriate intervention?

A Decrease insulin dose

B Focal photocoagulation (FP)

C Intravitreal ranibizumab

D Panretinal photocoagulation (PRP)

E Start ramipril

Explanation 

B Focal photocoagulation (FP)

This patient has sight-threatening macular oedema, which can be seen as patients intensify
their glycaemic control in pregnancy. In this case, with local changes affecting the left eye, FP
is the intervention of choice. At the very least, the National Institute for Health and Care
Excellence (NICE) guidelines recommend digital retinal assessment at the first antenatal visit
to look for changes in retinopathy which may require treatment. Intravitreal corticosteroids
have also been used to treat macular oedema, although they are not thought to be as
effective as laser therapy over the longer term.

A Decrease insulin dose

NICE guidelines on management of diabetes in pregnancy are clear that diabetes control
should not be compromised because of the risk of increasing birth defects and complications
of late pregnancy associated with diabetes.
C Intravitreal ranibizumab

Ranibizumab is an anti-vascular endothelial growth factor (VEGF) Fab fragment, which is


used for intravitreal treatment of macular oedema. There are no controlled data to support
the use of anti-VEGF therapies in pregnancy, and with alternative approaches such as laser
available, its use is not recommended.

D Panretinal photocoagulation (PRP)

PRP is indicated for proliferative retinopathy; the local changes seen here can be managed
with FP only.

E Start ramipril

Although angiotensin-converting enzyme (ACE) inhibitors are indicated to reduce the risk of
microvascular complications of diabetes mellitus, they are contraindicated in pregnancy
because of an increased risk of renal tract malformations.
51757

Rate this question:

Next Question

Previous Question Tag Question

Feedback End Review

Difficulty: Average

Peer Responses %

Q. Answered Flagged

Q1
Q. Answered Flagged

Q2

Q3

Q4

Q5

Q6

Q7

Q8 

 External Links

National Institute for Health and Care Excellence. 2015. Diabetes in pregnancy: management from precon…
nice.org.uk/guidance/ng3/resources/diabetes-in-pregnancy-management-from-preconception-to-the-postnatal-peri…
(https://www.nice.org.uk/guidance/ng3/resources/diabetes-in-pregnancy-
management-from-preconception-to-the-postnatal-period-51038446021)
0:00:17/03:00:00

A 27-year-old woman is brought to the Emergency Department by her husband. He is


concerned, as she has become increasingly confused and drowsy over the past week, with
poor short-term memory and a deterioration in her walking with a tendency to trip over. She
was previously well and takes no regular medication; she returned from a business trip to
Tokyo some three weeks ago.
Examination reveals pyrexia of 38.2 °C, and her blood pressure is 105/80 mmHg. She has an
ataxic gait. You can demonstrate both expressive dysphasia and short-term memory loss.
Investigations:

Haemoglobin (Hb) 12.8 g/dl (11.5–15.5 g/dl)

White cell count (WCC) 8.1 × 10 9/l (4–11 × 10 9/l)

Platelets (PLT) 209 × 10 9/l (150–400 × 10 9/l)

Sodium (Na +) 132 mmol/l (135–145 mmol/l)

Potassium (K +) 4.5 mmol/l (3.5–5.0 mmol/l)

Creatinine (Cr) 103 µmol/l (50–120 µmol/l)

Glucose 6.1 mmol/l (3.5–5.5 mmol/l)

Increased signal intensity


Magnetic resonance imaging (MRI) in the right temporal lobe
and insular cortex

Cerebrospinal fluid (CSF) protein 1.2 g/l (< 0.45 g/l)

CSF Lymphocytosis

Which of the following is the most likely diagnosis?

A Herpes simplex encephalitis

B Japanese encephalitis

C Mycoplasma meningitis

D Subacute sclerosing panencephalitis

E Tuberculous meningitis
Explanation 

A Herpes simplex encephalitis

The scenario here with a gradual decrease in consciousness, short-term memory loss and gait
disturbance fits well with a diagnosis of herpes simplex encephalitis. The MRI changes, with
one temporal lobe affected, also fits well with the diagnosis, as does the CSF lymphocytosis
and elevated CSF protein. Polymerase chain reaction (PCR) is indicated to confirm the
diagnosis, with intravenous (IV) aciclovir as the intervention of choice.

B Japanese encephalitis

The MRI appearance in Japanese encephalitis is markedly different to that seen here, with
lesions in the thalami, substantia nigra, basal ganglia, cerebral cortex, cerebellum, brainstem
and white matter. The incubation period for Japanese encephalitis is five to 15 days, which
does not fit with the scenario here.

C Mycoplasma meningitis

Central nervous system (CNS) Mycoplasma infection seems unlikely, given the absence of
other features consistent with Mycoplasma infection such as pneumonia or evidence of ear
infection. When Mycoplasma CNS infection does occur, it can lead to encephalitis or to
aseptic meningitis.

D Subacute sclerosing panencephalitis

This occurs as a consequence of measles infection and is associated with more global
changes on MRI; it is also associated with a more prolonged course of symptoms with
respect to cognitive impairment.

E Tuberculous meningitis

Tuberculous meningitis is associated with lymphocytic meningitis, although the time course
of symptoms is usually longer than that seen here. Chronic symptoms of respiratory infection
would be expected, and changes confined to the temporal lobe would be highly unusual.
51744

Rate this question:

Next Question
Previous Question Tag Question

Feedback End Review

Difficulty: Average

Peer Responses %

Q. Answered Flagged

Q1

Q2

Q3

Q4

Q5

Q6

Q7

Q8

Q9

0:00:17/03:00:00

A 42-year-old man comes to the clinic for review. He takes allopurinol 300 mg per day for
the treatment of gout, yet he has still had two acute attacks of gout over the past four
months, despite also taking naproxen. His only other medication is ramipril for hypertension.
His blood pressure is 132/82 mmHg, and pulse 72 bpm and regular. His body mass index
(BMI) is 26 kg/m 2.
Investigations:

Haemoglobin (Hb) 13.0 g/dl (13.5–17.5 g/dl)

White cell count (WCC) 7.2 × 10 9/l (4–11 × 10 9/l)

Platelets (PLT) 281 × 10 9/l (150–400 × 10 9/l)

Sodium (Na +) 145 mmol/l (135–145 mmol/l)

Potassium (K +) 4.9 mmol/l (3.5–5.0 mmol/l)

Creatinine (Cr) 102 µmol/l (50–120 µmol/l)

Uric acid 9.36 mmol/l (4.0–8.5 mg/dl)

Which of the following is the most appropriate intervention?

A Bendroflumethiazide

B Benzbromarone

C Colchicine

D Febuxostat

E Rasburicase

Explanation 

B Benzbromarone
Benzbromarone is an inhibitor of uric acid transporter 1 (URAT1) and a non-competitive
xanthine oxidase inhibitor. It reduces uric acid on top of allopurinol or febuxostat and can be
used in the United Kingdom on a named patient basis. This is because it was withdrawn from
general sale during the 1970s due to hepatotoxicity. In the event it is started in this patient,
regular monitoring of liver function tests is essential.

A Bendroflumethiazide

Thiazide diuretics are recognised to increase serum uric acid. As such, in this situation, they
may well worsen episodes of gout, rather than prevent them. Where patients present with
recurrent gout and they are taking a thiazide, the medication should be discontinued, if
possible.

C Colchicine

Colchicine is more commonly used as an acute treatment for gout in patients who are
unsuitable for therapy with either non-steroidal anti-inflammatory drugs (NSAIDs) or
corticosteroids. The dose is usually increased in 500-microgram increments until control of
symptoms is gained.

D Febuxostat

Febuxostat is an alternative to allopurinol. Trials of febuxostat in gout have shown a


numerical excess of cardiovascular events. As such, it is usually used in patients with gout
who cannot tolerate allopurinol.

E Rasburicase

Rasburicase is recombinant urate oxidase. It is used as a preventative therapy for uraemic


nephropathy in patients with haematological malignancy who are undergoing induction
chemotherapy where white cell death can lead to a precipitous increase in serum urate.
51775

Rate this question:

Next Question

Previous Question Tag Question

Feedback End Review

Difficulty: Average

Peer Responses %
Q. Answered Flagged

Q1

Q2

Q3

Q4

Q5

Q6

Q7

Q8

Q9

0:00:17/03:00:00

A 23-year-old woman is referred to the Dermatology Clinic with an intensely itchy vesicular
rash that has affected her buttocks, the backs of her thighs and the extensor surfaces of her
arms. She tells you she has lost weight and feels tired all the time, with intermittent diarrhoea
and abdominal bloating, although her general practitioner (GP) has told her this is down to
irritable bowel syndrome.
On examination, you can see an extensive vesicular rash, with obvious scratch marks. She is
slim with a body mass index of 21 kg/m 2.
Investigations:

Haemoglobin (Hb) 9.0 g/dl (macrocytosis) (11.5–15.5 g/dl)

White cell count (WCC) 7.2 × 10 9/l (4–11 × 10 9/l)

Platelet (PLT) 192 × 10 9/l (150–400 × 10 9/l)

Sodium (Na +) 142 mmol/l (135–145 mmol/l)

Potassium (K +) 3.9 mmol/l (3.5–5.0 mmol/l)

Creatinine (Cr) 98 µmol/l (50–120 µmol/l)

Which of the following is the most likely cause of her presentation?

A Coeliac disease

B Crohn’s disease

C Hypothyroidism

D Sarcoidosis

E Type 1 diabetes

Explanation 

A Coeliac disease
This patient’s rash is typical of that seen in patients with coeliac disease, with the multiple
vesicles particularly affecting the buttocks consistent with dermatitis herpetiformis. An anti-
tissue transglutaminase (anti-TTG) antibody test is the most appropriate next step in her
investigations and is likely to confirm the diagnosis. Absolute avoidance of gluten will
improve her anaemia, gastrointestinal symptoms and rash.

B Crohn’s disease

Crohn’s disease, which is likely to present with weight loss, abdominal pain, diarrhoea and
anaemia, is associated with a number of skin manifestations, including aphthous ulceration,
erythema nodosum (a raised red rash on the shins) and pyoderma gangrenosum (where a
papule grows rapidly into a shallow ulcer with a necrotic base). None of those fit with the
clinical picture seen here.

C Hypothyroidism

Hypothyroidism is associated with hair loss and a ‘doughy’ feel to the skin, not with the
vesicular rash seen here.

D Sarcoidosis

Like inflammatory bowel disease, sarcoidosis can present with a raised red rash affecting
both shins – erythema nodosum.

E Type 1 diabetes

Type 1 diabetes is associated with necrobiosis lipoidica, which begins as a red-brown plaque-
like lesion, usually on the lower limbs, and gradually enlarges, with the formation of yellow
atrophied skin at the base.
51714

Rate this question:

Next Question

Previous Question Tag Question

Feedback End Review

Difficulty: Average

Peer Responses %
Q. Answered Flagged

Q1

Q2

Q3

Q4

Q5

Q6

Q7

Q8

Q9

0:00:17/03:00:00

You review a 48-year-old man with suspected Cushing syndrome and plan to arrange cortisol
sampling to contribute to the diagnostic workup.
When are cortisol levels expected to be at their lowest in a normal individual?

A 0000 h

B 0500 h

C 0900 h

D 1200 h

E 1700 h

Explanation 

A 0000 h

Cortisol release is highest in the morning and lowest at midnight in normal individuals. Hence,
it is most appropriate to take the sample at 0000 h. Falsely elevated readings are possible,
however, with as little stress as having to have blood taken. Samples should be withdrawn
from an already placed intravenous (IV) line, with the patient as relaxed as possible. Salivary
cortisol levels are an appropriate alternative. A level of 350 nmol/l is generally used as the
upper limit of the normal range for midnight cortisol.

B 0500 h

Already by 0500 h, cortisol levels have begun to rise, meaning that at 0500 h, the sensitivity
to detect Cushing syndrome has already fallen significantly.

C 0900 h

0900 h is a time of peak awakening and is associated with the morning cortisol peak. This is
therefore the optimal time to screen for adrenal insufficiency, with a baseline cortisol level
and Synacthen ® test. The lower limit of normal for cortisol levels at 0900 h is 140 nmol/l, and
adrenal insufficiency is excluded in the Synacthen ® test by an incremental rise in cortisol of >
200 nmol/l and a 30-minute value of > 600 nmol/l.

D 1200 h

1200 h is in the middle of the day; hence, the sensitivity to detect Cushing syndrome by a
single cortisol sample is very low. The only situation when this may be useful is in permanent
night-shift workers where the normal release of cortisol may be reversed.

E 1700 h

Given most people are highly active at 1700 h, the chance of cortisol assay at 1700 h being
sensitive enough to detect Cushing syndrome is very low. Because of the difficulty taking
blood at midnight, and the unsuitability of other times in the day to take blood, 24-hour
urinary cortisol is usually preferred as a Cushing syndrome screening test.
51704

Rate this question:

Next Question

Previous Question Tag Question

Feedback End Review

Difficulty: Average

Peer Responses %

Q. Answered Flagged

Q1

Q2
Q. Answered Flagged

Q3

Q4

Q5

Q6

Q7

Q8
0:00:17/03:00:00

A 23-year-old woman is referred to the Gastroenterology Clinic for investigation of anaemia.


She complains of increasing tiredness over the past six months. She has suffered from
intermittent bloating and diarrhoea over the past three years. She has a normal menstrual
cycle, regulated by the combined oral contraceptive pill. Physical examination is
unremarkable. Her body mass index is 21 kg/m 2.
Investigations:

Haemoglobin (Hb) 9.2 g/dl (11.5–15.5 g/dl)

Mean corpuscular volume (MCV) 77.9 fl (76–98 fl)

White cell count (WCC) 6.1 × 10 9/l (4–11 × 10 9/l)

Platelets (PLT) 207 × 10 9/l (150–400 × 10 9/l)

Sodium (Na +) 143 mmol/l (135–145 mmol/l)

Potassium (K +) 4.5 mmol/l (3.5–5.0 mmol/l)

Creatinine (Cr) 102 µmol/l (50–120 µmol/l)

Alkaline phosphatase 189 iu/l (30–130 iu/l)

Calcium (Ca) 2.1 mmol/l (2.2–2.6 mmol/l)

Albumin 40 g/l (35–55 g/l)

Which of the following is the most appropriate next investigation?

A Anti-tissue transglutaminase (TTG) antibody testing

B Colonoscopy

C Endoscopy and duodenal biopsy

D Haptoglobin

E Hysteroscopy

Explanation 
A Anti-tissue transglutaminase (TTG) antibody testing

In young women with unexplained iron deficiency anaemia, guidelines recommend exclusion
of coeliac disease. The best way to do this is with anti-TTG antibody testing in the first
instance; total immunoglobulin A (IgA) measurement is also advised. In the event that the
patient has selective IgA deficiency, immunoglobulin G (IgG) TTG antibody testing will also
be required.

B Colonoscopy

There is no indication of colonic pathology, with absence of severe diarrhoea and no history
of passing blood or mucus rectally. Colonoscopy should be considered to rule out a right-
sided colonic lesion in the event that other investigations are negative.

C Endoscopy and duodenal biopsy

Duodenal biopsy is required to confirm the diagnosis once positive anti-TTG antibodies have
been confirmed. In the event that coeliac disease is diagnosed, a gluten-free diet is essential
and should lead to rapid resolution of anaemia.

D Haptoglobin

Haptoglobin decreases in response to haemolytic anaemia because its role is to bind to free
haemoglobin. Haptoglobin levels should not change significantly in patients with iron
deficiency anaemia due to coeliac disease.

E Hysteroscopy

This patient has a normal menstrual cycle. In the absence of heavy menses or a bulky uterus
on examination, there is no value in hysteroscopy in the investigation of this patient’s
anaemia.
51725

Rate this question:

Next Question

Previous Question Tag Question

Feedback End Review

Difficulty: Average
Peer Responses %

Q. Answered Flagged

Q1

Q2

Q3

Q4

Q5

Q6

Q7

Q8

Q9

0:00:17/03:00:00

A 42-year-old bank worker is admitted to the Emergency Department with extreme agitation
and central crushing chest pain. He admits to taking cocaine and is a regular user each day.
He also smokes 20 cigarettes per day.
His blood pressure is 155/95 mmHg, and pulse 105 bpm and regular. He has bilateral basal
crackles on auscultation of the chest, consistent with mild cardiac failure. He has been given
aspirin by the paramedics.
Investigations:

Haemoglobin (Hb) 12.9 g/dl (13.5–17.5 g/dl)

White cell count (WCC) 7.9 × 10 9/l (4–11 × 10 9/l)

Platelets (PLT) 303 × 10 9/l (150–400 × 10 9/l)

Sodium (Na +) 144 mmol/l (135–145 mmol/l)

Potassium (K +) 3.8 mmol/l (3.5–5.0 mmol/l)

Creatinine (Cr) 95 µmol/l (50–120 µmol/l)

Troponin Not elevated

Electrocardiogram (ECG) ST segment elevation in leads V1-V4

Which of the following is the most appropriate intervention for his chest pain?

A Atenolol

B Clopidogrel

C Diazepam

D Glyceryl trinitrate (GTN) infusion

E Primary percutaneous coronary intervention (PCI)

Explanation 

D Glyceryl trinitrate (GTN) infusion


This patient presents with symptoms of myocardial ischaemia against a background of
chronic cocaine abuse. Although the normal initial troponin does not exclude an acute
myocardial infarction, the ST segment elevation seen here fits better with coronary artery
spasm. This occurs in cocaine abuse through stimulation of alpha receptors within the
coronary vasculature. In this situation, a GTN infusion or calcium channel antagonists are
preferred to beta blockade.

A Atenolol

Beta-blockers are not recommended in patients with myocardial ischaemia who have abused
cocaine because of theoretical risks that unopposed alpha stimulation may actually worsen
coronary artery spasm and exacerbate the situation. Smaller studies do, however, support the
safe use of mixed alpha and beta blockers such as labetalol and carvedilol.

B Clopidogrel

Clopidogrel is used, in addition to aspirin, in patients who have suffered a myocardial


infarction. Platelet activation does occur in patients who abuse cocaine and may lead to
arterial occlusion, although aspirin given by the paramedics is likely to deal with any risk in
this situation.

C Diazepam

Diazepam may have a role in managing agitation, but it will not have an adequate effect in
treating myocardial ischaemia. As such, it could be considered an adjunct to GTN infusion or
use of calcium channel antagonists here.

E Primary percutaneous coronary intervention (PCI)

This patient’s troponin is not elevated, and the ST elevation here could be linked to coronary
artery spasm. As such, the initial intervention of choice is GTN infusion or calcium channel
antagonist therapy. PCI is usually considered in patients with persistent ST elevation, despite
medical intervention, where there is a clear history of cocaine use.
51759

Rate this question:

Next Question

Previous Question Tag Question

Feedback End Review

Difficulty: Average
Peer Responses %

Q. Answered Flagged

Q1

Q2

Q3

Q4

Q5

Q6

Q7

Q8

Q9

0:00:17/03:00:00

A 28-year-old woman is admitted from the Obstetric Clinic where she has attended for her
20-week check-up. Over the past two weeks, she has felt increasingly short of breath and
suffered from palpitations, particularly when she tries to exercise. A diastolic murmur was
noted at her booking visit, but this was thought to be non-significant.
Her blood pressure is 110/80 mmHg, and her pulse is 90 bpm (atrial fibrillation). There is a
mid-diastolic murmur best heard with the patient in the left lateral position, and there are
crackles at both bases, consistent with cardiac failure. Routine bloods are unremarkable.
Her general practitioner has started furosemide 40 mg, with little impact on her symptoms.

Which of the following is the intervention most likely to be successful?

A Bendroflumethiazide

B Bisoprolol

C Consideration of percutaneous mitral valvotomy (PMV)

D Ramipril

E Valsartan

Explanation 

C Consideration of percutaneous mitral valvotomy (PMV)

The murmur here, coupled with the development of atrial fibrillation and cardiac failure
during the pregnancy, raises the possibility of severe mitral stenosis. Therapeutic options are
limited in pregnancy, with long-acting nitrates and diuretics options to control heart failure,
and beta-blockers to control the ventricular rate. This makes PMV the optimal intervention
here. Contraindications include significant valve calcification and the presence of left atrial
thrombus; echocardiography is therefore essential before this is considered.

A Bendroflumethiazide

Thiazide diuretics should be avoided, if possible, during pregnancy and will offer little
additional benefit here on top of furosemide, at the risk of significant hyponatraemia.
B Bisoprolol

Bisoprolol is not the preferred beta-blocker for rate control of atrial fibrillation in pregnancy.
Highly selective beta-1 antagonist adrenal antagonists, such as metoprolol, are preferred.

D Ramipril

Angiotensin-converting enzyme (ACE) inhibitors and angiotensin II receptor blockers (ARBs)


are contraindicated in pregnancy because of their teratogenicity risk. Nitrates are the
vasodilators of choice in pregnancy.

E Valsartan

Like ACE inhibitors, ARBs such as valsartan are contraindicated in pregnancy because of risks
of teratogenicity. In this instance, if valvotomy is not indicated, then a long-acting nitrate
preparation should be added to her treatment regime.
51677

Rate this question:

Next Question

Previous Question Tag Question

Feedback End Review

Difficulty: Average

Peer Responses %

Q. Answered Flagged

Q1
Q. Answered Flagged

Q2

Q3

Q4

Q5

Q6

Q7

Q8
0:00:17/03:00:00

You are investigating a family for increased susceptibility to both rheumatoid arthritis and
type 1 diabetes. They are found to be positive for the major histocompatibility complex
(MHC) class II antigen DRB1.
Which of the following cells usually express MHC class II antigens?

A Cardiac myocytes

B Dendritic cells

C Goblet cells

D Neurones

E Red blood cells

Explanation 

B Dendritic cells

MHC class II antigens are expressed on antigen-presenting cells. They are responsible for
processing antigen and presenting them on their cell surface in a way that can be recognised
by T lymphocytes. They are vital for a successful adaptive immune response. Macrophages
and B cells are other examples of antigen-presenting cells.

A Cardiac myocytes

Cardiac myocytes express MHC class I antigens; these antigens are associated with peptide
fragments by proteasomal degradation, and when they are expressed by cells, they flag the
cells for immune surveillance.

C Goblet cells

Goblet cells are simple columnar epithelial cells which produce gel-forming mucins. They are
found in the respiratory, gastrointestinal and reproductive tracts and do not express MHC
class II antigens.
D Neurones

MHC class II antigens are seen in a small subpopulation of neurones when they are exposed
to interferon gamma. They are not usually expressed by neurones, which may express class I
antigens in response to viral infection.

E Red blood cells

Red blood cells do not possess either MHC class I or MHC class II antigens. This is because
mature red blood cells do not possess a nucleus.
51698

Rate this question:

Next Question

Previous Question Tag Question

Feedback End Review

Difficulty: Average

Peer Responses %

Q. Answered Flagged

Q1

Q2

Q3

Q4

Q5
Q. Answered Flagged

Q6

Q7

Q8 
0:00:17/03:00:00

A 29-year-old woman is reviewed in the Sexually Transmitted Infections Clinic. She has
suffered left iliac fossa pain and pain on passing urine over the past few days. She is 20
weeks pregnant and has recently separated from her partner because he has admitted to an
extra-marital affair. A vulvovaginal swab sent for nucleic acid amplification testing (NAAT) is
positive for Chlamydia.
Which of the following is the most appropriate antibiotic intervention?

A Amoxicillin

B Azithromycin

C Erythromycin

D Ofloxacin

E Trimethoprim

Explanation 

B Azithromycin

The World Health Organization (WHO) and British Association for Sexual Health and HIV
(BASHH) guidelines recommend a single 1 g dose of azithromycin for the treatment of
Chlamydia infection in pregnancy. Women who are treated in pregnancy should have a test of
cure some three weeks after completing antibiotic therapy.

A Amoxicillin

Amoxicillin is an alternative to azithromycin where macrolides are contraindicated. Its use


should, however, be discussed with a microbiologist because it can induce latency and
reactivation of Chlamydia infection later.

C Erythromycin
Erythromycin administered twice daily for 14 days is an alternative to azithromycin, although
it is considered second line because of compliance issues with repeated-dose macrolides due
to gastrointestinal side-effects.

D Ofloxacin

Quinolones are contraindicated in pregnancy because of tendon rupture seen in juvenile


toxicity models. A warning from the Committee on Safety of Medicines (CSM) has also
expressed the need to reduce the use of quinolones in adults who are not pregnant.

E Trimethoprim

Trimethoprim was trialled as a therapy for Chlamydia during the 1980s. Even then, it was
found to be significantly less effective than tetracyclines, with a cure rate of only around 50%.
51740

Rate this question:

Next Question

Previous Question Tag Question

Feedback End Review

Difficulty: Average

Peer Responses %

Q. Answered Flagged

Q1

Q2

Q3
Q. Answered Flagged

Q4

Q5

Q6

Q7

Q8 
0:00:17/03:00:00

An 18-year-old man comes to the Renal Clinic having been admitted with an Escherichia coli
urinary tract infection (UTI) some four weeks earlier. An ultrasound has shown a shrunken left
kidney with evidence of scarring. His general practitioner (GP) has noted borderline
hypertension, and on further questioning, it transpires the patient did suffer from recurrent
UTIs in childhood. His blood pressure is 145/88 mmHg, and pulse 75 bpm and regular.
Abdomen is soft and non-tender. His body mass index is 22 kg/m 2.
Investigations:

Haemoglobin (Hb) 11.5 g/dl (13.5–17.5 g/dl)

White cell count (WCC) 7.2 × 10 9/l (4–11 × 10 9/l)

Platelets (PLT) 292 × 10 9/l (150–400 × 10 9/l)

Sodium (Na +) 144 mmol/l (135–145 mmol/l)

Potassium (K +) 4.5 mmol/l (3.5–5.0 mmol/l)

Creatinine (Cr) 133 µmol/l (50–120 µmol/l)

Which of the following is the most appropriate next step?

A Computed tomography (CT) abdomen and pelvis with contrast

B Long-term trimethoprim therapy

C Micturating cystourethrogram

D Radionucleotide cystogram

E Repeat urine microscopy culture and sensitivity

Explanation 

C Micturating cystourethrogram

The suspicion here is that the patient has suffered from recurrent vesicoureteric reflux, which
has led to left renal scarring and hypertension. Reflux usually resolves by the time patients
reach adulthood, although in this patient’s case, it is important to establish whether this UTI
is associated with functional reflux. In the event that it is, surgery to correct this may be
required. The best way to establish the diagnosis is via a micturating cystourethrogram.

A Computed tomography (CT) abdomen and pelvis with contrast

Although this may demonstrate anatomical abnormalities of the renal tract, it will not
demonstrate whether these result in functional reflux. As such, a micturating
cystourethrogram is preferred.

B Long-term trimethoprim therapy

It is important to confirm whether functional reflux is occurring, or not, in this patient, and if it
is, correction is preferred to long-term antibiotic therapy because of the risk of resistant
strains of bacteria emerging.

D Radionucleotide cystogram

A radionucleotide cystogram is usually employed in children for investigation of reflux. It


provides less anatomic detail vs a micturating cystourethrogram and is considered an inferior
investigation.

E Repeat urine microscopy culture and sensitivity

It is quite possible that a repeat urine specimen will be negative. This will not exclude the
possibility of a future UTI nor will it exclude reflux, and as such, it is not an appropriate option
here.
51753

Rate this question:

Next Question

Previous Question Tag Question

Feedback End Review

Difficulty: Average

Peer Responses %
Q. Answered Flagged

Q1

Q2

Q3

Q4

Q5

Q6

Q7

Q8

Q9

0:00:17/03:00:00

A 25-year-old woman comes to the clinic for review. Her brother has been diagnosed with
cystic fibrosis (CF). Although she is well, she has recently undergone genetic testing and has
been found to be a carrier for the delta-F508 mutation. Carrier frequency for the mutation in
the local area is around 1 in 25.
Assuming she has a partner from the local area, what is her risk of having a child with CF?

A 1 in 2

B 1 in 25

C 1 in 50

D 1 in 100

E 1 in 2500

Explanation 

D 1 in 100

CF is an autosomal recessive disorder. Hence, her chance of passing on her defective gene is
1 in 2. An untested partner from the area has a 1 in 25 chance of carrying the gene, and if they
do, a 1 in 2 chance of passing it on, making this 1 in 50; 1 in 2 multiplied by 1 in 50 makes 1 in
100.

A 1 in 2

This would be the chance of a child inheriting CF if the disease were autosomal dominant,
which it is not. The woman’s chance of passing on an affected gene must be multiplied by the
man’s chance of passing on an affected gene, to calculate the percentage risk of CF in a
child.

B 1 in 25

This is the carrier frequency for CF and does not represent the chances of a couple having an
affected child.
C 1 in 50

A 1 in 50 chance is the chance of an unselected person from the area carrying the gene for
CF and passing it on to an affected child (1 in 25 multiplied by 1 in 2).

E 1 in 2500

One in 2500 is the general prevalence of CF in live births in the United Kingdom. It is
calculated by multiplying together the carrier probabilities of two unselected individuals,
multiplied by their chances of passing on an affected gene (1 in 25 × 1 in 2 × 1 in 25 × 1 in 2).
51763

Rate this question:

Next Question

Previous Question Tag Question

Feedback End Review

Difficulty: Average

Peer Responses %

Q. Answered Flagged

Q1

Q2

Q3

Q4

Q5
Q. Answered Flagged

Q6

Q7

Q8 
0:00:17/03:00:00

A 19-year-old man presents to the Rheumatology Clinic with progressively worsening lumbar
spine pain which has increased in severity over the past four months. He has tried taking
paracetamol, although this is no longer controlling his symptoms. He has also been
diagnosed with bilateral plantar fasciitis.
Clinical examination reveals bilateral tenderness over the sacroiliac joints and limited forward
and lateral flexion of the spine.
Investigations:

Haemoglobin (Hb) 13.2 g/dl (13.5–17.5 g/dl)

White cell count (WCC) 9.1 × 10 9/l (4–11 × 10 9/l)

Platelets (PLT) 239 × 10 9/l (150–400 × 10 9/l)

C-reactive protein (CRP) 65 mg/l (0–10 mg/l)

Sodium (Na +) 145 mmol/l (135–145 mmol/l)

Potassium (K +) 4.5 mmol/l (3.5–5.0 mmol/l)

Creatinine (Cr) 100 µmol/l (50–120 µmol/l)

Lumbar spine X-ray Sacroiliitis

Which of the following is the most appropriate intervention?

A Golimumab

B Infliximab

C Methotrexate

D Naproxen

E Prednisolone

Explanation 

D Naproxen
This patient’s clinical presentation fits well with a diagnosis of early ankylosing spondylitis.
Non-steroidal anti-inflammatory drugs (NSAIDs) are the backbone of initial therapy, coupled
with intensive physiotherapy to prevent permanent limitation of movement of the spine. In
the event that patients fail to respond to two NSAIDs, early progression to biological therapy
with anti-tumour necrosis factor (TNF) agents is recommended.

A Golimumab

Golimumab is a TNF-alpha inhibitor; it is one of the TNF antagonists which is endorsed by the
National Institute for Health and Care Excellence (NICE) guidelines on ankylosing spondylitis.
Etanercept and adalimumab are alternative TNF inhibitors. Patients should be screened for
tuberculosis (TB) before starting treatment, as the risk of reactivation is less than that seen in
rheumatoid arthritis.

B Infliximab

Infliximab is not recommended for the treatment of ankylosing spondylitis under current
NICE guidelines.

C Methotrexate

Methotrexate is the initial intervention of choice for the treatment of rheumatoid arthritis and
psoriatic arthritis. The clinical picture seen here fits better with a diagnosis of ankylosing
spondylitis where NSAIDs are the preferred initial option.

E Prednisolone

Patients with ankylosing spondylitis usually respond adequately to NSAIDs in the initial
stages of the disease, and the benefit–risk profile of long-term prednisolone is not considered
positive here. They may only have an impact on symptoms in the short term and probably
increase the risk of spinal osteoporosis.
51771

Rate this question:

Next Question

Previous Question Tag Question

Feedback End Review

Difficulty: Average

Peer Responses %
Q. Answered Flagged

Q1

Q2

Q3

Q4

Q5

Q6

Q7

Q8

Q9

 External Links

NICE CKS. Ankylosing Spondylitis.


cks.nice.org.uk/ankylosing-spondylitis
(https://cks.nice.org.uk/ankylosing-spondylitis)
0:00:17/03:00:00

A 32-year-old woman is admitted to the Acute Medical Ward suffering from pulmonary and
renal vasculitis. She is found to be suffering from granulomatosis with polyangiitis. A decision
is made to commence her on intravenous (IV) cyclophosphamide and methylprednisolone.
Which of the following correctly describes the primary mode of action of
cyclophosphamide?

A Bone marrow suppression

B Downregulation of acute phase protein production

C Inhibition of neutrophil migration

D T cell modulation

E Vascular endothelial growth factor (VEGF) inhibition

Explanation 

D T cell modulation

Cyclophosphamide is rapidly metabolised in the liver to the active metabolites 4-hydroxy-


cyclophosphamide and aldofosfamide (tautomeric form of 4-hydroxy-cyclophosphamide).
These exert their effects on cells which have low levels of alcohol dehydrogenase, which
include some subtypes of T lymphocytes, mainly CD4-positive lymphocytes and, to a lesser
extent, CD8-positive lymphocytes.

A Bone marrow suppression

Bone marrow stem cells have relatively high levels of alcohol dehydrogenase activity,
meaning that unselected downregulation of bone marrow activity is not usually seen.
Relatively high levels of cyclophosphamide can therefore be used in induction of
immunosuppressive therapy.

B Downregulation of acute phase protein production


Any effects of cyclophosphamide on the acute phase response would be considered
secondary to the effects of cyclophosphamide on lymphocyte function.

C Inhibition of neutrophil migration

A number of agents, including matrix metalloproteinase inhibitors, have been trialled as


inhibitors of neutrophil migration, although cyclophosphamide does not act through this
mechanism.

E Vascular endothelial growth factor (VEGF) inhibition

VEGF is a target in the treatment of macular degeneration, severe diabetic retinopathy and
solid tumours. Cyclophosphamide does not act via VEGF inhibition.
51695

Rate this question:

Next Question

Previous Question Tag Question

Feedback End Review

Difficulty: Average

Peer Responses %

Q. Answered Flagged

Q1

Q2

Q3
Q. Answered Flagged

Q4

Q5

Q6

Q7

Q8
0:00:17/03:00:00

A 45-year-old woman presents to the Dermatology Clinic for review. She complains of
increasing facial flushing which is becoming more permanent, mainly affecting her cheeks
and the area around her nose. She drinks ten glasses of wine per week but otherwise has no
significant past medical history.
Examination reveals erythema affecting both cheeks and the nasolabial folds, with small
pustules around the nose. In contrast to acne, the skin appears dry, rather than greasy.
Which of the following is the most appropriate initial intervention?

A Oral isotretinoin

B Oral oxytetracycline

C Pulsed light therapy

D Topical brimonidine

E Topical metronidazole

Explanation 

E Topical metronidazole

Topical metronidazole 0.75% is a commonly used initial intervention for rosacea, the most
likely diagnosis here. Dry skin with intermittent, becoming more permanent, facial flushing,
coupled with a pustulent rash around the nose is typical of the diagnosis, which occurs more
commonly in women than men and is most prevalent in middle age.

A Oral isotretinoin

Oral retinoids are used for rosacea which is resistant to other interventions, including topical
and oral antibiotics. One trial compared oral isotretinoin vs doxycycline for pustular rosacea
and showed greater improvement in flushing, nasal pustules and swelling and patient
satisfaction than the antibiotic alone.

B Oral oxytetracycline
Oral antibiotics are usually considered for patients who fail to respond to topical
interventions. A Cochrane review concluded that evidence is strongest for doxycycline.
Systemic antibiotics are thought to work primarily because of anti-inflammatory effects.

C Pulsed light therapy

Although laser therapy may have some benefit in managing telangiectasias, neither laser nor
pulsed light therapy are strongly supported by the Cochrane review.

D Topical brimonidine

Brimonidine is an alpha-2 agonist which, when used as topical therapy, can reduce episodes
of facial flushing. It may be less effective with respect to resolution of nasal pustules than
other interventions.

Further Reading:

Gollnick, H, Blume-Peytavi, U, Szabo, E L et al. 2010. Systemic isotretinoin in the treatment of


rosacea—doxycycline-and placebo-controlled, randomized clinical study. Journal der
Deutschen Dermatologischen Gesellschaft, 8, 505–515.
51712

Rate this question:

Next Question

Previous Question Tag Question

Feedback End Review

Difficulty: Average

Peer Responses %

Q. Answered Flagged
Q. Answered Flagged

Q1

Q2

Q3

Q4

Q5

Q6

Q7

Q8 
0:00:17/03:00:00

A 64-year-old man comes to the Cardiology Clinic for review. He has suffered two collapses
over the past three months. On the last occasion, he was attended by a first-aider, who noted
his pulse to be only around 20 bpm. He has ischaemic heart disease, for which he takes
ramipril 10 mg daily.
On examination in the clinic, his blood pressure is 110/70 mmHg and his pulse is 40 bpm. His
chest is clear, and there is minor pitting oedema of both ankles.
Investigations:

Haemoglobin (Hb) 13.2 g/dl (13.5–17.5 g/dl)

White cell count (WCC) 7.1 × 10 9/l (4–11 × 10 9/l)

Platelets (PLT) 302 × 10 9/l (150–400 × 10 9/l)

Sodium (Na +) 142 mmol/l (135–145 mmol/l)

Potassium (K +) 4.5 mmol/l (3.5–5.0 mmol/l)

Creatinine (Cr) 103 µmol/l (50–120 µmol/l)

Electrocardiogram (ECG) Mobitz type 2 heart block

Which of the following is the most appropriate intervention?

A Fludrocortisone

B Methyldopa

C Dual-chamber pacemaker implantation

D Salbutamol

E Thromboembolic deterrent (TED) stockings

Explanation 

C Dual-chamber pacemaker implantation


This patient has suffered two syncopal episodes, potentially due to episodes of complete
heart block (CHB). Given the resting ECG in the Cardiology Clinic shows Mobitz type 2 heart
block, the risk of permanent CHB is very high and intervention is required. Biventricular
pacing is also considered for patients with heart failure which is resistant to medical
intervention.

A Fludrocortisone

Fludrocortisone is used in the treatment of orthostatic hypotension which does not respond
to TED stockings or midodrine, an alpha-1 receptor agonist used to maintain blood pressure.

B Methyldopa

Methyldopa is an anti-hypertensive, used most commonly in the treatment of hypertension in


pregnancy. It is an alpha-2 agonist which has no role in the treatment of heart block.

D Salbutamol

Salbutamol is a beta-2 agonist. Although it may cause tachycardia, it has no role in the
treatment of Mobitz type 2 heart block. Salbutamol is used in the initial treatment of
hyperkalaemia, especially when intravenous access is difficult to obtain.

E Thromboembolic deterrent (TED) stockings

TED stockings are used in the treatment of orthostatic hypotension, and they do not have a
role in the management of syncopal episodes related to CHB.
51678

Rate this question:

Next Question

Previous Question Tag Question

Feedback End Review

Difficulty: Average

Peer Responses %
Q. Answered Flagged

Q1

Q2

Q3

Q4

Q5

Q6

Q7

Q8

Q9

0:00:17/03:00:00

A 36-year-old woman comes to the Rheumatology Clinic for review. She has rheumatoid
arthritis and is currently treated with methotrexate. Unfortunately, she has persistent joint
pain and swelling. A decision is made to add adalimumab to her treatment regime.
Which of the following correctly represents the mode of action of adalimumab?

A Anti-CD20

B Anti-interleukin 6 (IL6)

C Calcineurin inhibitor

D CTLA4 immunoglobulin

E Tumour necrosis factor (TNF) inhibitor

Explanation 

E Tumour necrosis factor (TNF) inhibitor

Adalimumab binds specifically to TNF and neutralises the biological function of TNF by
blocking its interaction with p55 and p75 cell surface TNF receptors. Adalimumab also
modulates biological responses that are induced or regulated by TNF, including changes in
the levels of adhesion molecules responsible for leukocyte migration (ELAM-1, VCAM-1 and
ICAM-1, with an IC 50 of 0.1–0.2 nM). It is used primarily in the treatment of rheumatoid
arthritis and inflammatory bowel disease.

A Anti-CD20

CD20 is a B cell receptor. Agents such as rituximab which bind to the CD20 receptor
downregulate B cell activity and antibody production. They are used in patients with
inflammatory diseases, including rheumatoid arthritis, who fail to respond to other
interventions.

B Anti-interleukin 6 (IL6)
Toclizumab is an antibody directed against the IL6 receptor. It has a role in the treatment of
rheumatoid arthritis and juvenile chronic arthritis. In rheumatoid arthritis, it is usually
considered for patients who fail to respond to anti-TNF-based therapies.

C Calcineurin inhibitor

Sirolimus and tacrolimus are examples of calcineurin inhibitors. They inhibit interleukin 2 (IL2)
production and receptor expression which leads to downregulation of T cell activity. They are
used in the treatment of inflammatory disorders and in the prevention of transplant rejection.

D CTLA4 immunoglobulin

Abatacept blocks T cell co-stimulation, leading to downregulation of inflammatory activity. In


patients with rheumatoid arthritis, abatacept is shown to downregulate levels of the soluble
IL2 receptor, IL6, rheumatoid factor, C-reactive protein and TNF-alpha.
51692

Rate this question:

Next Question

Previous Question Tag Question

Feedback End Review

Difficulty: Average

Peer Responses %

Q. Answered Flagged

Q1

Q2
Q. Answered Flagged

Q3

Q4

Q5

Q6

Q7

Q8
0:00:17/03:00:00

A 78-year-old woman, who is known to have type 2 diabetes, ischaemic heart disease and
chronic kidney disease stage 3 (CKD-3) renal impairment, is admitted to the Emergency
Department with nausea, vomiting and dehydration. According to her daughter, she has
become increasingly confused over the past 48 hours. Her blood pressure on admission is
115/80 mmHg, and pulse 70 bpm and regular. She looks pale. Her heart sounds are normal,
and there are bilateral crackles at both lung bases on auscultation, consistent with fluid
overload. Abdominal palpation is normal.
Investigations:

Haemoglobin (Hb) 10.0 g/dl (11.5–15.5 g/dl)

White cell count (WCC) 11.9 × 10 9/l (4–11 × 10 9/l)

Platelets (PLT) 203 × 10 9/l (150–400 × 10 9/l)

Sodium (Na +) 146 mmol/l (135–145 mmol/l)

Potassium (K +) 7.2 mmol/l (3.5–5.0 mmol/l)

Bicarbonate (HCO 3 –) 15 mmol/l (24–30 mmol/l)

Creatinine (Cr) 712 µmol/l (50–120 µmol/l)

Electrocardiogram (ECG) Broad, bizarre-shaped complexes

You administer calcium gluconate and attach her to a cardiac monitor.

Which of the following other interventions is most important?

A Intravenous (IV) insulin and dextrose

B IV sodium bicarbonate

C IV sodium chloride

D Oral calcium resonium

E Salbutamol nebuliser

Explanation 
A Intravenous (IV) insulin and dextrose

In this patient, urgent lowering of potassium level is required because the risk of ventricular
arrhythmia and subsequent cardiac arrest is considered very high. IV insulin and dextrose is
most effective at driving potassium into cells and is the most appropriate next step here. It
can be repeated after 30 minutes in the event that the potassium level is still elevated.
Glucose monitoring is, of course, essential to guard against hypoglycaemia. Driving
potassium into the intracellular space in effect buys time, so that the cause of renal failure
can be further evaluated and a decision made on the level of intervention.

B IV sodium bicarbonate

Although correction of acidosis may improve hyperkalaemia to a limited extent, when there
are some signs of fluid overload as here, it may actually worsen symptoms and increase the
urgency of progression to dialysis.

C IV sodium chloride

Some of this patient’s blood picture could be due to relative renal hypoperfusion, although
the crackles on auscultation of the lungs hint that giving normal saline may actually worsen
the picture with respect to gas exchange. Large-volume fluid resuscitation should be
potentially avoided until an accurate picture as to the central venous pressure can be
obtained.

D Oral calcium resonium

Calcium resonium is an ion exchange resin which reduces absorption of potassium and
increases the amount of potassium within the gut lumen. It takes some time to work and can
be associated with significant gastrointestinal adverse effects. It is not the acute therapy
which is required here.

E Salbutamol nebuliser

Salbutamol nebulisers are effective at lowering the potassium level quickly, although, in this
case, with IV access already established, insulin and glucose is the better option. Where the
potassium level is resistant to lowering with insulin and glucose, salbutamol nebuliser could
be added, although it may increase the risk of arrhythmias occurring.
51702

Rate this question:

Next Question
Previous Question Tag Question

Feedback End Review

Difficulty: Average

Peer Responses %

Q. Answered Flagged

Q1

Q2

Q3

Q4

Q5

Q6

Q7

Q8

Q9

0:00:17/03:00:00

A 62-year-old man is admitted to the Acute Medical Ward with suspected endocarditis. He
has felt unwell for the past few weeks with night sweats, lethargy and gradual weight loss. He
has no significant medical illnesses.
He has developed a pansystolic murmur, loudest at the apex, and an echocardiogram has
revealed vegetations on the valve. Blood cultures have confirmed the presence of
Streptococcus gallolyticus (Streptococcus bovis) infection.
Which of the following is the most important investigation?

A Colonoscopy

B Cystoscopy

C Dental examination

D Endoscopic retrograde cholangiopancreatography (ERCP)

E Upper gastrointestinal (GI) endoscopy

Explanation 

A Colonoscopy

S. bovis (also called S. gallolyticus) endocarditis is strongly associated with underlying colon
cancer. Case series put the association at around 60% when underlying pre-malignant
adenomas are included. There is no strong evidence that S. bovis identification has any
positive or negative impact on outcomes from the cancer. Where an initial colonoscopy is
negative, repeat colonoscopy is indicated after four to six months.

B Cystoscopy

Bladder pathogens are a rarer cause of endocarditis, and S. bovis is not a usual resident in
the urinary tract. A number of urinary pathogens have been recognised as causes of
endocarditis, including Klebsiella and Escherichia coli. Other urinary pathogens known to
cause endocarditis include Globicatella sanguinis, which may be confused on microscopy
with Streptococcus viridans.
C Dental examination

Dental caries are a common route or cause for bacterial endocarditis, although they usually
lead to S. viridans, rather than S. bovis, endocarditis.

D Endoscopic retrograde cholangiopancreatography (ERCP)

Although abnormalities of the biliary tree do result in an increased risk of S. bovis infection,
they are less likely to be the underlying problem, compared to colon cancer. Now that
magnetic resonance cholangiopancreatography (MRCP) is widely available, it, rather than
ERCP, is the logical next step in patients where colonoscopy is negative.

E Upper gastrointestinal (GI) endoscopy

Upper GI tract neoplasms are much less likely to be associated with S. bovis endocarditis
than colon cancer, meaning that upper GI endoscopy has less utility in this situation.
51738

Rate this question:

Next Question

Previous Question Tag Question

Feedback End Review

Difficulty: Average

Peer Responses %

Q. Answered Flagged

Q1
Q. Answered Flagged

Q2

Q3

Q4

Q5

Q6

Q7

Q8

 External Links

Boleij, A, Schaeps, R M J, Tjalsma, H. 2009. Association between Streptococcus bovis and colon cancer. J…
jcm.asm.org/content/47/2/516
(https://jcm.asm.org/content/47/2/516)
0:00:17/03:00:00

A 72-year-old woman comes to the clinic for review. She has suffered from increasing weight
loss, diarrhoea and episodes of facial flushing over the past five months. She has lost 5 kg in
weight. She has hypertension treated with ramipril 10 mg daily but is otherwise well. Her
blood pressure is 132/82 mmHg, and pulse 80 bpm and regular. Palpation of the abdomen
reveals 2-cm hepatomegaly. Her body mass index (BMI) is 19 kg/m 2.
Investigations:

Haemoglobin (Hb) 9.0 g/dl (11.5–15.5 g/dl)

White cell count (WCC) 8.2 × 10 9/l (4–11 × 10 9/l)

Platelets (PLT) 193 × 10 9/l (150–400 × 10 9/l)

Sodium (Na +) 142 mmol/l (135–145 mmol/l)

Potassium (K +) 3.3 mmol/l (3.5–5.0 mmol/l)

Creatinine (Cr) 92 µmol/l (50–120 µmol/l)

Alanine aminotransferase (ALT) 198 iu/l (5–30 iu/l)

Alkaline phosphatase 295 iu/l (30–130 iu/l)

Bilirubin 14 µmol/l (2–17 µmol/l)

Which of the following is the most useful next investigation?

A Anti-tissue transglutaminase (TTG) antibodies

B 24-hour urinary 5-hydroxyindoleacetic acid (HIAA)

C 24-hour urinary free cortisol

D 24-hour urinary catecholamines

E Insulin tolerance test

Explanation 

B 24-hour urinary 5-hydroxyindoleacetic acid (HIAA)


The episodes of facial flushing here, together with weight loss and diarrhoea and coupled
with abnormal liver function tests, fit well with a diagnosis of carcinoid syndrome. Urinary 5-
HIAA is the initial screening test of choice, and 24-hour excretion of > 25 mg provides strong
evidence for the diagnosis of carcinoid syndrome. Patients should be advised that bananas,
kiwi fruit, pineapple, plums and tomato products may interfere with measurements of 5-HIAA.

A Anti-tissue transglutaminase (TTG) antibodies

Anti-TTG antibodies are the investigation of choice for the diagnosis of coeliac disease, which
is suggested by diarrhoea and weight loss, but not by the facial flushing and abnormal liver
function seen here.

C 24-hour urinary free cortisol

The 24-hour urinary free cortisol test is the initial screening test of choice for Cushing
syndrome, which is characterised by weight gain, hypertension and hyperglycaemia, rather
than the picture seen here of weight loss and diarrhoea.

D 24-hour urinary catecholamines

The 24-hour urinary catecholamine test is the initial screening test of choice for
phaeochromocytoma, characterised by paroxysms of hypertension with associated
symptoms such as intermittent headaches. Hyperglycaemia may also be seen.

E Insulin tolerance test

Insulin tolerance testing is utilised in the evaluation of growth hormone deficiency. It has
largely been retired as an endocrine test because of risks of targeting hypoglycaemia to drive
up growth hormone release.
51718

Rate this question:

Next Question

Previous Question Tag Question

Feedback End Review

Difficulty: Average

Peer Responses %
Q. Answered Flagged

Q1

Q2

Q3

Q4

Q5

Q6

Q7

Q8

Q9

0:00:17/03:00:00

An obese 57-year-old man is reviewed in the Oncology Clinic, having been diagnosed with
gastric cancer. He smokes 15 cigarettes per day and likes to eat processed foods.
He asks about risk factors for gastric cancer, as he is worried about his three children.
Which of the following has the greatest impact on increasing the risk of gastric cancer?

A Chronic non-steroidal anti-inflammatory drug (NSAID) use

B Increased green vegetable consumption

C Obesity

D Processed meat consumption

E Smoking

Explanation 

E Smoking

A meta-analysis of cohort studies suggested that smoking increases the risk of gastric cancer
by around 50% (odds ratio 1.53). A prospective cohort study (EPIC) then established that the
risk diminishes after ten years of smoking cessation. Around 18% of cases of gastric cancer
are thought to be related to smoking.

A Chronic non-steroidal anti-inflammatory drug (NSAID) use

NSAIDs are thought to be protective against gastric cancer. A retrospective cohort study
suggested around 20% reduction in the incidence of gastric cancer for every year of NSAID
use.

B Increased green vegetable consumption

High intake of fruits is thought to be protective against gastric cancer, reducing the risk by
approximately 10%. No discernible protective effect of green vegetables has been seen,
although consumption of dietary fibre may reduce the risk of gastric cancer by up to 40%.
C Obesity

Obesity does increase the risk of gastric cancer, although to a lesser extent than smoking
(odds ratio 1.22). A risk increase is seen for body mass index (BMI) of > 25 kg/m 2, and the risk
of gastric cancer increases more with higher BMI.

D Processed meat consumption

Processed meat consumption of 30 g per day is thought to increase the risk of gastric cancer
by around 15%. The negative effect of processed meat consumption may be synergistic with
Helicobacter pylori positivity.
51732

Rate this question:

Next Question

Previous Question Tag Question

Feedback End Review

Difficulty: Average

Peer Responses %

Q. Answered Flagged

Q1

Q2

Q3

Q4

Q5
Q. Answered Flagged

Q6

Q7

Q8 
0:00:17/03:00:00

A 36-year-old man who has emigrated to the United Kingdom from Turkey comes to the
Rheumatology Clinic for review. He has suffered a third episode of anterior uveitis in the last
year. He also has recurrent oral and genital ulceration, with erythema nodosum affecting both
shins. Two years ago, he suffered a left below-knee deep vein thrombosis.
Examination in the clinic confirms the presence of aphthous ulcers and erythema nodosum
affecting both shins.
Investigations:

Haemoglobin (Hb) 10.0 g/dl (13.5–17.5 g/dl)

White cell count (WCC) 9.1 × 10 9/l (4–11 × 10 9/l)

Platelets (PLT) 352 × 10 9/l (150–400 × 10 9/l)

Sodium (Na +) 144 mmol/l (135–145 mmol/l)

Potassium (K +) 4.3 mmol/l (3.5–5.0 mmol/l)

Creatinine (Cr) 105 µmol/l (50–140 µmol/l)

C-reactive protein (CRP) 82 mg/l (0–10 mg/l)

Antiphospholipid antibody Positive

Which of the following human leukocyte antigen (HLA) types is most closely associated
with this patient’s disease?

A B*1502

B B27

C B*51

D DR2

E DR4

Explanation 

C B*51
B*51 is strongly associated with the development of Behçet syndrome, the most likely
diagnosis here, given the presence of recurrent uveitis, thrombophilia, aphthous ulceration
and erythema nodosum. It is also associated with mucocutaneous lymph node syndrome and
increased susceptibility to German measles infection. It is an HLA class I antigen and may
drive the development of autoimmune disease through antigen-dependent and independent
mechanisms.

A B*1502

B*1502 is associated with an increased risk of Stevens–Johnson syndrome in patients taking


carbamazepine who are of Han Chinese ethnic origin. The association is not found in
Caucasians, and the United States Food and Drug Administration (FDA) recommends testing
in patients of Chinese ethnic origin before initiating therapy.

B B27

B27’s carrier frequency is highest in people of Caucasian origin and lowest in those of
Japanese ethnic origin. It is associated with psoriasis, ankylosing spondylitis, inflammatory
bowel disease and reactive arthritis.

D DR2

DR2 positivity is associated with anti-glomerular basement membrane (GBM) disease,


systemic lupus erythematosus, multiple sclerosis and narcolepsy. It is also associated with
tuberculoid leprosy, ulcerative colitis in Japanese individuals, primary biliary cirrhosis and
autoimmune hepatitis.

E DR4

DR4 positivity is most strongly associated with rheumatoid arthritis, which is not suggested
here by the presence of aphthous ulceration. The other features could potentially fit with
rheumatoid arthritis as the underlying diagnosis. It is also associated with hydralazine-
induced systemic lupus erythematosus (SLE), immunoglobulin A (IgA) nephropathy and
polymyalgia rheumatica.
51769

Rate this question:

Next Question

Previous Question Tag Question

Feedback End Review


Difficulty: Average

Peer Responses %

Q. Answered Flagged

Q1

Q2

Q3

Q4

Q5

Q6

Q7

Q8

Q9

0:00:17/03:00:00

A 71-year-old woman comes to the Rheumatology Clinic for review. She has a T score of –4.5
and has suffered a left Colles’ fracture some two years earlier. Unfortunately, she has failed to
tolerate weekly risedronate due to symptoms of gastro-oesophageal reflux disease. She also
takes insulin for type 2 diabetes, making it difficult for her to fast for a period after taking the
medication. Routine bloods, including calcium, are in the normal range.
Which of the following is the most appropriate treatment for her osteoporosis?

A Calcitonin

B Calcium and vitamin D only

C Denosumab

D Risedronate

E Strontium ranelate

Explanation 

C Denosumab

This is a receptor activator of nuclear factor kappa-B (RANK) ligand inhibitor, which
downregulates osteoclast activity and pre-osteoclast to osteoclast maturation. It has a
significant advantage over bisphosphonates here because it is given as a six-monthly
subcutaneous injection. Like bisphosphonates, it is associated with osteonecrosis of the jaw
as a rare adverse event, although unlike bisphosphonates, it does not accumulate in bone and
requires a prolonged washout period. Teriparatide (synthetic parathyroid hormone analogue)
is an alternative option, although it requires daily injection.

A Calcitonin

Calcitonin is not used as a long-term therapy for osteoporosis because of an association


between calcitonin use and osteosarcoma. It is still used for acute treatment of
hypercalcaemia.

B Calcium and vitamin D only


Reducing this patient’s therapy to calcium and vitamin D only is inappropriate, given that
other options for bone protection exist. Calcium and vitamin D only is proven to be inferior to
denosumab and to teriparatide.

D Risedronate

Risedronate is a synthetic oestrogen receptor modulator, (SERM). It is not as effective as


bisphosphonates, teriparatide or denosumab at maintaining bone mineral density, although it
may be considered if these options are inappropriate or contraindicated. It has been shown
to reduce the risk of breast cancer in patients who use it.

E Strontium ranelate

Strontium ranelate is effective as a treatment for osteoporosis, although it has been shown to
increase the risk of venous thromboembolism and may increase cardiovascular risk. For this
reason, other options are preferred.
51774

Rate this question:

Next Question

Previous Question Tag Question

Feedback End Review

Difficulty: Average

Peer Responses %

Q. Answered Flagged

Q1

Q2
Q. Answered Flagged

Q3

Q4

Q5

Q6

Q7

Q8 
0:00:17/03:00:00

You are reviewing the distribution of bodyweight in a group of men vs a group of women
who have non-alcoholic steatohepatitis (NASH).
Which of the following is the most appropriate statistical test to look for differences in the
weight distribution between the two groups?

A Analysis of covariance (ANCOVA) test

B Paired t-test

C Unpaired t-test

D Spearman’s rank correlation coefficient

E Wilcoxon rank sum test

Explanation 

E Wilcoxon rank sum test

The Wilcoxon rank sum test or Mann–Whitney test is a non-parametric alternative to the
unpaired two-sample t-test, which can be used to compare two independent groups of
samples. It is used when data are not normally distributed. Here, in a population of men and
women with NASH, it is highly likely that the distribution will be skewed towards the upper
end of the weight scale (ie not normally distributed).

A Analysis of covariance (ANCOVA) test

This blends analysis of variance and regression and is used to determine whether the means
of a dependent variable are equal across an independent variable (usually a medical
intervention). It is not therefore a relevant option here.

B Paired t-test

The paired t-test is used to determine whether the mean difference between two different
sets of observations is zero. In a paired t-test, two measurements are made in the same
subject or patient, and the observations are expected to be normally distributed; hence, it is
not an option here.

C Unpaired t-test

An unpaired t-test can be used to compare the means of two unmatched groups, assuming
they are normally distributed. In this case, it is not likely that weights are normally distributed.

D Spearman’s rank correlation coefficient

Spearman’s rank correlation coefficient is used to compare the statistical dependence of two
variables by rank. It is a non-parametric test. An example of where it could be used is to look
at the correlation between intelligence quotient (IQ) and the number of hours spent playing
computer games.
51710

Rate this question:

Next Question

Previous Question Tag Question

Feedback End Review

Difficulty: Average

Peer Responses %

Q. Answered Flagged

Q1

Q2

Q3
Q. Answered Flagged

Q4

Q5

Q6

Q7

Q8
0:00:17/03:00:00

A 58-year-old man is referred to the Pharmacology Clinic for management of his


hypertension. He is currently managed with felodipine once a day, although his blood
pressure is still elevated at 155/94 mmHg. He has no significant past medical history, and his
body mass index (BMI) is 26 kg/m 2.
Cardiovascular examination in the clinic is unremarkable, apart from the raised blood
pressure.
Investigations:

Haemoglobin (Hb) 12.9 g/dl (13.5–17.5 g/dl)

White cell count (WCC) 6.4 × 10 9/l (4–11 × 10 9/l)

Platelets (PLT) 203 × 10 9/l (150–400 × 10 9/l)

Sodium (Na +) 144 mmol/l (135–145 mmol/l)

Potassium (K +) 4.9 mmol/l (3.5–5.0 mmol/l)

Creatinine (Cr) 89 µmol/l (50–120 µmol/l)

Bicarbonate (HCO 3) 25 mmol/l (24–30 mmol/l)

Glucose (fasting) 5.4 mmol/l (< 7 mmol/l)

Which of the following is the most appropriate next step in managing his blood pressure?

A Atenolol

B Doxazosin

C Hydrochlorthiazide

D Indapamide

E Ramipril

Explanation 

E Ramipril
In this situation, with failure to control blood pressure on a calcium antagonist alone, the
National Institute for Health and Care Excellence (NICE) recommends either an angiotensin-
converting enzyme (ACE) inhibitor or a low-cost angiotensin II receptor blocker (ARB) as the
next step in therapy. Potassium and creatinine monitoring is recommended around two
weeks after initiating therapy with these classes, in case of undiagnosed renovascular
disease.

A Atenolol

Beta-blockers are generally not recommended early in the management of hypertension,


unless there is strong evidence of increased sympathetic drive. They were shown in a meta-
analysis of large anti-hypertensive trials to impair glucose handling, which has reduced their
utility earlier in the treatment pathway. They may be used earlier in the management of
hypertension for patients with coexisting angina.

B Doxazosin

Alpha-blockers are generally not recommended early in the management of hypertension


because they can cause tachycardia and postural hypotension. It has also been shown in
large trials that they may exacerbate cardiac failure.

C Hydrochlorthiazide

Like beta-blockers, thiazide diuretics may worsen glucose tolerance. They are also associated
with symptomatic hyponatraemia, particularly in the elderly population.

D Indapamide

Indapamide is a thiazide like diuretic which is not thought to impair glucose handling and is
less likely to cause hyponatraemia than traditional thiazides. The class of thiazide-like
diuretics is commonly used third line in patients whose blood pressure is not controlled on an
ACE or ARB combined with a calcium channel antagonist.
51680

Rate this question:

Next Question

Previous Question Tag Question

Feedback End Review

Difficulty: Average
Peer Responses %

Q. Answered Flagged

Q1

Q2

Q3

Q4

Q5

Q6

Q7

Q8

Q9

 External Links

National Institute for Health and Care Excellence. 2019. Treatment steps for hypertension.
pathways.nice.org.uk/pathways/hypertension/treatment-steps-for-hypertension#content=view-node%3Anodes-step…
(https://pathways.nice.org.uk/pathways/hypertension/treatment-steps-for-
hypertension#content=view-node%3Anodes-step-1-ace-inhibitor-or-low-cost-
angiotensin-ii-receptor-blocker)
0:00:17/03:00:00

A 32-year-old woman comes to the Respiratory Clinic for review. She has been treated for
the past three months with sildenafil for primary pulmonary hypertension but is suffering
from further shortness of breath and decreased exercise tolerance. A decision is made to
begin treating her with ambrisentan.
Which of the following correctly describes the mode of action of ambrisentan?

A Calcium channel antagonist

B Endothelin A receptor antagonist

C Endothelin B receptor agonist

D I f channel inhibitor

E Phosphodiesterase type 4 (PDE-4) inhibitor

Explanation 

B Endothelin A receptor antagonist

Ambrisentan is about 4000 times more selective for the endothelin A receptor vs the
endothelin B receptor. The endothelin A receptor mediates vasoconstriction and smooth
muscle cell proliferation, particularly in the pulmonary arterial tree; hence, selective
antagonism of the endothelin A receptor is highly desirable. The endothelin B receptor
mediates production of nitric oxide and prostacyclin, which are vasodilators.

A Calcium channel antagonist

Calcium channel antagonists, such as nifedipine, are used as initial therapy in the treatment of
primary pulmonary hypertension, although < 50% of patients respond to calcium channel
antagonists.

C Endothelin B receptor agonist


Although ambrisentan is a selective antagonist for the endothelin A receptor, it does not
work as an agonist at the B receptor. Bosentan is a non-selective endothelin A and B
receptor antagonist.

D I f channel inhibitor

Ivabradine is an I f channel inhibitor; it is used in the treatment of angina and heart failure
where it has been proven to reduce ventricular rate and impact positively on cardiovascular
outcomes.

E Phosphodiesterase type 4 (PDE-4) inhibitor

Roflumilast is a PDE-4 inhibitor used in the treatment of chronic obstructive pulmonary


disease where patients are symptomatic despite treatment with maximal inhaled therapy.
Sildenafil is an example of a PDE-5 inhibitor used in the treatment of primary pulmonary
hypertension and erectile dysfunction.
51693

Rate this question:

Next Question

Previous Question Tag Question

Feedback End Review

Difficulty: Average

Peer Responses %

Q. Answered Flagged

Q1

Q2
Q. Answered Flagged

Q3

Q4

Q5

Q6

Q7

Q8 
0:00:17/03:00:00

A 23-year-old man is referred to the Endocrinology Clinic for review. He has recurrent
balanitis, for which he has been buying over-the-counter fluconazole, and has felt generally
tired over the past few months. He has a strong family history of type 2 diabetes.
On examination, his blood pressure (BP) is 110/70 mmHg, and pulse 70 bpm and regular. He
does not look dehydrated. His body mass index is 22.5 kg/m 2.
Investigations:

Haemoglobin (Hb) 13.2 g/dl (13.5–17.5 g/dl)

White cell count (WCC) 6.0 × 10 9/l (4–11 × 10 9/l)

Platelets (PLT) 209 × 10 9/l (150–400 × 10 9/l)

Sodium (Na +) 144 mmol/l (135–145 mmol/l)

Potassium (K +) 4.0 mmol/l (3.5–5.0 mmol/l)

Creatinine (Cr) 90 µmol/l (50–120 µmol/l)

Fasting blood glucose × 2 7.8 mmol/l, 7.3 mmol/l (< 7 mmol/l)

Anti-glutamic acid decarboxylase (GAD),


anti-tyrosine phosphatase islet antigen 2 Negative
(IA2) antibodies

A Impaired fasting glucose

B Latent autoimmune diabetes (LADA)

C Maturity-onset diabetes of the young (MODY)

D Type 1 diabetes

E Type 2 diabetes

Explanation 

C Maturity-onset diabetes of the young (MODY)


The clues here to a diagnosis of MODY include a strong family history of type 2 diabetes and
presentation with chronic symptoms of diabetes in a man of normal weight where insulin
autoantibodies are negative. MODY2 and 3 are the most common forms. MODY2 is caused by
a mutation in the glucokinase gene and leads to mild life-long hyperglycaemia, and MODY3 is
caused by a mutation in the hepatocyte nuclear factor 1 (HNF-1) alpha gene which leads to
diabetes that is very responsive to sulfonylurea.

A Impaired fasting glucose

Impaired fasting glucose is associated with fasting glucose levels of between 6.1 and 7.0
mmol/l. It is an indicator of both increased risk of cardiovascular disease and increased risk of
progression to type 2 diabetes.

B Latent autoimmune diabetes (LADA)

LADA is, in essence, ‘slow-burn’ type 1 diabetes where patients have positive diabetes
autoantibodies, such as anti-GAD, but have gradually progressive hyperglycaemia vs rapidly
progressive hyperglycaemia and ketosis seen in classical type 1 diabetes.

D Type 1 diabetes

Type 1 diabetes is associated with rapidly progressive hyperglycaemia, weight loss due to
metabolism of fat and muscle and worsening ketoacidosis. It does not fit with the mild
symptoms of hyperglycaemia seen here.

E Type 2 diabetes

Classical type 2 diabetes presents with gradually progressive hyperglycaemia in patients with
chronic weight gain and other features of insulin resistance such as hypertriglyceridaemia,
low high-density lipoprotein (HDL) cholesterol and hypertension.
51717

Rate this question:

Next Question

Previous Question Tag Question

Feedback End Review

Difficulty: Average

Peer Responses %
Q. Answered Flagged

Q1

Q2

Q3

Q4

Q5

Q6

Q7

Q8

Q9

0:00:17/03:00:00

A 34-year-old man presents to the Sexually Transmitted Diseases Clinic following a business
trip to Latin America. He has no current regular sexual partner and admits to unprotected
sexual intercourse during the trip.
He has developed two ulcers over the glans penis, around 1 cm in diameter; they have an
erythematous base and are covered with yellow purulent exudate. You can feel marked
inguinal lymphadenopathy on physical examination.
Which of the following is the most likely diagnosis?

A Chancroid

B Gonorrhoea

C Granuloma inguinale

D Herpes simplex infection

E Syphilis

Explanation 

A Chancroid

Chancroid is rarely reported in the United States or Western Europe, but infection is thought
to be relatively common in areas of Latin America and sub-Saharan Africa. It is characterised
by the development of one to two large ulcers on areas of the skin where abrasions occur
during sexual intercourse, and a few days later by enlarged fluctuant inguinal lymph nodes
due to the formation of pus. It is caused by Haemophilus ducreyi and can be treated with
single-dose azithromycin or intramuscular ceftriaxone therapy.

B Gonorrhoea

Gonococcal infection is associated with symptoms of urethritis, including dysuria and urethral
discharge. It is not usually associated with the formation of genital ulcers or marked inguinal
lymphadenopathy, as seen here. Gonorrhoea is usually treated with a combination of
azithromycin and ceftriaxone.
C Granuloma inguinale

Granuloma inguinale presents with a single painless, progressive ulcerative lesion, which is
highly vascular. Klebsiella granulomatis, the cause of granuloma inguinale, is sensitive to both
macrolides and quinolones as possible treatment options. The major difference from
chancroid is the lesions in granuloma inguinale tend to be vascular and invasive, whereas in
chancroid, there is early spread to lymph nodes which become large and fluctuant.

D Herpes simplex infection

Herpes simplex is characterised by crops of small vesicles which are intensely itchy; they are
de-roofed to form small, shallow ulcers. The marked inguinal lymphadenopathy would also be
very unusual for herpes infection.

E Syphilis

Chancroid can be differentiated from syphilis on the basis that syphilis presents with a
painless, indurated, clean-based ulcer, rather than the painful ulcers with an erythematous,
sloughy base described here.
51736

Rate this question:

Next Question

Previous Question Tag Question

Feedback End Review

Difficulty: Average

Peer Responses %

Q. Answered Flagged
Q. Answered Flagged

Q1

Q2

Q3

Q4

Q5

Q6

Q7

Q8

0:00:17/03:00:00

A 30-year-old woman who is a manager in the local chemical plant is brought to the
Emergency Department by her colleagues. She believes that she can communicate with
everyone in the company just by transmitting her thoughts and can speak up to 50 languages
fluently. According to her colleagues, she began removing items of clothing in the office and
making sexually suggestive remarks.
You examine her with a chaperone in the department, and she displays pressure of speech
and flight of ideas and is very easily distracted. Physical examination is unremarkable.
She is happy to accept medication.

Which of the following is the most appropriate initial intervention for her?

A Chlordiazepoxide

B Chlorpromazine

C Lithium

D Risperidone

E Sodium valproate

Explanation 

D Risperidone

This patient presents with a first episode of mania, with grandiose ideas, pressure of speech
and sexual disinhibition. She does not appear aggressive and is happy to accept medication.
An atypical antipsychotic, such as olanzapine, quetiapine and risperidone, is an appropriate
initial intervention and is supported by the National Institute for Health and Care Excellence
(NICE) guidelines. Where patients are taking an antidepressant and they present with a
manic episode, this is discontinued.

A Chlordiazepoxide

Chlordiazepoxide is a benzodiazepine which is used in the management of alcohol


withdrawal. Agents such as lorazepam, rather than chlordiazepoxide, may be used when
patients display initial agitation or aggression and refuse intervention.
B Chlorpromazine

Chlorpromazine is a conventional dopamine antagonist antipsychotic agent. It is not used as


first-line therapy for the management of mania because of its adverse event profile, which
includes cardiovascular changes, such as QT prolongation, and acute dystonia.

C Lithium

Lithium is the mainstay of prophylaxis for patients with bipolar disorder, although it can take
up to six months to reach maximum therapeutic effect. For this reason, it is not a first-line
agent here.

E Sodium valproate

Sodium valproate can be used to prevent cycling from mania to depression in bipolar
disorder, although it is not recommended for women of childbearing potential because of
teratogenicity risks. It can be used in isolation or as an adjunct to other prophylaxis.
51758

Rate this question:

Next Question

Previous Question Tag Question

Feedback End Review

Difficulty: Average

Peer Responses %

Q. Answered Flagged

Q1
Q. Answered Flagged

Q2

Q3

Q4

Q5

Q6

Q7

Q8 

 External Links

NICE. Psychosis and schizophrenia in adults: Prevention and management. Clinical guideline 178 (CG178). …
nice.org.uk/guidance/cg178
(https://www.nice.org.uk/guidance/cg178)
0:00:17/03:00:00

A 43-year-old builder comes to the Rheumatology Clinic for review. He has extensive
psoriasis and has developed a deforming distal interphalangeal joint polyarthropathy over the
course of the past year. He is now unable to continue his job due to pain in his hands.
Currently, therapy includes weekly methotrexate and as required (prn) naproxen for pain
relief. He has already failed a trial of sulfasalazine to control his symptoms.
Examination confirms evidence of extensive synovitis affecting his fingers, wrists, toes and
both elbows. You also note psoriasis affecting his scalp and both elbows and knees.
Investigations:

Haemoglobin (Hb) 13.2 g/dl (13.5–17.5 g/dl)

White cell count (WCC) 7.1 × 10 9/l (4–11 × 10 9/l)

Platelets (PLT) 201 × 10 9/l (150–400 × 10 9/l)

Sodium (Na +) 144 mmol/l (135–145 mmol/l)

Potassium (K +) 4.3 mmol/l (3.5–5.0 mmol/l)

Creatinine (Cr) 95 µmol/l (50–120 µmol/l)

C-reactive protein (CRP) 68 mg/l (0–10 mg/l)

Which of the following is the most appropriate next step?

A Add golimumab

B Add low-dose prednisolone

C Add tocilizumab

D Add ustekinumab

E Switch methotrexate to hydroxychloroquine

Explanation 

A Add golimumab
In patients who have failed two disease-modifying antirheumatic drugs (DMARDs), like this
patient with psoriatic arthritis who has symptoms despite therapy with sulfasalazine and then
methotrexate, a tumour necrosis factor (TNF)-alpha antagonist is the next most appropriate
step. Guidelines recommend assessment of response to therapy at 12 weeks, and a trial of an
alternative agent in the event an adequate response has not been seen to the first one.
Response can be assessed by changes in inflammatory markers and scoring systems such as
the Disease Activity in PSoriatic Arthritis (DAPSA) score which take into account measures of
joint pain and stiffness.

B Add low-dose prednisolone

Given other disease-modifying agents, including TNF-alpha antagonists, are available for this
patient, the benefit–risk profile of adding low-dose prednisolone to this patient’s regimen
would not be considered positive, given the long-term impact on bone mineral density,
weight gain and glucose tolerance.

C Add tocilizumab

Toclizumab is an interleukin 6 (IL6) antagonist which is used in the treatment of patients with
symptoms resistant to conventional treatments, including methotrexate, TNF antagonists and
other agents such as anti-CD20 monoclonals. There is no extensive evidence to support its
use in psoriatic arthritis.

D Add ustekinumab

Ustekinumab is a combined inhibitor of IL12 and IL23, and it is recommended in psoriatic


arthritis for patients where symptoms have not been controlled on a TNF-alpha antagonist
therapy.

E Switch methotrexate to hydroxychloroquine

Switching of methotrexate to hydroxychloroquine is not appropriate here, because


methotrexate can cause exfoliative dermatitis, which may only add to the skin symptoms
from underlying psoriasis.
51773

Rate this question:

Next Question

Previous Question Tag Question

Feedback End Review


Difficulty: Average

Peer Responses %

Q. Answered Flagged

Q1

Q2

Q3

Q4

Q5

Q6

Q7

Q8

Q9

0:00:17/03:00:00

A 59-year-old woman who takes a number of medications, including prophylaxis for


recurrent urinary tract infection, ischaemic heart disease and diabetes mellitus, comes to the
Renal Clinic for review. She has developed increasing shortness of breath over the past few
months and a dry, nagging cough.
On examination, her blood pressure is 149/88 mmHg and her pulse is 67 bpm and regular.
There are scattered crackles on auscultation of the chest, but no evidence of wheeze.
Abdomen is soft and non-tender; her body mass index (BMI) is 27 kg/m 2.
Investigations:

Haemoglobin (Hb) 11.9 g/dl (11.5–15.5 g/dl)

White cell count (WCC) 7.9 × 10 9/l (4–11 × 10 9/l)

Platelets (PLT) 281 × 10 9/l (150–400 × 10 9/l)

Sodium (Na +) 143 mmol/l (135–145 mmol/l)

Potassium (K +) 4.5 mmol/l (3.5–5.0 mmol/l)

Creatinine (Cr) 109 µmol/l (50–120 µmol/l)

Chest X-ray Patchy interstitial shadowing

Forced vital capacity (FVC) 60%


of predicted value
Lung function testing
Forced expiratory volume
in 1 second (FEV1)/FVC 70%

Which of the following medications is most likely to have caused her shortness of breath?

A Bisoprolol

B Metformin

C Nitrofurantoin

D Ofloxacin

E Ramipril
Explanation 

C Nitrofurantoin

Long-term use of nitrofurantoin is recognised as a cause of interstitial pneumonitis and


pulmonary fibrosis, which may develop insidiously. For this reason, monitoring of lung
function is recommended in patients like this who are taking it for a prolonged period.
Nitrofurantoin is also recognised to cause hepatitis, and for this reason, regular monitoring of
liver function is also recommended. It may also lead to folate deficiency, which can be more
marked in patients with concomitant diabetes mellitus.

A Bisoprolol

Bisoprolol is associated with obstructive lung disease in some patients with asthma. This
patient does not suffer from asthma, and the clinical picture here with a restrictive lung
defect and interstitial shadowing on the X-ray fits better with a fibrotic process.

B Metformin

The main concern with respect to metformin use is bile acid malabsorption, which can lead to
chronic diarrhoea. This can often be managed by introducing dose increments of metformin
more slowly in susceptible patients.

D Ofloxacin

Ofloxacin is a quinolone. Quinolones do not cause pulmonary fibrosis, although advice has
been given to restrict their use, where possible, because of an increased risk of tendon
rupture in patients who take them.

E Ramipril

Angiotensin-converting enzyme (ACE) inhibitors are recognised to cause a dry cough, but
this is related to decreased metabolism of bradykinin, rather than due to pulmonary fibrosis.
The cough usually resolves by switching patients to an angiotensin receptor blocker.
51755

Rate this question:

Next Question

Previous Question Tag Question

Feedback End Review


Difficulty: Average

Peer Responses %

Q. Answered Flagged

Q1

Q2

Q3

Q4

Q5

Q6

Q7

Q8

Q9

0:00:17/03:00:00

A 19-year-old man who has type 1 diabetes comes to the Dermatology Clinic for review. He
complains of a number of thin-roofed, flaccid, blister-like lesions which appear over his lower
abdomen around his waist band. Once the lesions rupture, they leave an area of moist
erythema which takes a few days to heal and looks like an area of scalded skin. When they
burst, they lose straw-coloured fluid which eventually turn cloudy and dark yellow.
On examination, you note 2–3 small areas (< 3 cm in diameter) of erythematous skin, and one
flaccid bulla which is approximately 2 cm across.
Which of the following is the most likely cause of the lesions?

A Bullous pemphigoid

B Dermatitis herpetiformis

C Klebsiella pneumoniae (KA)

D Staphylococcus aureus (SA)

E Streptococcus pyogenes (SP)

Explanation 

D Staphylococcus aureus (SA)

Bullous impetigo, fitting with the history of flaccid bullae which leak straw-coloured fluid and
heal with an erythematous base, is caused almost invariably by SA. The area of infection may
be difficult to keep clean, and this patient’s type 1 diabetes may increase the risk of acquiring
the lesions. Mupirocin and fusidic acid are both effective topical therapies.

A Bullous pemphigoid

Bullous pemphigoid results in the formation of multiple tense blisters, which are usually very
itchy. Episodes of the rash may wax and wane and are treated with topical corticosteroids in
the first instance. Lesions are found most often on the upper arms and inner thighs and are
rarely seen in the oral cavity.

B Dermatitis herpetiformis
Dermatitis is associated with an intensely itchy vesicular rash, which most commonly affects
the thighs and buttocks. It is seen in association with coeliac disease and improves
significantly on avoidance of gluten. Topical dapsone may be used in patients where the rash
fails to resolve on gluten avoidance alone.

C Klebsiella pneumoniae (KA)

KA is a cause of cavitating pneumonia, which is seen more commonly in patients with a


history of alcoholism and malnutrition and in those who are immunosuppressed for other
reasons.

E Streptococcus pyogenes (SP)

SP is a common cause of non-bullous impetigo, alongside SA, although it is almost never the
underlying cause of the bullous form of the disease. Treatment for SP-induced impetigo is
similar to that for SA-related bullous impetigo with topical antibiotics.
51713

Rate this question:

Next Question

Previous Question Tag Question

Feedback End Review

Difficulty: Average

Peer Responses %

Q. Answered Flagged

Q1
Q. Answered Flagged

Q2

Q3

Q4

Q5

Q6

Q7

Q8
0:00:17/03:00:00

A 49-year-old man presents to the Gastroenterology Clinic for review. He has been referred
by his general practitioner (GP) with increased skin pigmentation, diabetes mellitus
controlled with metformin monotherapy and osteoarthritis of both knees. He also admits to
erectile dysfunction, for which he takes over-the-counter sildenafil. He drinks four pints of
beer each weekend.
Examination reveals a blood pressure of 138/88 mmHg, and his pulse is 80 bpm and regular.
There are no heart murmurs. His jugular venous pressure (JVP) is elevated, and he has pitting
oedema of both ankles. His chest is clear. Abdomen is soft and non-tender. You note a
number of spider naevi over the upper body.

Investigations:

Haemoglobin (Hb) 14.9 g/dl (13.5–17.5 g/dl)

White cell count (WCC) 6.7 × 10 9/l (4–11 × 10 9/l)

Platelets (PLT) 302 × 10 9/l (150–400 × 10 9/l)

Sodium (Na +) 140 mmol/l (135–145 mmol/l)

Potassium (K +) 4.5 mmol/l (3.5–5.0 mmol/l)

Creatinine (Cr) 95 µmol/l (50–120 µmol/l)

Alanine aminotransferase (ALT) 122 iu/l (5–20 iu/l)

Alkaline phosphatase 189 iu/l (30–130 iu/l)

Bilirubin 10 µmol/l (2–17 µmol/l)

Albumin 32 g/l (35–55 g/l)

Glucose (fasting) 7.3 mmol/l (< 7 mmol/l)

Which of the following is the most likely cause of this patient’s right heart failure?

A Alcoholic cardiomyopathy

B Decreased hepatic albumin synthesis

C Diabetes mellitus

D Hypertrophic obstructive cardiomyopathy


E Iron overload

Explanation 

E Iron overload

This patient has a number of the cardinal features of hereditary haemochromatosis, including
increased skin pigmentation, diabetes mellitus, evidence of hepatic impairment, sexual
dysfunction and probable chondrocalcinosis. Iron overload may present as here with right
heart failure. Venesection is the intervention of choice. Cardiac iron content can be measured
using magnetic resonance imaging (MRI), which may assist in confirming the diagnosis where
it is in doubt. Other causes of iron overload include repeated blood transfusions in patients
with thalassaemia major.

A Alcoholic cardiomyopathy

Alcoholic cardiomyopathy leads to the development of biventricular failure and atrial


fibrillation. In addition, this patient’s consumption of alcohol is extremely modest, making a
diagnosis of alcoholic cardiomyopathy very unlikely.

B Decreased hepatic albumin synthesis

The albumin level here is just below the lower limit of the normal range. A level of 32 g/l is
not one at which significant oedema would be expected, were hypoalbuminaemia to be the
primary cause.

C Diabetes mellitus

This patient has very modestly elevated blood glucose level of 7.3 mmol/l on metformin
monotherapy. Although he may have developed diastolic heart failure, it is more likely this
has occurred as a result of iron overload, rather than diabetes mellitus.

D Hypertrophic obstructive cardiomyopathy

Hypertrophic obstructive cardiomyopathy presents with symptoms of left ventricular outflow


obstruction, including angina, and episodes of arrhythmia which may precipitate collapse,
rather than the right ventricular failure seen here.
51689

Rate this question:

Next Question
Previous Question Tag Question

Feedback End Review

Difficulty: Average

Peer Responses %

Q. Answered Flagged

Q1

Q2

Q3

Q4

Q5

Q6

Q7

Q8

Q9

0:00:17/03:00:00

A 34-year-old woman comes to the Cardiology Clinic for review. She has suffered increasing
shortness of breath and decreased exercise tolerance over the past six months and is now
unable to climb the stairs without needing to stop. She gave birth to her first child some nine
months earlier and before the pregnancy would have considered herself fit and healthy.
Her blood pressure is 139/85 mmHg, and her pulse is 88 bpm and regular. She has a pan-
systolic murmur, loudest in the fourth intercostal space in the left parasternal region. You can
feel a pulsatile liver edge, and she has pitting oedema of both ankles. Routine blood tests are
unremarkable. Her body mass index (BMI) is 23 kg/m 2.

Which of the following is the most likely cause of her murmur?

A Aortic regurgitation

B Aortic stenosis

C Mitral regurgitation

D Pulmonary stenosis

E Tricuspid regurgitation

Explanation 

E Tricuspid regurgitation

The suspicion here is of primary pulmonary hypertension (PPH) which has led to tricuspid
regurgitation, characterised by a systolic murmur, loudest in the fourth intercostal space to
the left of the sternum. It rarely occurs due to primary valve disease and is much more likely
to be seen because of pulmonary hypertension. In this case, confirmation of the diagnosis of
PPH is essential, with endothelin receptor antagonists and phosphodiesterase type 5 (PDE-5)
inhibitors both potential initial interventions to reduce pulmonary artery pressure.

Pregnancy in patients with PPH carries significant risks, including worsening of pulmonary
artery pressure, right heart failure and development of permanent atrial fibrillation. It also
significantly increases the risks of thromboembolism. Patients diagnosed with PPH should
therefore avoid pregnancy.

A Aortic regurgitation
Aortic regurgitation is associated with an early diastolic murmur, loudest in the third left
intercostal space. Severe aortic regurgitation leads to left ventricular failure.

B Aortic stenosis

Aortic stenosis is associated with an ejection systolic murmur, loudest in the second right
intercostal space. It is associated with syncopal episodes, angina and the gradual
development of left ventricular failure.

C Mitral regurgitation

Mitral regurgitation is associated with a pan-systolic murmur, best heard at the apex, and
with the development of left ventricular failure, rather than the signs of right heart failure
seen here.

D Pulmonary stenosis

Pulmonary stenosis sounds very similar to aortic stenosis and is best heard in the second left
intercostal space. It is associated with the development of right heart failure, and the majority
of cases of pulmonary stenosis are congenital.
51679

Rate this question:

Next Question

Previous Question Tag Question

Feedback End Review

Difficulty: Average

Peer Responses %
Q. Answered Flagged

Q1

Q2

Q3

Q4

Q5

Q6

Q7

Q8

Q9 
0:00:17/03:00:00

A 29-year-old man is treated with intravenous (IV) flecainide 1 mg/kg after presenting to the
Emergency Department with paroxysmal atrial fibrillation (AF) after drinking to excess at a
wedding. He cardioverts to sinus rhythm soon after the IV loading dose.
Which of the following correctly describes the mode of action of flecainide?

A Calcium channel inhibition

B Fast sodium channel inhibition

C Muscarinic receptor antagonism

D Na +/K + ATPase inhibition

E Potassium channel inhibition

Explanation 

B Fast sodium channel inhibition

Flecainide is a class IC anti-arrhythmic which is based on three properties:


1. Marked inhibition of the fast sodium channel in the heart
2. Slow onset and offset kinetics of inhibition of the sodium channel (reflecting slow
attachment to, and dissociation from, sodium channels)
3. The differential effect of the drug on the action potential duration in ventricular muscle
vs Purkinje fibres, having no effect in the former and markedly shortening it in the latter.

A Calcium channel inhibition

Verapamil and diltiazem are examples of calcium channel antagonists with cardiac selectivity.
They bind to L-type calcium channels which regulate the influx of calcium into muscle cells.
This governs both the heart rate and the force of cardiac contraction.

C Muscarinic receptor antagonism


Atropine is a muscarinic receptor antagonist used as a treatment for symptomatic
bradycardia. It may be given as repeated doses of up to 3 mg where maximum effect should
be reached.

D Na +/K + ATPase inhibition

Cardiac glycosides, such as digoxin, are Na +/K + ATPase inhibitors. The pump is electrogenic
because it transports three sodium ions out of the cell for every two potassium ions which
enter it, thus increasing the membrane potential. Digoxin increases myocardial contractility
and has other effects in increasing the vagal tone, thus reducing the heart rate.

E Potassium channel inhibition

Class III anti-arrhythmics, such as amiodarone, block cardiac potassium channels. They are
used in the treatment of ventricular tachycardia. Sotalol, which also has class II anti-
arrhythmic properties (beta-blockade), is used in the treatment of atrial flutter and AF.
51696

Rate this question:

Next Question

Previous Question Tag Question

Feedback End Review

Difficulty: Average

Peer Responses %

Q. Answered Flagged

Q1

Q2
Q. Answered Flagged

Q3

Q4

Q5

Q6

Q7

Q8 
0:00:17/03:00:00

A 43-year-old electrician comes to the clinic for review. He has extensive psoriasis affecting
both elbows, knees, extensor surfaces of his hands, and his scalp. He has been utilising
combination topical therapy with corticosteroids and vitamin D analogue. He now feels that
his fingers are becoming very stiff and swollen and he is unable to work.
On examination of the hands, you note nail pitting and obvious distal interphalangeal joint
synovitis.
Investigations:

Haemoglobin (Hb) 13.0 g/dl (13.5–17.5 g/dl)

White cell count (WCC) 8.1 × 10 9/l (4–11 × 10 9/l)

Platelet (PLT) 203 × 10 9/l (150–400 × 10 9/l)

Sodium (Na +) 142 mmol/l (135–145 mmol/l)

Potassium (K +) 4.5 mmol/l (3.5–5.0 mmol/l)

Creatinine (Cr) 102 µmol/l (50–120 µmol/l)

C-reactive protein (CRP) 62 mg/l (0–10 mg/l)

Which of the following is the most appropriate next intervention?

A Acitretin

B Etanercept

C Leflunomide

D Methotrexate

E Rituximab

Explanation 

D Methotrexate
In this situation, with significant psoriatic arthritis which threatens this patient’s livelihood,
extensive psoriatic plaques and elevated CRP, systemic intervention is warranted. Monitoring
of full blood count and liver and renal function should be carried out weekly until the
treatment dose is stabilised, and then every two to three months thereafter. Both hepatic and
pulmonary fibrosis are possible adverse effects over the longer term. Folic acid
supplementation is essential in conjunction with methotrexate therapy.

A Acitretin

Acitretin is indicated for extensive plaque psoriasis, particularly where there is pustular
psoriasis of the hands and feet. It does not have the positive effects on joint disease that are
seen with methotrexate, meaning that it is not an appropriate option here.

B Etanercept

Anti-tumour necrosis factor (TNF) therapies are indicated where there has been an
inadequate response to traditional systemic interventions such as methotrexate and
ciclosporin. They are effective in managing joint disease and may be an option in patients like
this in the event that the response to methotrexate is inadequate. Patients should be
screened for tuberculosis and viral hepatitis before starting an anti-TNF intervention.

C Leflunomide

Leflunomide is a second- or third-line systemic therapy in patients who have failed to


respond to other interventions such as methotrexate or ciclosporin. It inhibits dihydroorotate
dehydrogenase which reduces white blood cell proliferation.

E Rituximab

Rituximab is an anti-CD20 monoclonal antibody which depletes B cell antibody production.


Rituximab is not usually employed as a psoriasis treatment because it is believed the disease
is primarily driven by T cell and macrophage activity.
51711

Rate this question:

Next Question

Previous Question Tag Question

Feedback End Review

Difficulty: Average
Peer Responses %

Q. Answered Flagged

Q1

Q2

Q3

Q4

Q5

Q6

Q7

Q8

Q9

0:00:17/03:00:00

An 18-year-old student comes to the Emergency Department complaining of jaundice. He has


felt unwell and off his food, with a heavy cold for the past 48 hours, and awoke that morning
with jaundiced sclerae. He is usually well, although he reports an episode of jaundice some
three years ago.
On examination, his blood pressure is 122/82 mmHg, and pulse 74 bpm and regular. His
temperature is 37.3 °C. Abdomen is soft and non-tender, with no palpable masses.
Investigations:

Haemoglobin (Hb) 13.1 g/dl (13.5–17.5 g/dl)

White cell count (WCC) 6.7 × 10 9/l (4–11 × 10 9/l)

Platelets (PLT) 203 × 10 9/l (150–400 × 10 9/l)

Sodium (Na +) 144 mmol/l (135–145 mmol/l)

Potassium (K +) 4.5 mmol/l (3.5–5.0 mmol/l)

Creatinine (Cr) 93 µmol/l (50–120 µmol/l)

Alanine aminotransferase (ALT) 19 iu/l (5–30 iu/l)

Bilirubin 45 µmol/l (2–17 µmol/l)

Alkaline phosphatase 90 iu/l (30–130 iu/l)

Albumin 40 g/l (35–55 g/l)

Which of the following is the most likely diagnosis?

A Epstein–Barr virus (EBV) infection

B Gilbert syndrome

C Hepatitis A

D Hepatitis C

E Hereditary spherocytosis

Explanation 
B Gilbert syndrome

This patient has isolated hyperbilirubinaemia (a second episode) against a background of an


intercurrent illness (a cold). Patients may present with Gilbert syndrome if they become
unwell or have a period of fasting, lack of sleep or extreme physical exertion or if they are
taking antiretroviral drugs. It is an autosomal recessive disorder affecting the UGT1A1 gene,
which leads to a 60–70% reduction in the body’s ability to conjugate bilirubin. It appears the
mutation may be protective against cancer and atherosclerosis, accounting for a disease
prevalence of 5–10% in the Western European population. No treatment is required, and the
jaundice will resolve over a few days.

A Epstein–Barr virus (EBV) infection

EBV is associated with hepatocellular dysfunction and a rise in transaminases, rather than the
normal ALT level seen here. In addition, symptoms of acute infection, such as pharyngitis,
would be expected.

C Hepatitis A

Hepatitis A is associated with flu-like symptoms as a prodrome to the onset of jaundice. It is


also associated with marked hepatocellular dysfunction and a rise in transaminases, which is
not reported here.

D Hepatitis C

Chronic hepatitis C infection presents with a picture of slowly progressive hepatic fibrosis.
The picture seen here with an isolated rise in bilirubin levels is inconsistent with the diagnosis.
There is no reported history of injecting drug abuse to suggest exposure to hepatitis C either.

E Hereditary spherocytosis

Hereditary spherocytosis may be associated with hyperbilirubinaemia and, because of this, an


increased risk of gallstones. However, the increase in bilirubin levels occurs against a
background of chronic haemolysis, and spherocytes are usually recognised on blood film.
51726

Rate this question:

Next Question

Previous Question Tag Question

Feedback End Review


Difficulty: Average

Peer Responses %

Q. Answered Flagged

Q1

Q2

Q3

Q4

Q5

Q6

Q7

Q8

Q9

0:00:17/03:00:00

A 21-year-old student presents to the Emergency Department a few days after returning from
a holiday to Spain. He has pain and a urethral discharge when he tries to pass urine, and
multiple joint pains, particularly affecting his fingers and toes, wrists, ankles, elbows and
knees. You can express some of the urethral discharge, and nucleic acid amplification test
(NAAT) is positive for Chlamydia.
Which of the following is the most appropriate intervention?

A Amoxicillin three times daily (tds) for seven days

B Doxycycline twice daily (bd) for seven days

C Erythromycin bd for 14 days

D Ofloxacin bd for seven days

E Trimethoprim bd for seven days

Explanation 

B Doxycycline twice daily (bd) for seven days

Oral doxycycline at a usual dose of 100 mg bd for seven days or a single 1 g dose of
azithromycin is the intervention of choice for Chlamydia infection. If possible, the student
should inform any sexual contacts and advise them to undergo testing and treatment. Most
Genitourinary Medicine (GUM) Clinics offer assistance with this through confidentially
informing previous partners. He should also be tested for other sexually transmitted
infections. His reactive arthritis can be managed with a regular non-steroidal anti-
inflammatory drug (NSAID) and will improve with treatment of the underlying infection.

A Amoxicillin three times daily (tds) for seven days

Although Chlamydia is sensitive to amoxicillin, treatment can induce latency and later
reactivation of the bacterium, meaning it is usually reserved as an option for women who are
pregnant where other interventions for Chlamydia are unsuitable or are contraindicated.

C Erythromycin bd for 14 days


This is another option for treatment of Chlamydia in pregnancy where it can also be used
four times daily as a shorter course over seven days. Often compliance is an issue due to
nausea and diarrhoea associated with its use.

D Ofloxacin bd for seven days

Although quinolones can be used for the treatment of Chlamydia, a recent Committee on
Safety of Medicines (CSM) warning has encouraged physicians to limit their use because of a
risk of tendon rupture associated with the drug class.

E Trimethoprim bd for seven days

Trimethoprim is the standard intervention for a urinary tract infection. The presence of
Chlamydia identified on NAAT prompts us to reach for doxycycline in the first instance.
51734

Rate this question:

Next Question

Previous Question Tag Question

Feedback End Review

Difficulty: Average

Peer Responses %

Q. Answered Flagged

Q1

Q2

Q3
Q. Answered Flagged

Q4

Q5

Q6

Q7

Q8 
0:00:17/03:00:00

A 45-year-old man presents to the Emergency Department, following a pulmonary


haemorrhage. He is currently being investigated by his general practitioner (GP) for weight
loss, intermittent night sweats and an increase in his creatinine level. He takes amlodipine for
hypertension and a nasal spray for sinusitis, which was recently diagnosed.
Examination reveals a blood pressure of 155/95 mmHg and a pulse of 90 bpm and regular.
There are scattered crackles on auscultation of the chest. Abdomen is soft and non-tender.
Investigations:

Haemoglobin (Hb) 10.5 g/dl (13.5–17.5 g/dl)

White cell count (WCC) 10.2 × 10 9/l (4–11 × 10 9/l)

Sodium (Na +) 144 mmol/l (135–145 mmol/l)

Potassium (K +) 5.2 mmol/l (3.5–5.0 mmol/l)

239 µmol/l (up from


Creatinine (Cr) 189 µmol/l checked by (50–120 µmol/l)
GP seven days earlier)

Cytoplasmic pattern anti-neutrophil


Positive
cytoplasmic antibodies (cANCA)

Which of the following is the most likely finding on renal biopsy?

A Crescenteric glomerulonephritis with linear immunoglobulin G (IgG) deposition

B Focal segmental glomerulosclerosis (FSGS)

C Membranous glomerulonephritis

D Minimal change glomerulonephritis

E Pauci-immune crescenteric glomerulonephritis

Explanation 

E Pauci-immune crescenteric glomerulonephritis


In most patients with granulomatosis with polyangiitis (GPA), the most likely diagnosis here,
the glomerulonephritis has few or no immune deposits in the glomeruli on
immunofluorescence and electron microscopy. Rarer variants include presentation with
interstitial nephritis. Immune complex deposition is seen in some patients, but this is much
rarer than pauci-immune disease.

A Crescenteric glomerulonephritis with linear immunoglobulin G (IgG) deposition

This is the hallmark of patients suffering from anti-glomerular basement membrane (GBM)
disease where pulmonary haemorrhage is seen, although patients are usually ANCA-negative.

B Focal segmental glomerulosclerosis (FSGS)

FSGS is more commonly seen in patients with diabetes, those taking antiretroviral
medication, sickle-cell disease and systemic lupus erythematosus. It is not usually a feature of
GPA.

C Membranous glomerulonephritis

Among inflammatory disorders, membranous disease is seen in patients with systemic lupus
erythematosus and polyarteritis nodosa. It is not the usual picture seen in patients with
aggressive GPA.

D Minimal change glomerulonephritis

Minimal change, by definition, shows little evidence of disease on light microscopy; it


definitely does not fit with the expected findings in GPA. Minimal change disease is
associated with marked proteinuria and is seen most commonly in children and young adults.
It responds well to oral corticosteroids.
51751

Rate this question:

Next Question

Previous Question Tag Question

Feedback End Review

Difficulty: Average

Peer Responses %
Q. Answered Flagged

Q1

Q2

Q3

Q4

Q5

Q6

Q7

Q8

Q9

0:00:17/03:00:00

A 19-year-old woman comes to the Respiratory Clinic for review. She has recently begun
work in a laboratory animal facility, and she is concerned because during the course of a
working week, she develops gradually worsening shortness of breath and wheeze, which
seems to improve during her days off. She is a non-smoker and has no previous history of
asthma. She does, however, suffer from hay fever during the summer months.
Examination in the clinic after two days off is entirely normal. Her peak expiratory flow rate
(PEFR) is 490 litres/min, with 510 expected. Routine bloods are unremarkable.
Which of the following is the most appropriate next step?

A Patch testing

B Re-deployment to a new area of the company

C Serial peak flow measurement

D Trial of montelukast

E Trial of salbutamol

Explanation 

C Serial peak flow measurement

The clinical scenario here is entirely consistent with occupational asthma where symptoms
worsen significantly during the working week and resolve during time off. Establishing the
pattern of peak flows, which should progressively worsen in tandem with symptoms, is the
best way to establish the diagnosis. The initial intervention should then be to examine
working practices to ensure that exposure to potential animal allergens is minimised.

A Patch testing

Patch testing may be useful in confirming what this patient is allergic to, although it is no
substitute for establishing the diagnosis of asthma first and working with the business to
minimise exposure to animal allergens.

B Re-deployment to a new area of the company


This should only be considered in the event that the patient still has symptoms of
occupational asthma despite appropriate procedures to minimise exposure to allergens. Just
re-deploying this patient now risks the health of others working in the same area if
procedures are not in place to minimise allergen exposure.

D Trial of montelukast

Montelukast is indicated for exercise-induced asthma and in patients where symptoms are
not controlled on inhaled corticosteroids. In this situation, a trial of four to eight weeks of
therapy, followed by reassessment, is advised.

E Trial of salbutamol

A trial of salbutamol would only be considered in the event this patient had an unpredictable
pattern of asthma symptoms. Given the close link to occupational exposure, examination of
the working environment is the first step in her management.
51768

Rate this question:

Next Question

Previous Question Tag Question

Feedback End Review

Difficulty: Average

Peer Responses %

Q. Answered Flagged

Q1
Q. Answered Flagged

Q2

Q3

Q4

Q5

Q6

Q7

Q8
0:00:17/03:00:00

A 52-year-old man who has type 1 diabetes and takes insulin presents with a plantar ulcer
and extensive local erythema affecting the ball of his left foot. He is known to have diabetic
neuropathy and has lost sensation below the shin bilaterally. He has just returned from a
beach holiday where unfortunately, he tells you he was walking around in bare feet.
Investigations:

Haemoglobin (Hb) 12.2 g/dl (13.5–17.5 g/dl)

White cell count (WCC) 18.4 × 10 9/l (4–11 × 10 9/l)

Platelets (PLT) 287 × 10 9/l (150–400 × 10 9/l)

Sodium (Na +) 144 mmol/l (135–145 mmol/l)

Potassium (K +) 4.5 mmol/l (3.5–5.0 mmol/l)

Creatinine (Cr) 109 µmol/l (50–120 µmol/l)

C-reactive protein (CRP) 210 mg/l (0–10 mg/l)

Evidence of osteomyelitis of
Magnetic resonance imaging
the first metatarsophalangeal
(MRI) foot and ankle
(MTP) joint on the left

Which of the following is the most appropriate antibiotic intervention?

A Clindamycin

B Co-amoxiclav

C Flucloxacillin

D Meropenem

E Vancomycin

Explanation 

B Co-amoxiclav
In this situation, with osteomyelitis likely due to mixed organisms, bearing in mind the history
of diabetes mellitus, an antibiotic with wide coverage against Gram-positive and negative
bacteria and anaerobes, such as co-amoxiclav, is required. It is usually given intravenously
(IV) at a dose of 1.2 g IV three times daily (tds). This patient also requires magnetic
resonance imaging (MRI) to further evaluate the extent of any infection and should be
reviewed by the surgical team. Up to four to six weeks of antibiotic therapy are usually
considered for osteomyelitis in patients with diabetes mellitus.

A Clindamycin

Clindamycin achieves excellent tissue penetration and is a first-line option for osteomyelitis in
patients with diabetic foot infection who are allergic to penicillin. It is strongly associated
with the development of Clostridium difficile colitis, and it is important to be alert to this
possibility in patients who develop diarrhoea on therapy.

C Flucloxacillin

Flucloxacillin is a first-choice option for patients with a low probability of mixed infection (ie
those without diabetes mellitus). Here an agent such as co-amoxiclav with wider coverage is
required.

D Meropenem

Meropenem is the usual first-choice option given IV for patients with severe diabetic foot
infection who are allergic to penicillin. Here, in a patient who is not penicillin-allergic, it is
reasonable to opt for co-amoxiclav.

E Vancomycin

Vancomycin should be used when there is a high index of suspicion for meticillin-resistant
Staphylocccus aureus (MRSA) infection or it is confirmed on microbiological testing. MRSA is
less likely here, bearing in mind infection is thought to have occurred in the community
through walking barefoot.
51772

Rate this question:

Next Question

Previous Question Tag Question

Feedback End Review

Difficulty: Average
Peer Responses %

Q. Answered Flagged

Q1

Q2

Q3

Q4

Q5

Q6

Q7

Q8

Q9

0:00:17/03:00:00

You are asked to review a 72-year-old man who was admitted a few days earlier with cardiac
chest pain. He underwent percutaneous coronary intervention (PCI) and stenting but has
continued to feel unwell with nausea and lethargy and has been unable to mobilise out of
bed.
Past medical history of note before his admission included a previous inferior myocardial
infarction and chronic renal impairment.
His blood pressure is 149/84 mmHg, and his pulse is 92 bpm and regular. His temperature is
37.9 °C. His toes on both feet have a blue discoloration, and there are small, punched-out
ulcers over both lower limbs. You note a dressing over the femoral puncture wound used for
angiography access.
Investigations:

Haemoglobin (Hb) 10.9 g/dl (13.5–17.5 g/dl)

White cell count (WCC) 13.2 × 10 9/l (eosinophilia) (4–11 × 10 9/l)

Platelets (PLT) 231 × 10 9/l (150–400 × 10 9/l)

Sodium (Na +) 144 mmol/l (135–145 mmol/l)

Potassium (K +) 5.9 mmol/l (3.5–5.0 mmol/l)

Creatinine (Cr) 237 µmol/l (up to 132 on admission) (50–120 µmol/l)

C-reactive protein (CRP) 89 mg/l (0–10 mg)

Low-density lipoprotein (LDL) 4.5 mmol/l (< 3.5 mmol/l)

High-density lipoprotein (HDL) 0.7 mmol/l (> 1.0 mmol/l)

Triglycerides (TG) 2.1 mmol/l (0–1.5 mmol/l)

Which of the following interventions is most appropriate?

A Intravenous (IV) methylprednisolone

B IV piperacillin/tazobactam

C Intra-arterial alteplase

D Subcutaneous (sc) treatment-dose low-molecular weight heparin (LMWH)


E Supportive therapy only

Explanation 

E Supportive therapy only

This patient has suffered cholesterol emboli as a result of his coronary angiography. The
clinical appearance of his lower limbs, eosinophilia, raised CRP and acute renal failure are all
consistent with the diagnosis. Unfortunately, no pharmacological interventions have proved
of value in managing the condition, with supportive therapy, including dialysis if needed, the
optimal way to manage him.

A Intravenous (IV) methylprednisolone

Corticosteroids have no role in the management of cholesterol embolism. Evidence of benefit


comes only from non-randomised case reports.

B IV piperacillin/tazobactam

There is no evidence of an infective process which has led to this patient’s skin changes and
renal impairment. Piperacillin/tazobactam therefore has no role in the management of this
patient. In the event that significant peripheral ischaemia is identified, surgical debridement
may be required.

C Intra-arterial alteplase

Intra-arterial thrombolysis does not improve peripheral perfusion and increases the risks from
bleeding. Therefore, it does not have a role in the management of cholesterol embolism.

D Subcutaneous (sc) treatment-dose low-molecular weight heparin (LMWH)

Similar to corticosteroids, there is no randomised controlled trial evidence to support the use
of LMWH. Case reports are equivocal, with some reporting improvement of symptoms after
cessation of anticoagulation and some resolution with anticoagulation.
51681

Rate this question:

Next Question

Previous Question Tag Question


Feedback End Review

Difficulty: Average

Peer Responses %

Q. Answered Flagged

Q1

Q2

Q3

Q4

Q5

Q6

Q7

Q8

Q9

0:00:17/03:00:00

A 29-year-old woman presents with weakness affecting her left arm. She was involved in a
rear-shunt car accident some two weeks earlier and sustained dull neck pain afterwards.
Examination reveals weakness of the biceps on the left side. There is numbness and tingling,
extending down the outside of her hand, over her thumb.
Which of the following nerve roots is most likely to have been affected by her neck injury?

A C5

B C6

C C7

D C8

E T1

Explanation 

B C6

The C6 nerve root supplies sensation over the lateral aspect of the arm and over the side of
the hand nearest the thumb. Its main motor supply is to the biceps which leads to weakness
of elbow flexion on the affected side.

A C5

C5 nerve root damage leads to weakness of shoulder abduction and abnormal sensation over
the lateral aspect of the upper arm. It does not fit with the clinical findings described here.

C C7

The C7 nerve root supplies the triceps, which leads to weakness of elbow extension. With
regard to sensation, it is responsible for supplying sensation to the middle three fingers of
the hand.
D C8

The C8 nerve root supplies the muscles which control wrist flexion, and it is responsible for
sensory supply to the little finger.

E T1

The T1 nerve root supplies sensation to the medial (ulnar) aspect of the forearm. With regard
to motor function, it is responsible for finger abduction.
51699

Rate this question:

Next Question

Previous Question Tag Question

Feedback End Review

Difficulty: Average

Peer Responses %

Q. Answered Flagged

Q1

Q2

Q3

Q4

Q5

Q6
Q. Answered Flagged

Q7

Q8

0:00:17/03:00:00

An 18-year-old woman is reviewed in the Endocrine Clinic with primary amenorrhoea. Her
general practitioner (GP) has also been watching her because of borderline hypertension.
When you review her in the clinic, her blood pressure is 142/82 mmHg, and pulse 70 bpm and
regular. She is of short stature at only 156 cm in height, and you note a relatively short neck
and low-set ears. She appears to have normal pubic hair growth and female external
genitalia.
What do you expect to find on further workup?

A Absent sense of smell

B Bilateral undescended testes on ultrasound scan (USS)

C XO karyotype

D XXY karyotype

E XY karyotype

Explanation 

C XO karyotype

This patient’s short stature with borderline hypertension, a short neck, low-set ears, and
primary amenorrhoea raises the possibility of Turner syndrome, which is associated with an
XO karyotype. There is an increased incidence of autoimmune disease in patients with Turner
syndrome, and full workup for both cardiovascular and genitourinary abnormalities is
essential. Abnormalities can include bicuspid aortic valve, coarctation of the aorta, single
horseshoe kidney and abnormal ureter implantation.

A Absent sense of smell

An absent sense of smell is associated with underlying Kallman syndrome where the olfactory
bulb fails to develop. This is associated with a lack of gonadotrophin-releasing hormone
(GnRH) production and consequent failure to produce luteinising hormone (LH) and follicle-
stimulating hormone (FSH) and enter puberty normally.
B Bilateral undescended testes on ultrasound scan (USS)

Bilateral undescended testes are seen in patients with an XY karyotype and androgen
insensitivity syndrome. This does not fit with the short stature seen here or with the
borderline hypertension.

D XXY karyotype

XXY karyotype is associated with Klinefelter syndrome where the patient has male external
genitalia, although with bilateral small testes, low testosterone levels and difficulties entering
sexual relationships.

E XY karyotype

An XY karyotype with female external genitalia is related to androgen insensitivity syndrome


where, although circulating testosterone levels are normal or high, there is insensitivity to
testosterone action. These patients often present in childhood with bilateral inguinal hernias
due to undescended testes, or bilateral undescended testes are identified on ultrasound scan.
51705

Rate this question:

Next Question

Previous Question Tag Question

Feedback End Review

Difficulty: Average

Peer Responses %

Q. Answered Flagged
Q. Answered Flagged

Q1

Q2

Q3

Q4

Q5

Q6

Q7

Q8
0:00:17/03:00:00

A 40-year-old woman comes to the Endocrine Clinic for review. Her periods have stopped for
more than six months and she has a follicle-stimulating hormone (FSH) level of > 40 iu/l. Her
oestrogen is in the post-menopausal range and thyroid function and prolactin are both
normal. Her body mass index (BMI) is 22 kg/m 2 and she has no gynaecological surgery, family
history of deep vein thrombosis (DVT) or breast cancer.
She asks about bone protection.
Which of the following is the most appropriate intervention for her?

A Continuous combined hormone replacement therapy (HRT)

B Combined oral contraceptive pill

C Oral bisphosphonate

D Sequential combined HRT

E Subcutaneous denosumab

Explanation 

D Sequential combined HRT

This woman has early menopause, as evidenced by her cessation of menses of more than six
months and her blood results, including a marked elevation in FSH levels. Where menses have
ceased for less than one year, a sequential, rather than continuous, HRT preparation is
recommended. The normal BMI and absence of DVT or a breast cancer history fit with a low
risk of side-effects of HRT.

A Continuous combined hormone replacement therapy (HRT)

Continuous HRT preparations are only recommended when they have received a sequential
preparation for one year or more, or it has been at least two years from their last menstrual
period when premature menopause has occurred.

B Combined oral contraceptive pill


Given the combined oral contraceptive pill contains a much higher dose of oestrogen, this is
inappropriate, given that protection against potential pregnancy is not required here.

C Oral bisphosphonate

Given long bone fractures have occurred in patients taking bisphosphonates for five to seven
years or more, beginning bisphosphonates in a 40-year-old when other options are available
is not recommended. The long bone fractures are thought to occur due to bisphosphonate-
related decreases in bone turnover.

E Subcutaneous denosumab

Subcutaneous denosumab is recommended for the treatment of osteoporosis in patients


where other options are unsuitable (ie multiple therapies are not tolerated or severe
osteoporosis is present). In this case, initiation of HRT is more appropriate.
51721

Rate this question:

Next Question

Previous Question Tag Question

Feedback End Review

Difficulty: Average

Peer Responses %

Q. Answered Flagged

Q1

Q2
Q. Answered Flagged

Q3

Q4

Q5

Q6

Q7

Q8

 External Links

NICE CKS. Menopause.


cks.nice.org.uk/menopause#
(https://cks.nice.org.uk/menopause#)
0:00:17/03:00:00

A 72-year-old man is reviewed in the clinic one month after admission to the Emergency
Department with aspiration pneumonia. He admits to difficulty swallowing, often choking on
his food, and he has weakness affecting his shoulders and hips, such that he finds it very
difficult to get up out of a low chair. He tells you that he often has muscle cramps at the end
of each day.
His blood pressure is 135/85 mmHg, and there is no postural drop. There is obvious 4/5
power weakness, and he is unable to stand from a chair without using his arms.
Investigations:

Haemoglobin (Hb) 12.8 g/dl (13.5–17.5 g/dl)

White cell count (WCC) 7.8 × 10 9/l (4–11 × 10 9/l)

Platelets (PLT) 281 × 10 9/l (150–400 × 10 9/l)

Sodium (Na +) 144 mmol/l (135–145 mmol/l)

Potassium (K +) 4.5 mmol/l (3.5–5.0 mmol/l)

Creatinine (Cr) 105 µmol/l (50–140 µmol/l)

Creatinine kinase (CK) 3105 iu/l

Which of the following investigations is most useful to establish the diagnosis?

A Anti-Jo antibody testing

B Anti-Mi-2 antibody testing

C Anti-VGCC antibody testing

D Muscle biopsy

E Thyroid function testing

Explanation 

D Muscle biopsy
This patient’s clinical presentation with bulbar palsy and proximal myopathy is consistent
with a diagnosis of polymyositis. Electromyography (EMG) helps establish the diagnosis of
polymyositis, and typical inflammatory muscle changes are confirmed with muscle biopsy.
Corticosteroids are the initial intervention of choice.

A Anti-Jo antibody testing

Only around 20% of patients with polymyositis are positive for anti-Jo antibodies. They are
associated with a poorer outcome, and interstitial lung disease is seen in around one-third of
patients with anti-Jo antibodies.

B Anti-Mi-2 antibody testing

Anti-Mi-2 antibodies are specific for dermatomyositis but are only found in around 25% of
patients with the disease. There is no evidence of rash here, which favours polymyositis.

C Anti-VGCC antibody testing

Anti-VGCC antibodies are seen in patients with Lambert–Eaton myasthenic syndrome and
result in proximal muscle weakness, but there is no rise in CK. Muscle weakness may improve
in Lambert–Eaton syndrome with reinforcement. The disease is associated with underlying
bronchial carcinoma.

E Thyroid function testing

Hypothyroidism does result in proximal muscle weakness and a small rise in CK. There is
magnitude of CK elevation seen here and the absence of other features of hypothyroidism
such as weight gain and bradycardia.
51770

Rate this question:

Next Question

Previous Question Tag Question

Feedback End Review

Difficulty: Average

Peer Responses %
Q. Answered Flagged

Q1

Q2

Q3

Q4

Q5

Q6

Q7

Q8

Q9

0:00:17/03:00:00

You are asked to take part in a clinical trial of an agent that moves the oxygen dissociation
curve to the right, making increased oxygen available for tissues in times of sepsis.
Which of the following conditions is known to do this?

A Carbon dioxide (CO 2) retention

B Decreased 2,3-bisphosphoglyceric acid (2,3-BG)

C Decreased pH

D Decreased hydrogen (H +) ion levels in the blood

E Hypothermia

Explanation 

A Carbon dioxide (CO 2) retention

You can imagine that physiologically, it would be a good idea for metabolic stress to result in
increased oxygen delivery to tissues. This happens in CO 2 retention where increased H + ions
in the blood lead to them binding to haemoglobin. Binding of H + ions promotes the formation
of the taut (T) form of haemoglobin, which is less able to bind oxygen, driving an increased
release at the tissue level.

B Decreased 2,3-bisphosphoglyceric acid (2,3-BG)

Increased levels of 2,3-BG are associated with a shift in the oxygen dissociation curve to the
right. 2,3-BG is generated by an alternative pathway to the normal glycolytic one, which
regulates oxygen release at the tissue level in times of metabolic stress.

C Decreased pH

Decreased pH results in a shift in the oxygen dissociation curve to the left, not to the right,
meaning that haemoglobin molecules more avidly hang on to oxygen, reducing their delivery
to tissues.
D Decreased hydrogen (H +) ion levels in the blood

This is the major mechanism by which a right shift in the dissociation curve exists. Because
oxygen release is mediated by H + ion levels, this allows increased oxygen delivery to tissues in
acidosis per se and does not differentiate between metabolic and respiratory acidosis states.

E Hypothermia

Hypothermia results in a left shift of the oxygen dissociation curve. This may be one reason
why cardiac arrhythmias are known to increase in patients with very low body temperature
because of relative myocardial ischaemia.
51764

Rate this question:

Next Question

Previous Question Tag Question

Feedback End Review

Difficulty: Average

Peer Responses %

Q. Answered Flagged

Q1

Q2

Q3

Q4

Q5
Q. Answered Flagged

Q6

Q7

Q8

0:00:17/03:00:00

A 49-year-old man comes to the Neurology Clinic, complaining of recurrent severe


headaches which come on predominantly at night. They are always behind the left eye and
typically last for between 45 and 90 minutes. Nasal congestion, reddening of the eye and
lacrimation are also seen. He tells you that the headaches are worse when he drinks alcohol.
Neurological examination in the clinic is entirely normal. All routine bloods are also in the
normal range.
Which of the following acute treatments is most likely to be effective for his headaches?

A High-flow oxygen therapy

B Intranasal lidocaine

C Metoclopramide

D Sumatriptan nasal spray

E Verapamil

Explanation 

A High-flow oxygen therapy

This patient’s symptoms of episodic unilateral headaches, worse at night, with nasal
congestion are entirely consistent with a diagnosis of cluster headache. High-flow oxygen is
very effective for relief of acute headache, given for a period of 15 minutes via a tight-fitting
mask. Patients are often given a card which allows them fast access via the Emergency
Department.

B Intranasal lidocaine

A total of 1 ml of 4% lidocaine, which can be given intranasally to the affected side is used as
an adjunct to high-flow oxygen, not as a primary intervention.

C Metoclopramide
Metoclopramide can be used to manage both pain and nausea associated with cluster
headache, although it is not recommended as a routine intervention because of the risk of
extra-pyramidal side-effects.

D Sumatriptan nasal spray

Sumatriptan can be used as an acute intervention for cluster headache, although it is less
effective vs high-flow oxygen. It also appears that subcutaneous sumatriptan is more
effective than the intra-nasal preparation.

E Verapamil

Verapamil is the first choice for prophylaxis in patients with cluster headache. It is
commenced at a dose of 40 mg twice daily and is slowly uptitrated to higher doses
(sometimes up to > 900 mg may be required).
51745

Rate this question:

Next Question

Previous Question Tag Question

Feedback End Review

Difficulty: Average

Peer Responses %

Q. Answered Flagged

Q1

Q2
Q. Answered Flagged

Q3

Q4

Q5

Q6

Q7

Q8
0:00:17/03:00:00

A 78-year-old woman is admitted to the Emergency Department with acute abdominal pain.
She had diarrhoea and colicky abdominal pain over the previous evening and is now confined
to bed with severe nausea and is unable to move due to pain. She has atrial fibrillation, type 2
diabetes and hypertension.
Her blood pressure is 90/70 mmHg, and pulse 120 bpm (atrial fibrillation). Her Glasgow
Coma Scale (GCS) score is 13. The abdomen is generally tender, with quiet bowel sounds.
Rectal examination reveals an empty rectum.
Investigations:

Haemoglobin (Hb) 11.9 g/dl (11.5–15.5 g/dl)

White cell count (WCC) 14.1 × 10 9/l (4–11 × 10 9/l)

Platelets (PLT) 203 × 10 9/l (150–400 × 10 9/l)

Sodium (Na +) 143 mmol/l (135–145 mmol/l)

Potassium (K +) 6.3 mmol/l (3.5–5.0 mmol/l)

Creatinine (Cr) 193 µmol/l (50–120 µmol/l)

Alanine aminotransferase (ALT) 79 iu/l (5–30 iu/l)

Alkaline phosphatase 95 iu/l (30–130 iu/l)

Bicarbonate (HCO 3) 14 mmol/l (24–30 mmol/l)

Lactate 4.5 mmol/l (0.5–2.2 mmol/l)

Glucose 4.6 mmol/l (> 3.9 mmol/l)

Bilirubin 12 µmol/l (2–17 µmol/l)

Amylase 220 iu/l (< 200 iu/l)

Which of the following is the most likely diagnosis?

A Acute appendicitis

B Ascending cholangitis

C Cholecystitis
D Acute pancreatitis

E Ischaemic bowel

Explanation 

E Ischaemic bowel

This patient is an arteriopath with both type 2 diabetes and atrial fibrillation. Given the history
of initial diarrhoea, and now abdominal pain, renal impairment with acidosis and a marked
elevation in venous lactate levels, ischaemic bowel is the most likely diagnosis. A computed
tomography (CT) angiogram is the initial investigation of choice, despite the elevated
creatinine level seen here. Early diagnosis and progression to surgery are directly linked to
outcomes.

A Acute appendicitis

Acute appendicitis is more likely to present with central or right-sided abdominal pain and
without the precipitous rise in lactate levels seen here, which is indicative of significant tissue
ischaemia.

B Ascending cholangitis

Ascending cholangitis is more likely to be associated with epigastric or right upper quadrant
pain, fevers and rigors, rather than the generalised abdominal pain and metabolic acidosis
seen here. Agents targeting Gram-negative bacteria, such as quinolones, are the intervention
of choice here.

C Cholecystitis

Cholecystitis is more likely to be associated with pyrexia, nausea, vomiting and right upper
quadrant pain and an obstructive picture on liver function testing vs the metabolic acidosis
and elevated lactate levels seen here.

D Acute pancreatitis

Acute pancreatitis is likely to be associated with a much greater elevation in amylase levels
than seen here, and a more common presentation is with epigastric pain and vomiting. A
lactate level of 4.5 mmol/l would be unusual in acute pancreatitis.
51729

Rate this question:


Next Question

Previous Question Tag Question

Feedback End Review

Difficulty: Average

Peer Responses %

Q. Answered Flagged

Q1

Q2

Q3

Q4

Q5

Q6

Q7

Q8

Q9

0:00:17/03:00:00

You are reviewing the results of a new liquid biopsy to look for lung cancer recurrence in
patients who have undergone potentially curative surgery.
The performance of the test is as follows:

Lung cancer confirmed Lung cancer negative Total

Positive screening test 170 10 180

Negative screening test 110 41 151

Total 280 51

Which of the following is the sensitivity of this screening test?

A 27%

B 61%

C 80%

D 81%

E 94%

Explanation 

B 61%

Sensitivity is the number of true positives (170), divided by the number of true positives +
false negatives (280). This makes 170/280 = 61%.

A 27%

This the negative predictive value, the percentage of patients who test negative who actually
do not have the disease, which is 41/(110 + 41) = 27%.

C 80%
This is the specificity (true negatives/true negatives + false positives), which equals 41/(41 +
10) or 81%.

D 81%

This does not fit with any of the recognised calculations to do with the performance of a
screening test (sensitivity, specificity, positive or negative predictive value).

E 94%

This is the positive predictive value, the percentage of patients who test positive who actually
have the disease (170/170 + 10).
51708

Rate this question:

Next Question

Previous Question Tag Question

Feedback End Review

Difficulty: Average

Peer Responses %

Q. Answered Flagged

Q1

Q2

Q3

Q4
Q. Answered Flagged

Q5

Q6

Q7

Q8 
0:00:17/03:00:00

A 27-year-old woman is referred to the Haematology Clinic after presenting to her 12-week
pregnancy check with an abnormal full blood count.
She is otherwise well, runs some 20 km per week and takes no regular medication. Her
parents are originally from Greece and have no significant family history of medical illnesses.
Investigations:

10.9 g/dl (mean corpuscular volume (11.5–15.5 g/dl) (MCV 76–


Haemoglobin (Hb)
(MCV) 74 fl) 98 fl)

White cell count


9.2 × 10 9/l (4–11 × 10 9/l)
(WCC)

Platelets (PLT) 231 × 10 9/l (150–400 × 10 9/l)

Sodium (Na +) 144 mmol/l (135–145 mmol/l)

Potassium (K +) 4.5 mmol/l (3.5–5.0 mmol/l)

Creatinine (Cr) 92 µmol/l (50–120 µmol/l)

Which of the following is the most appropriate investigation to confirm the underlying
diagnosis?

A Bone marrow aspiration

B Cytogenetic studies

C Haemoglobin electrophoresis

D Serum ferritin

E Skeletal survey

Explanation 

C Haemoglobin electrophoresis
The most likely diagnosis here is beta-thalassaemia trait, given this patient’s Mediterranean
origin and lack of symptoms of iron deficiency despite microcytosis on complete blood count
testing. Haemoglobin electrophoresis demonstrates raised HbA 2. Confirmation of the
diagnosis is important, because iron supplementation may actually be detrimental in this
situation and should be avoided.

A Bone marrow aspiration

There is no suggestion of an underlying haematological malignancy or myelodysplastic


syndrome here, given this patient is well and has an isolated low MCV. Bone marrow
aspiration does not therefore have a role in the management of this patient.

B Cytogenetic studies

Cytogenetic studies are used to differentiate between haematological malignancies and


myelodysplastic syndromes. They do not have a role in diagnosing thalassaemia. In some
cases, DNA testing may be considered in patients with thalassaemia trait to guide
management where couples are at risk of having a child with thalassaemia major.

D Serum ferritin

In this case, serum ferritin is likely to be elevated, as it is an acute phase protein and the
patient is likely to be iron-replete. As such, it does not play a role in diagnosing the cause of
this patient’s microcytosis.

E Skeletal survey

Skeletal surveys only have a role in the evaluation of thalassaemia major where they may
show classical changes such as a hair-on-end appearance on skull X-ray and long bone
deformity. Any skeletal survey reviewed here is likely to be normal.
51685

Rate this question:

Next Question

Previous Question Tag Question

Feedback End Review

Difficulty: Average

Peer Responses %
Q. Answered Flagged

Q1

Q2

Q3

Q4

Q5

Q6

Q7

Q8

Q9

0:00:17/03:00:00

A 55-year-old man is referred to the Renal Clinic. His general practitioner (GP) is concerned,
as he was unable to tolerate ramipril because of a rise in his potassium level one week after
starting therapy. His creatinine level remained relatively stable at 125 mmol/l. His blood
pressure is 155/85 mmHg, and pulse 78 bpm and regular. Heart sounds are normal, and no
bruits are identified. Body mass index (BMI) is 24 kg/m 2.
Investigations:

Haemoglobin (Hb) 12.9 g/dl (13.5–17.5 g/dl)

White cell count (WCC) 7.1 × 10 9/l (4–11 × 10 9/l)

Platelets (PLT) 281 × 10 9/l (150–400 × 10 9/l)

Sodium (Na +) 135 mmol/l (135–145 mmol/l)

Potassium (K +) 5.8 mmol/l (3.5–5.0 mmol/l)

Bicarbonate (HCO 3) 17 mmol/l (24–30 mmol/l)

Creatinine (Cr) 125 µmol/l (50–120 µmol/l)

Which of the following is the most likely cause of his hyperkalaemia?

A Hyporeninaemic hyperaldosteronism

B Renal artery stenosis

C Renal tubular acidosis type 1

D Renal tubular acidosis type 2

E Renal tubular acidosis type 4

Explanation 

E Renal tubular acidosis type 4

This patient has hyperkalaemic metabolic acidosis which worsened with the introduction of
an angiotensin-converting enzyme (ACE) inhibitor. The most likely diagnosis therefore is one
of renal tubular acidosis type 4, which is related to deficient aldosterone action. Causes may
include diabetes mellitus, systemic lupus erythematosus, amyloidosis and chronic
tubulointerstitial disease.

A Hyporeninaemic hyperaldosteronism

Hyporeninaemic hyperaldosteronism causes hypokalaemia with metabolic alkalosis and


hypertension related to an adrenal adenoma or bilateral adrenal hyperplasia driving increased
production of aldosterone.

B Renal artery stenosis

Renal artery stenosis is associated with a rise in both potassium and creatinine levels on
starting ACE inhibitor therapy because of decreased renal perfusion. If anything, in the
absence of ACE inhibitor therapy, a potassium level towards the lower end of the normal
range would be expected.

C Renal tubular acidosis type 1

Renal tubular acidosis type 1 is associated with hypokalaemia and metabolic acidosis. Causes
include autoimmune renal disease, hyperparathyroidism, analgesic nephropathy and chronic
urinary tract infection. The cause of renal tubular acidosis type 1 is an inability to form acid
urine in the distal tubule. Potassium supplementation is the initial intervention of choice.

D Renal tubular acidosis type 2

Renal tubular acidosis type 2, like type 1, is associated with hypokalaemic metabolic acidosis.
It is often associated with Fanconi syndrome and leads to excess bicarbonate excretion in the
urine. High-dose bicarbonate supplementation is the initial intervention of choice.
51749

Rate this question:

Next Question

Previous Question Tag Question

Feedback End Review

Difficulty: Average

Peer Responses %
Q. Answered Flagged

Q1

Q2

Q3

Q4

Q5

Q6

Q7

Q8

Q9

0:00:17/03:00:00

A 29-year-old man is being treated on the Medical Ward after a mixed overdose. He is known
to have a long psychiatric history and has previously presented with episodes of acute
psychosis. He awoke from an unconscious state some 30 minutes ago and has been trying to
leave the ward. The psychiatric nurse proposes to detain him under the Mental Health Act.
Which of the following is the most appropriate of the Mental Health Act under which to hold
him?

A 2

B 3

C 52

D 54

E 7

Explanation 

D 54

Section 5 4 allows a psychiatric nurse to detain the patient for a period of up to six hours.
Section 5 4 is valid if they determine that the patient poses a significant risk either to
themselves or to others. During the course of the six hours, they must make appropriate
arrangements to obtain Section 5 2 permission or allow the patient to leave when the time
lapses.

A 2

This is the Section of the Mental Health Act which allows compulsory detention for mental
health assessment for a period of 28 days. An approved mental health professional makes the
application on the recommendation of two senior doctors, one of whom must fit the criteria
for Section 12 2 of the Act (a consultant psychiatrist or a very senior specialty registrar). A
Section 2 cannot be renewed.

B 3
Section 3 is an admission for treatment order under the Mental Health Act where the exact
mental disorder must be stated, and detention can be renewed for a further six months and
then annually after that. Two doctors must sign the appropriate forms and state that the
patient cannot be treated in the community.

C 52

This order allows the doctor in charge (or, where the person in charge is a consultant
psychiatrist, his or her deputy) to detain a patient for up to 72 hours. This order cannot be
used for patients in the Emergency Department because they have not technically been
admitted to the hospital.

E 7

Section 7 is an application for guardianship; it is used to enable patients to receive


community care where it cannot be provided without the use of compulsory powers. The
guardian is usually a mental health social worker, and the order specifies a place to live and a
place where treatment is administered.
51761

Rate this question:

Next Question

Previous Question Tag Question

Feedback End Review

Difficulty: Average

Peer Responses %

Q. Answered Flagged

Q1
Q. Answered Flagged

Q2

Q3

Q4

Q5

Q6

Q7

Q8

 External Links

NHS: Mental Health Act


nhs.uk/using-the-nhs/nhs-services/mental-health-services/mental-health-act/
(https://www.nhs.uk/using-the-nhs/nhs-services/mental-health-services/mental-
health-act/)
0:00:17/03:00:00

A 61-year-old man presents to the Respiratory Clinic with gradually worsening shortness of
breath on exertion, a dry cough and progressive finger clubbing. His level of energy has been
gradually decreasing over the past six months, and he has lost 5 kg in weight. His blood
pressure is 138/84 mmHg, and pulse 78 bpm and regular. There are bilateral inspiratory
crackles on auscultation of the chest.
Investigations:

Haemoglobin (Hb) 13.1 g/dl (13.5–17.5 g/dl)

White cell count (WCC) 9.8 × 10 9/l (4–11 × 10 9/l)

Platelets (PLT) 309 × 10 9/l (150–400 × 10 9/l)

Sodium (Na +) 144 mmol/l (135–145 mmol/l)

Potassium (K +) 4.5 mmol/l (3.5–5.0 mmol/l)

Creatinine (Cr) 95 µmol/l (50–140 µmol/l)

Glucose 6.1 mmol/l

Chest X-ray Bilateral interstitial shadowing

Forced expiratory volume in 1 second


0.82
(FEV1)/forced vital capacity (FVC) ratio

FVC 60% of predicted

Which of the following medications has the best evidence for improving outcomes in this
case?

A Cyclophosphamide

B Methylprednisolone

C Nintedanib

D Roflumilast

E Tacrolimus
Explanation 

C Nintedanib

Nintedanib is an intracellular tyrosine kinase inhibitor with antifibrotic and anti-inflammatory


properties. It is approved by the National Institute for Health and Care Excellence (NICE),
under similar guidance to pirfenidone, where it can be used if the FVC is between 50% and
80% and therapy should be discontinued if disease progression is seen (a worsening of 10%
in FVC).

A Cyclophosphamide

Cyclophosphamide has limited efficacy in patients with idiopathic pulmonary fibrosis (IPF)
where response rates in small clinical trials are 50% or less. Therapy is often also at the
expense of significant bone marrow depression, limiting its use.

B Methylprednisolone

Methylprednisolone has been trialed in pulsed therapy regimens with cyclophosphamide.


Effects on progression of IPF have been very modest, and the side-effect profile of high-dose
corticosteroids in the age group most affected by IPF significantly limits their use.

D Roflumilast

Roflumilast is a phosphodiesterase type 4 (PDE-4) inhibitor used in the treatment of chronic


obstructive pulmonary disease (COPD) where patients have persistent symptoms despite
triple inhaled therapy with a long-acting muscarinic antagonist, a long-acting beta agonist
and an inhaled corticosteroid.

E Tacrolimus

Tacrolimus has shown benefit in reducing exacerbations of IPF in a number of small studies.
Evidence is stronger, however, to support the use of nintedanib or pirfenidone.
51766

Rate this question:

Next Question

Previous Question Tag Question

Feedback End Review


Difficulty: Average

Peer Responses %

Q. Answered Flagged

Q1

Q2

Q3

Q4

Q5

Q6

Q7

Q8

Q9

 External Links

Nintedanib for treating idiopathic pulmonary fibrosis


nice.org.uk/guidance/ta379/chapter/1-Recommendations
(https://www.nice.org.uk/guidance/ta379/chapter/1-Recommendations)
0:00:17/03:00:00

A 23-year-old woman is referred to the Cardiology Clinic in the twelfth week of pregnancy.
She has been noted to have a systolic murmur, and the general practitioner (GP) is
requesting advice on diagnosis and further management. She has no past medical history of
note and regularly plays squash.
Examination in the clinic reveals a loud pan-systolic murmur, best heard in the third left
intercostal space where there is also a palpable thrill. There are no signs of cyanosis or
associated cardiac failure. Her blood pressure is 115/82 mmHg, and her pulse is 65 bpm and
regular. Oxygen saturation is 93% on air.

What is the best advice with respect to her pregnancy?

A She has a high risk of complications and should be offered a termination

B She can be reassured that no intervention is likely to be required

C She should be started on an angiotensin-converting enzyme (ACE) inhibitor

D She should be started on methyldopa

E She will require percutaneous valvotomy

Explanation 

B She can be reassured that no intervention is likely to be required

This patient has had no symptoms of cardiac insufficiency and has a loud systolic murmur
consistent with a ventricular septal defect (VSD). Small VSDs are not likely to lead to
complications in pregnancy, and the patient can therefore be initially reassured. An
echocardiogram is of course required to confirm the diagnosis. Unrepaired cyanotic heart
defects and Eisenmenger syndrome are associated with much higher rates of complications.

A She has a high risk of complications and should be offered a termination

Isolated small VSDs, the likely diagnosis here, are associated with < 1% chance of
complications occurring in pregnancy. Those with high-risk cardiovascular conditions, such as
Eisenmenger or untreated severe aortic stenosis, should be offered advice about the risks of
continuing with the pregnancy.
C She should be started on an angiotensin-converting enzyme (ACE) inhibitor

This patient has no signs of cardiac failure, and her blood pressure is normal. ACE inhibitors
are also teratogenic and should be avoided in pregnancy. There is no role for this class of
agents in the management of small VSDs.

D She should be started on methyldopa

Methyldopa is a standard intervention for the treatment of hypertension in pregnancy. Given


this patient’s blood pressure is in the normal range, there is no indication for its use here.

E She will require percutaneous valvotomy

Percutaneous valvotomy is used for the treatment of mitral stenosis in pregnancy. Mitral
stenosis is associated with a progressive decrease in exercise tolerance and a mid-diastolic
murmur best heard in the left lateral position.
51682

Rate this question:

Next Question

Previous Question Tag Question

Feedback End Review

Difficulty: Average

Peer Responses %

Q. Answered Flagged

Q1
Q. Answered Flagged

Q2

Q3

Q4

Q5

Q6

Q7

Q8
0:00:17/03:00:00

You receive a crash call from the Emergency Department (ED) to review a 56-year-old man
with alcoholic cirrhosis who has become acutely unwell, following a blood transfusion for
variceal haemorrhage. When you arrive, there is also a trauma patient being managed by ED
staff. The nurse managing your patient has just stopped a blood transfusion as, within five to
ten minutes of the transfusion beginning, his temperature has risen by 1.5 °C, his blood
pressure has actually fallen by 20 mmHg and his pulse has increased by 25 bpm. He also
complains of severe abdominal and muscle cramps, and he is acutely short of breath; his
respiratory rate has increased to 30 breaths per minute.

Which of the following is the most likely diagnosis?

A ABO incompatibility

B Bacterial contamination of the unit

C Febrile non-haemolytic reaction

D Graft-versus-host disease

E Transfusion-associated lung injury (TRALI)

Explanation 

A ABO incompatibility

The scenario here with an acutely unwell patient whose symptoms have deteriorated a very
short time after beginning blood transfusion is typical of that seen for ABO incompatibility. A
mix-up of transfusions may have occurred between this and the trauma patient. Retyping of
donor and recipient red blood cells is essential to confirm the diagnosis, with a review of
transfusion procedures in the ED if the diagnosis is confirmed. Other evidence of acute
haemolytic reaction seen in the patient may include a further fall in Hb (although this is
difficult to differentiate from variceal bleeding as a cause) and evidence of red cell
destruction with elevated lactate dehydrogenase (LDH). The direct antiglobulin test will be
positive in this patient.

B Bacterial contamination of the unit


Bacterial contamination of red cell units is extremely rare; it is much more commonly seen in
platelet transfusions, as they are stored in the blood bank at a higher temperature. Release of
bacterial endotoxin may cause a similar picture to that seen here, with hypotension and
tachycardia, although onset is likely to be more gradual.

C Febrile non-haemolytic reaction

This is usually characterised by a rise in temperature without the signs of shock appearing
here after relatively minimal transfusion. It can, of course, be managed with anti-pyretics and
continuing the transfusion.

D Graft-versus-host disease

This is seen when non-irradiated transfusions are given to patients with an underlying
haematological malignancy such as Hodgkin’s disease. It is not relevant here in a patient who
has suffered a variceal haemorrhage.

E Transfusion-associated lung injury (TRALI)

TRALI is associated with bilateral pulmonary infiltrates and marked hypoxia seen on blood
gas analysis. It is caused by donor antibodies against the patient’s own leukocytes, and onset
of symptoms occurs within six hours of a transfusion.
51687

Rate this question:

Next Question

Previous Question Tag Question

Feedback End Review

Difficulty: Average

Peer Responses %
Q. Answered Flagged

Q1

Q2

Q3

Q4

Q5

Q6

Q7

Q8

Q9

0:00:17/03:00:00

A 62-year-old man who has type 2 diabetes comes to the Emergency Department with
profuse sweating and central crushing chest pain radiating down his left arm. His blood
pressure is 95/60 mmHg and pulse 45 bpm and regular. A 12-lead electrocardiogram (ECG)
reveals ST elevation in leads II, III and aVF, with reciprocal ST depression in lead I.
Which of the following arteries is most likely to have been occluded?

A Left anterior descending artery (LAD)

B Left circumflex artery

C Left main stem coronary artery

D Obtuse marginal artery

E Right coronary artery

Explanation 

E Right coronary artery

This patient’s ECG changes are typical of those seen in an inferior myocardial infarction.
Around 80% of inferior myocardial infarctions are due to occlusion of a dominant right
coronary artery. This is particularly suggested by the reciprocal ST depression in lead I, which
is not usually seen in patients with a left circumflex artery occlusion.
Coronary artery ’dominance‘ is defined in terms of which coronary artery supplies the
posterior descending artery and the posterior left ventricular branches.

A Left anterior descending artery (LAD)

LAD lesions result in ST elevation affecting the anterior and lateral leads, with ST elevation
seen in V1–V6. Where there is a very severe proximal occlusion, then ST elevation may extend
to the inferior leads.

B Left circumflex artery


Left circumflex artery occlusion can be the cause of up to 20% of inferior myocardial
infarctions, although ST depression is usually absent in lead I. Lateral ST elevation may also
be seen on the ECG.

C Left main stem coronary artery

This leads to both anterior and inferior ST elevation, and there is limited opportunity to
develop collateral vessels, so that where very proximal left coronary artery occlusion exists,
many patients actually present with cardiac arrest.

D Obtuse marginal artery

The obtuse marginal artery supplies the lateral wall of the left ventricle. Isolated occlusion of
the obtuse marginal artery therefore results in changes affecting the lateral ECG leads only.
51700

Rate this question:

Next Question

Previous Question Tag Question

Feedback End Review

Difficulty: Average

Peer Responses %

Q. Answered Flagged

Q1

Q2

Q3
Q. Answered Flagged

Q4

Q5

Q6

Q7

Q8 
0:00:17/03:00:00

A 45-year-old man presents to the Emergency Department with a sudden-onset headache at


the back of his head and neck – the ‘worst headache he has ever had’. He has a computed
tomography (CT) head which is suspicious for a subarachnoid haemorrhage (SAH). His
Glasgow Coma scale (GCS) score is 15, although he is complaining of a severe headache. His
blood pressure is 182/100 mmHg. He wants to know the percentage chance that he has really
had an SAH.
A table of results of CT scanning done by a junior in the hospital is shown below:

SAH confirmed SAH negative Total

CT scan positive 90 15 105

CT scan negative 10 110 120

Total 100 125

Which of the following is the positive predictive value for CT scanning in the diagnosis of
SAH?

A 10%

B 86%

C 88%

D 90%

E 92%

Explanation 

B 86%

The positive predictive value is the number of true positives divided by the number of (true
positives + false positives). This equates to 90/(90 + 15) = 86%.

A 10%
10% is merely the total percentage of negative CT scans out of the total number of scans
where SAH was confirmed, ie the opposite of the sensitivity.

C 88%

This is the specificity of the test. This is the number of true negatives divided by the number
of (true negatives + false positives), which is 110/(110 + 15).

D 90%

90% is the sensitivity of the test. This is the number of true positives divided by the number
of (true positives + false negatives), which is 90/(90 + 10) = 90%.

E 92%

This is the negative predictive value, ie the chance of a negative CT scan really being
associated with no underlying SAH. It is calculated as the number of true negatives divided
by the number of (true negatives + false negatives), which is 110/(110 + 10) = 92%.
51709

Rate this question:

Next Question

Previous Question Tag Question

Feedback End Review

Difficulty: Average

Peer Responses %

Q. Answered Flagged
Q. Answered Flagged

Q1

Q2

Q3

Q4

Q5

Q6

Q7

Q8
0:00:17/03:00:00

A 72-year-old woman comes to the Urology Clinic for review. She tells you that she suffers
from incontinence two to three times per day where she has an uncontrollable urge to pass
urine. She often does not make it to the toilet in time. She has osteoarthritis of both hips and
is awaiting bilateral elective hip replacements. Her body mass index is 23 kg/m 2.
Which of the following is the most likely cause of her symptoms?

A Bladder outflow obstruction

B Detrusor overactivity

C Increased intra-abdominal pressure

D Urethral prolapse

E Uterine prolapse

Explanation 

B Detrusor overactivity

What is described here is classical urge incontinence, which is driven by detrusor overactivity.
Interventions to improve symptoms can include a reduction in caffeine intake and bladder
training. Where patients fail to respond to these interventions, treatment with anticholinergic
drugs can be considered, although risks and benefits should be viewed against data
suggesting they are associated with cognitive impairment in long-term use.

A Bladder outflow obstruction

Bladder outflow obstruction is associated with continuous feelings of a full bladder, pain
during urination, problems starting urination and uneven flow. The most common cause of
bladder outflow obstruction is prostatic hypertrophy.

C Increased intra-abdominal pressure

Increased intra-abdominal pressure is a cause of incontinence, although here, where the body
mass index is 23 kg/m 2, this is unlikely to be contributing significantly to her symptoms.
D Urethral prolapse

Urethral prolapse can lead to urgency and urinary frequency, although it is more rarely seen
than detrusor overactivity and is associated with dysuria and haematuria.

E Uterine prolapse

Uterine prolapse is more likely to be associated with stress incontinence, rather than the
symptoms seen here of urge incontinence. Patients also complain of a sensation of heaviness
in the pelvis, tissue protruding from the vagina and/or trouble passing a bowel movement.
51723

Rate this question:

Next Question

Previous Question Tag Question

Feedback End Review

Difficulty: Average

Peer Responses %

Q. Answered Flagged

Q1

Q2

Q3

Q4

Q5
Q. Answered Flagged

Q6

Q7

Q8 
0:00:17/03:00:00

You are asked to review one of the nurses in the Emergency Department who has received a
needlestick injury while assessing an intravenous (IV) drug abuser. He is unaware if he has
ever been vaccinated.
The patient consented to hepatitis B serology testing, and his results are shown below:
HBsAg negative
Anti-HBc IgG negative
Anti-HBc IgM negative
Anti-HBs positive

Which of the following fits best with this serology picture?

A Acute hepatitis B virus (HBV) infection

B Chronic active HBV infection

C Previous vaccination

D Resolved HBV infection and subsequently immune

E Susceptibility to hepatitis B infection

Explanation 

C Previous vaccination

The absence of core antibodies and surface antigen negativity fit best with previous hepatitis
B vaccination. It is likely he was offered the vaccination as part of a health promotion
programme.

A Acute hepatitis B virus (HBV) infection

Acute infection with HBV is associated with surface antigen positivity, surface antibody
negativity and anti-HBc IgM antibodies, which remain positive for around six months after
acute infection.
B Chronic active HBV infection

Chronic active infection has a similar picture to acute infection, although anti-HBc antibodies
are IgG, rather than IgM, antibodies.

D Resolved HBV infection and subsequently immune

Resolved HBV infection is associated with absence of hepatitis B antigen and positive
antibodies to both hepatitis B core and surface antigens.

E Susceptibility to hepatitis B infection

This would be associated with both negative antigen testing and negative antibody testing. In
this case, given this patient is an IV drug abuser, in the event his serology profile was
classified as susceptible to infection, vaccination would be offered.
51737

Rate this question:

Next Question

Previous Question Tag Question

Feedback End Review

Difficulty: Average

Peer Responses %

Q. Answered Flagged

Q1

Q2
Q. Answered Flagged

Q3

Q4

Q5

Q6

Q7

Q8 

 External Links

Interpretation of Hepatitis B Serologic Test Results


cdc.gov/hepatitis/hbv/pdfs/serologicchartv8.pdf
(https://www.cdc.gov/hepatitis/hbv/pdfs/serologicchartv8.pdf)
0:00:17/03:00:00

A 19-year-old man is brought to the Emergency Department by the police under a Section
136 order. He was found running down the high street, extremely agitated, believing he was
being pulled by three horses. He claims not to have slept for the past two days and has taken
both ecstasy and ketamine recently. He is non-compliant with your assessment in the hospital
and believes that you have come to kill him.
Which of the following is the most appropriate intervention?

A Intramuscular (im) haloperidol

B im lorazepam

C Oral atenolol

D Oral lithium

E Oral sodium valproate

Explanation 

A Intramuscular (im) haloperidol

This patient is agitated and paranoid, with likely drug-induced psychosis. He is currently
unmanageable, making im haloperidol the most appropriate antipsychotic option here.
Where patients will take oral agents, then risperidone, quetiapine and olanzapine are all
potential initial interventions.

B im lorazepam

Lorazepam is an ideal option to manage agitation but would only be considered here if
haloperidol was contraindicated and the patient was refusing alternative oral antipsychotics.

C Oral atenolol

Beta blockade is not used in the management of agitation due to drug-induced psychosis. It
is not used for the treatment of myocardial ischaemia in cocaine or metamphetamine
overdose because of the risk of unopposed alpha agonism.
D Oral lithium

Lithium is used as a prophylactic intervention in patients with bipolar disorder. It can take up
to six months to reach maximum efficacy and is therefore not of value here in the acute
management of drug-induced psychosis.

E Oral sodium valproate

Oral sodium valproate is not recommended as acute therapy for drug-induced psychosis. It is
used as a long-term intervention for bipolar disorder where lithium is unsuitable or not
tolerated. Oral sodium valproate is not used in women of childbearing potential.
51760

Rate this question:

Next Question

Previous Question Tag Question

Feedback End Review

Difficulty: Average

Peer Responses %

Q. Answered Flagged

Q1

Q2

Q3

Q4

Q5
Q. Answered Flagged

Q6

Q7

Q8

0:00:17/03:00:00

A 27-year-old man is referred to the Respiratory Clinic by his general practitioner (GP) who is
concerned about a deterioration in his asthma control. The patient paid for private patch
testing and was found to be allergic to trees and grasses and feels that his symptoms are
worse on exercise and if the weather is stormy. He is a non-smoker. Current medications
include inhaled beclomethasone 400 micrograms twice daily (bd) and as required (prn)
salbutamol.
Examination in the clinic reveals a blood pressure of 112/82 mmHg and a pulse of 67 bpm and
regular. There is mild wheeze on auscultation of the chest. His peak expiratory flow rate
(PEFR) is 389 litres/min (predicted 595). Routine bloods are unremarkable.

Which of the following is the most appropriate next step in his asthma management?

A Add prn ipratropium

B Add montelukast

C Add roflumilast

D Add salmeterol

E Increase inhaled corticosteroid to 800 micrograms bd

Explanation 

B Add montelukast

The National Institute for Health and Care Excellence (NICE) guidelines from 2017
recommend adding montelukast to inhaled corticosteroids if patients are not achieving
control on steroids alone. This is in contrast to the British Thoracic Society (BTS)/Scottish
Intercollegiate Guidelines Network (SIGN) guidelines from the previous year which still
recommend adding in a long-acting beta agonist. Because not everyone responds to
montelukast, the guidelines also recommend reassessing response to therapy at four to eight
weeks and, if there is not an adequate response, considering a long-acting beta agonist at
this stage.

A Add prn ipratropium


prn ipratropium is not useful in the chronic management of asthma. It is considered acutely in
nebulised form for patients presenting with asthma exacerbations, and long-acting
muscarinic antagonists, such as tiotropium, are an important part of the management of
chronic obstructive pulmonary disease (COPD).

C Add roflumilast

Roflumilast is an oral phosphodiesterase type 4 (PDE-4) inhibitor. It is used in the treatment


of COPD where patients have failed to gain an improvement in their symptoms on maximal
inhaled therapy (triple with a long-acting muscarinic antagonist, a long-acting beta agonist
and inhaled corticosteroids).

D Add salmeterol

Adding a long-acting beta agonist is the most appropriate next step if patients fail to gain
control of their symptoms when montelukast is added to their inhaled corticosteroid dose.

E Increase inhaled corticosteroid to 800 micrograms bd

Conventional treatment of asthma suggests that addition of a second therapy is preferable to


increasing the inhaled corticosteroid dose because of limited efficacy benefits going to very
high doses of corticosteroids and the potential downside of increased systemic absorption
and steroid-related adverse events.
51765

Rate this question:

Next Question

Previous Question Tag Question

Feedback End Review

Difficulty: Average

Peer Responses %
Q. Answered Flagged

Q1

Q2

Q3

Q4

Q5

Q6

Q7

Q8

Q9

 External Links

National Institute for Health and Care Excellence. (2017). Asthma: diagnosis, monitoring and chronic asth…
nice.org.uk/guidance/ng80
(https://www.nice.org.uk/guidance/ng80)
0:00:17/03:00:00

A 28-year-old woman who is 38 weeks pregnant presents to the Emergency Department


with a severe, sudden-onset bilateral headache, blurred vision and dizziness. Her dizziness
persists, and the following morning, she is reviewed on the ward. She has a residual dull
headache, and her blood pressure is 100/70 mmHg and pulse 82 bpm and regular. Visual
field testing reveals bitemporal hemianopia. Routine bloods are unremarkable.
Which of the following is the most likely diagnosis?

A Cluster headache

B Ischaemic stroke

C Migraine

D Pituitary apoplexy

E Subarachnoid haemorrhage

Explanation 

D Pituitary apoplexy

The bitemporal hemianopia suggests pressure on the optic chiasm, which raises the
possibility of a pituitary enlargement. Haemorrhage into the gland is more common in late
pregnancy and fits with the sudden-onset headache seen here, in the absence of features
consistent with a stroke (ie the blood pressure is low-normal). Magnetic resonance imaging is
indicated to confirm the diagnosis, and given the stage of pregnancy, delivery should be
considered around a week later, under hydrocortisone cover. This should be followed with
pituitary function testing.

A Cluster headache

Cluster headache is characterised by recurrent unilateral headaches, usually around the eye.
They are associated with redness and tearing of the eye and are extremely painful.

B Ischaemic stroke
The very localised symptoms consistent with optic chiasm pressure and the normal blood
pressure both count against a diagnosis of ischaemic stroke. This patient also has no risk
factors for stroke, as far as we are aware.

C Migraine

Migraine is associated with an aura (a visual disturbance) and sensitivity to light. Migraines
are unilateral and are known to increase in frequency during pregnancy in some patients.
Ergot-derived therapies for migraine are not recommended in pregnancy because they can
cause uterine contractions and increase the risk of miscarriage.

E Subarachnoid haemorrhage

This does not fit with the clinical scenario here. Sudden-onset occipital headache would be
expected, with increasing drowsiness, confusion and other focal neurological deficits.
Hypertension, not normal blood pressure, is a feature of subarachnoid haemorrhage.
51742

Rate this question:

Next Question

Previous Question Tag Question

Feedback End Review

Difficulty: Average

Peer Responses %

Q. Answered Flagged

Q1

Q2
Q. Answered Flagged

Q3

Q4

Q5

Q6

Q7

Q8 
0:00:17/03:00:00

A 23-year-old woman presents to the Sexually Transmitted Diseases Clinic with urethral
discharge and pain on passing urine a few days after returning from a holiday to Spain. She
admits to unprotected sex during the holiday.
Screening identifies her as infected with gonorrhoea. She is allergic to penicillin, with a
history of anaphylaxis.
Which of the following is the most appropriate intervention?

A Azithromycin 2 g

B Ceftriaxone 500 mg intramuscular (im) with azithromycin 1 g

C Ciprofloxacin 500 mg twice daily (bd) for one week

D Co-amoxiclav 625 mg three times daily (tds) for one week

E Doxycycline 100 mg twice daily (bd) for one week

Explanation 

A Azithromycin 2 g

In this situation, a single high dose of azithromycin is warranted to achieve clearance of


gonorrhoea infection. It is associated with a high incidence of gastrointestinal side-effects,
and a test of clearance is required as resistance to a single agent may be a problem.

B Ceftriaxone 500 mg intramuscular (im) with azithromycin 1 g

Given this patient has suffered from anaphylaxis to penicillin previously, it is not appropriate
to intervene with cephalosporins here. As such, a single high dose of a macrolide is more
appropriate.

C Ciprofloxacin 500 mg twice daily (bd) for one week

Quinolones are no longer recommended as first-line treatment for gonorrhoea infection


because of resistance. They can be considered for patients who have previously responded to
them.
D Co-amoxiclav 625 mg three times daily (tds) for one week

Co-amoxiclav contains amoxicillin, a penicillin; it is therefore not an appropriate option here.

E Doxycycline 100 mg twice daily (bd) for one week

Doxycycline therapy is the intervention of choice for chlamydia infection. It does not have a
role in the treatment of gonococcal infection.
51691

Rate this question:

Next Question

Previous Question Tag Question

Feedback End Review

Difficulty: Average

Peer Responses %

Q. Answered Flagged

Q1

Q2

Q3

Q4

Q5

Q6
Q. Answered Flagged

Q7

Q8
0:00:17/03:00:00

A venous blood gas sample is received in the biochemistry lab for processing. The only
information you have is that the patient is ‘unwell’.
Results are as follows:

Sodium (Na +) 138 mmol/l (135–145 mmol/l)

Potassium (K +) 3.4 mmol/l (3.5–5.0 mmol/l)

Bicarbonate (HCO 3 –) 13 mmol/l (24–30 mmol/l)

Chloride (Cl –) 116 mmol/l (98–106 mmol/l)

Which of the following is the most likely cause of this patient’s acidosis?

A Diarrhoea

B Diabetic ketoacidosis

C Ethylene glycol overdose

D Lactic acidosis

E Sampling error

Explanation 

A Diarrhoea

This patient has a normal anion gap metabolic acidosis (138 + 3.4 –13 – 116 = 12.4). Given the
potassium level is also low at 3.4 mmol/l, the most likely diagnosis for this patient would be
acidosis as a result of diarrhoea and potassium loss into the faeces. Chloride is relatively well
preserved in this situation because gastrointestinal fluid losses contain higher amounts of
sodium, compared to chloride.

B Diabetic ketoacidosis
Diabetic ketoacidosis is associated with the production of ketones, including acetone and
beta-hydroxybutyrate. These lead to a raised anion gap metabolic acidosis, which does not fit
with the scenario here.

C Ethylene glycol overdose

Ethylene glycol, like diabetic ketoacidosis, is associated with a raised anion gap metabolic
acidosis. The intervention of choice for ethylene glycol overdose is fomepizole, which is a
competitive inhibitor of alcohol dehydrogenase, reducing the production of toxic metabolites,
including calcium oxalate.

D Lactic acidosis

Lactic acidosis also leads to a raised anion gap metabolic acidosis. It is most often seen when
there is decreased tissue perfusion, eg in trauma or shock, and may also be associated with
metformin overdose. The normal range for the calculated anion gap, including potassium, is
usually considered to be 10–20.

E Sampling error

Given the values fit with metabolic acidosis due to diarrhoea, with a normal anion gap and
low serum potassium level, and the patient is ‘unwell’, sampling error seems unlikely here.
51701

Rate this question:

Next Question

Previous Question Tag Question

Feedback End Review

Difficulty: Average

Peer Responses %
Q. Answered Flagged

Q1

Q2

Q3

Q4

Q5

Q6

Q7

Q8

Q9

0:00:17/03:00:00

A 71-year-old man is admitted to the Emergency Department after collapsing at the airport
on his return from a convention hotel in Florida. According to his wife, he became unwell
around 48 hours earlier with a dry cough, fevers and worsening confusion.
On arrival in the Emergency Department, his blood pressure is 100/70 mmHg, and pulse 105
bpm and regular. His temperature is 39.2 °C. There are coarse crackles on auscultation of the
chest over the right lower zone. His abdomen is soft, and there is mild tenderness on
palpation. He has soiled himself with diarrhoea.
Investigations:

Haemoglobin (Hb) 11.9 g/dl (13.5–17.5 g/dl)

White cell count (WCC) 11.2 × 10 9/l (4–11 × 10 9/l)

Platelets (PLT) 135 × 10 9/l (150–400 × 10 9/l)

Sodium (Na +) 122 mmol/l (135–145 mmol/l)

Potassium (K +) 4.5 mmol/l (3.5–5.0 mmol/l)

Creatinine (Cr) 149 µmol/l (50–120 µmol/l)

Alanine aminotransferase (ALT) 85 IU/l (5–30 IU/l)

Alkaline phosphatase 210 IU/l (30–130 IU/l)

Bilirubin 13 mmol/l (2–17 mmol/l)

Chest X-ray Right lower lobe consolidation

Which of the following is the most appropriate intervention?

A Azithromycin

B Ceftriaxone

C Co-amoxiclav

D Doxycycline

E Gentamicin
Explanation 

A Azithromycin

This patient is most likely to have Legionnaires’ disease, as evidenced by his right lower lobe
pneumonia with hyponatraemia, renal impairment and abnormal liver function tests. Infection
is classically acquired via inadequately sterilised air conditioning equipment in convention
hotels. Modern macrolides which achieve optimal tissue penetration, like azithromycin, are
the intervention of choice, with levofloxacin the alternative where macrolides are
contraindicated. There is evidence that high-dose quinolones may result in a shorter hospital
stay, although they are usually avoided, if possible, because of Committee on Safety of
Medicines (CSM) warnings about the risks of tendon rupture associated with their use.

B Ceftriaxone

Laboratory studies have not shown efficacy of ceftriaxone with respect to killing of Legionella
bacteria. As such, it is not an appropriate intervention here. It is a first-line antibiotic in the
treatment of suspected bacterial meningitis.

C Co-amoxiclav

Co-amoxiclav is ineffective in treating Legionnaires’ disease; it is, however, the first-line


option for treating community-acquired pneumonia, in conjunction with a macrolide,
according to guidelines in the United Kingdom.

D Doxycycline

Tetracyclines are usually considered a third-choice option for treating Legionella infection in
patients where both macrolides and quinolones are not an option.

E Gentamicin

Gentamicin is not a first-line option for treating Legionella infection. It is classically used for
treating Gram-negative sepsis related to infections of the urinary tract or the biliary tree.
51762

Rate this question:

Next Question

Previous Question Tag Question

Feedback End Review


Difficulty: Average

Peer Responses %

Q. Answered Flagged

Q1

Q2

Q3

Q4

Q5

Q6

Q7

Q8

Q9

0:00:17/03:00:00

A 70-year-old man is referred to the Haematology Clinic. He has suffered from progressive
fatigue over the past few months, gradual unintended weight loss and intermittent night
sweats. He is treated with ramipril for hypertension but is otherwise well.
You note left upper quadrant tenderness, with palpable splenomegaly, on abdominal
palpation and a number of petechial haemorrhages affecting his arms and legs.
Investigations:

Haemoglobin (Hb) 9.8 g/dl (13.5–17.5 g/dl)

White cell count (WCC) 5.9 × 10 9/l (4–11 × 10 9/l)

Platelets (PLT) 482 × 10 9/l (150–400 × 10 9/l)

Sodium (Na +) 142 mmol/l (135–145 mmol/l)

Potassium (K +) 5.0 mmol/l (3.5–5.0 mmol/l)

Creatinine (Cr) 112 µmol/l (50–120 µmol/l)

Which of the following would fit best with a diagnosis of myelofibrosis in this patient?

A Presence of the Janus kinase 2 (JAK-2) mutation on cytogenetic testing

B Basophilic stippling seen on blood film

C Early and sustained remission in response to JAK inhibition

D Heinz bodies identified on blood film

E Presence of the BCR–ABL1 mutation on cytogenetic testing

Explanation 

A Presence of the Janus kinase 2 (JAK-2) mutation on cytogenetic testing

The JAK-2 V617F mutation is present in approximately 45–68% of patients with a diagnosis of
myelofibrosis. Under the World Health Organization (WHO) classification for confirming the
diagnosis of myelofibrosis, it is considered an A-criterion, alongside a bone marrow fibrosis
score of > 3 on the 0–4 scale. Diagnosis of the condition requires A1 and A2 criteria and any
two of the B-criteria:

A1: bone marrow fibrosis > 3 (on 0–4 scale)


A2: pathogenetic mutation (eg in JAK-2 or MPL) or absence of both BCR–ABL1 and
reactive causes of bone marrow fibrosis
B1: palpable splenomegaly
B2: unexplained anaemia
B3: leuko-erythroblastosis
B4: tear drop red cells
B5: constitutional symptoms – drenching night sweats, weight loss of > 10% over six
months, unexplained fever (> 37.5 °C) or diffuse bone pains
B6: histological evidence of extramedullary haematopoiesis.

B Basophilic stippling seen on blood film

Basophilic stippling is associated with lead poisoning where microcytic anaemia is also seen
on routine full blood count testing. There is no suggestion of lead poisoning being the
underlying diagnosis here.

C Early and sustained remission in response to JAK inhibition

Although JAK inhibitors are important in controlling splenomegaly and constitutional


symptoms of myelofibrosis, their effects on bone marrow fibrosis are more limited. They tend
to be used late in the disease, with hydroxyurea as the initial therapy of choice for many
patients.

D Heinz bodies identified on blood film

Heinz bodies are identified in red blood cells with decreased ability to deal with oxidants.
They are most commonly reported in associated with glucose-6-phosphate dehydrogenase
(G6-PD) deficiency.

E Presence of the BCR–ABL1 mutation on cytogenetic testing

The absence of this mutation is important in making the diagnosis of myelofibrosis. The JAK-
2 mutation is more likely to be seen in association with the condition.
51684

Rate this question:


Next Question

Previous Question Tag Question

Feedback End Review

Difficulty: Average

Peer Responses %

Q. Answered Flagged

Q1

Q2

Q3

Q4

Q5

Q6

Q7

Q8

Q9

 External Links

BMJ - WHO Myelofibrosis Criteria


newbp.bmj.com/topics/en-gb/1132
(https://newbp.bmj.com/topics/en-gb/1132)
0:00:17/03:00:00

A 61-year-old woman is reviewed in the Emergency Department, following a urea and


electrolytes (U&E) check by her general practitioner (GP). She presented with symptoms of a
urinary tract infection one week earlier and was treated with trimethoprim 200 mg twice
daily (bd). She admits to taking intermittent ibuprofen during the week for a fever.
The GP is concerned, as there has been a rise in the creatinine level from 122 µmol/l pre-
treatment to 149 µmol/l now. Other bloods are unremarkable. The patient now feels well.
Which of the following is the most likely cause of the increased creatinine level?

A Continuing low-grade infection

B Ibuprofen use

C Normal variability in serum creatinine level

D Pre-renal failure

E Trimethoprim use

Explanation 

E Trimethoprim use

Trimethoprim reversibly inhibits renal tubular secretion of creatinine. This is independent of


the glomerular filtration rate (GFR), which usually remains unchanged. It is important for
prescribers to be aware of this, and in this case, it is likely to account for the rise in creatinine
level from 122 to 149 µmol/l.

A Continuing low-grade infection

Given other bloods are unremarkable and the patient now feels well, continuing low-grade
infection is extremely unlikely. Persistent symptoms, the presence of nitrites and white cells in
the urine and a raised CRP level would suggest this as an alternative diagnosis.

B Ibuprofen use
Chronic ibuprofen use may result in chronic interstitial nephritis; short-term, intermittent use
as here is highly unlikely to result in the rise in creatinine level seen with other bloods being
entirely normal.

C Normal variability in serum creatinine level

Although a variance of 5% in creatinine level would be considered acceptable, the rise in


creatinine level seen here, approaching 20%, is certainly abnormal.

D Pre-renal failure

Given other bloods, apart from creatinine, are normal and the patient feels well, pre-renal
failure is unlikely to be the cause of the raised creatinine level. In the event that pre-renal
failure were the cause of the increased creatinine level seen here, symptoms such as postural
hypotension may have been expected or abnormalities in other electrolytes, including a rise
in serum potassium level.
51690

Rate this question:

Next Question

Previous Question Tag Question

Feedback End Review

Difficulty: Average

Peer Responses %

Q. Answered Flagged

Q1

Q2
Q. Answered Flagged

Q3

Q4

Q5

Q6

Q7

Q8 
0:00:17/03:00:00

A 23-year-old woman presents to the Emergency Department with itching affecting her
genital area, which has progressively worsened over the past few days. She has developed an
itchy, red papular rash in the pubic region. She also told you that, over the past few days, she
has noticed a number of small black specks in her underwear at the end of the day.
Which of the following is the most appropriate intervention?

A Fluconazole

B Fucidin ointment

C Malathion lotion

D Shaving of pubic hair

E Terbinafine

Explanation 

C Malathion lotion

The presentation of a red papular rash, coupled with likely louse dirt found in the underwear,
is consistent with infection with pubic lice, otherwise known as crabs. Both malathion and
permethrin are suitable interventions; two applications are given some seven days apart.
Patients should refrain from sexual contact until they and their partner have completed
treatment.

A Fluconazole

Fluconazole is indicated for the treatment of Candida infection, which occurs in skinfolds in
the pubic area and may lead to moist, erythematous skin areas. It leads to a white discharge
when infection affects the vulva.

B Fucidin ointment
The characteristics of the rash seen here and the presence of louse dirt count against a
bacterial cause for this patient’s rash. Bacterial infection is more likely to be associated with
pustular lesions vs the papules seen here. Fucidin is effective in treating topical
staphylococcal infection.

D Shaving of pubic hair

Shaving of pubic hair does not treat pubic lice infection and has no role in its management.

E Terbinafine

Terbinafine may be an option for dermatophyte infection. In the case of the groin, tinea cruris
can lead to a confluent area of erythema with skin scaling. This is different from the papular
rash described here.
51733

Rate this question:

Next Question

Previous Question Tag Question

Feedback End Review

Difficulty: Average

Peer Responses %

Q. Answered Flagged

Q1

Q2

Q3
Q. Answered Flagged

Q4

Q5

Q6

Q7

Q8 
0:00:17/03:00:00

A 29-year-old chef presents to the clinic with progressive weakness affecting both upper
limbs. He is also suffering from loss of sensation affecting his arms. A magnetic resonance
imaging (MRI) scan has demonstrated evidence of syringomyelia.
Which of the following is most likely to be found on examination of the upper limbs?

A Hyperreflexia

B Loss of pain and temperature sensation over the shoulders

C Loss of vibration, light touch and position sense in the hands

D Proximal muscle wasting

E Waiter’s tip deformity

Explanation 

B Loss of pain and temperature sensation over the shoulders

Spinothalamic tract involvement in syringomyelia involves the shoulders and upper arms first
and a shawl-like distribution. This occurs because of compression/obliteration of the anterior
white commissure by the expanding syrinx. Symptoms vary according to the size of the
syrinx, and when the syrinx extends into the brainstem (syringobulbia), cranial nerve palsies
are seen.

A Hyperreflexia

Motor symptoms begin with damage to lower motor neurones of the anterior horn cells. This
leads to hyporeflexia, rather than hyperreflexia, in keeping with a lower motor neurone lesion.

C Loss of vibration, light touch and position sense in the hands

Loss of vibration, light touch and position sense occurs late in the disease in the feet as the
syrinx extends into the dorsal columns.

D Proximal muscle wasting


Muscle wasting begins initially in the hands and can lead to claw hand deformity. As the
disease progresses, the proximal portion of the upper limbs may become involved.

E Waiter’s tip deformity

This is seen in patients with a brachial plexus injury (C5 and C6), most often seen in children
with shoulder dystocia. Claw hand deformity is seen in patients with syringomyelia.
51747

Rate this question:

Next Question

Previous Question Tag Question

Feedback End Review

Difficulty: Average

Peer Responses %

Q. Answered Flagged

Q1

Q2

Q3

Q4

Q5

Q6

Q7
Q. Answered Flagged

Q8 
0:00:17/03:00:00

A 58-year-old man who has diabetes mellitus and ischaemic heart disease presents to the
Emergency Department with a sudden marked deterioration in the vision of his left eye. He
tells you that he awoke from sleep barely able to see his fingers in front of his left eye and
that over the past two hours, the vision has deteriorated further, such that he can only see
light and dark.
Examination of the fundus on the left reveals multiple haemorrhages across the retina in a
‘blood storm’ pattern. Right fundus examination reveals only occasional dot haemorrhages.
Which of the following is the most likely cause of his visual loss?

A Central retinal artery occlusion

B Central retinal vein occlusion

C Cytomegalovirus (CMV) retinitis

D Hypertensive retinopathy

E Ischaemic optic neuropathy

Explanation 

B Central retinal vein occlusion

This patient’s sudden visual loss against a background of a number of vascular risk factors,
coupled with a blood storm pattern on fundoscopy, is consistent with central retinal vein
occlusion. It is most likely ischaemic in origin and therefore carries risks of a very poor
outcome with respect to improvement in visual acuity. This patient should be reviewed every
two to three months to look for evidence of revascularisation, and where it is seen,
intravitreal anti-vascular endothelial growth factor (VEGF) agents are the intervention of
choice.

A Central retinal artery occlusion

This is also associated with sudden deterioration in vision, although fundoscopy usually
reveals a pale retina with an absence of vessels. A few hours after central retinal artery
occlusion, a cherry red spot may be observed; this is related to continued perfusion of a
limited area of the retina via the ciliary arteries. Several treatments have been attempted for
central retinal artery occlusion, although so far, none have proven benefit on outcomes.

C Cytomegalovirus (CMV) retinitis

CMV retinitis is associated with a pizza-like appearance to the retina, with areas of
haemorrhage and cotton wool spots. Infection with CMV occurs against a background of
immunocompromise, and intravenous (IV) ganciclovir is the intervention of choice.

D Hypertensive retinopathy

Although hypertensive retinopathy may be associated with flame-shaped haemorrhages and


cotton wool spots, the sudden marked deterioration in vision seen here would be inconsistent
with the diagnosis. Patients with severe hypertension may, of course, present with bilateral
blurred vision.

E Ischaemic optic neuropathy

Ischaemic optic neuropathy leads to deterioration in vision, but the cause is usually arteritis
affecting the blood supply to the optic nerve. Changes are seen on ophthalmoscopy with
optic nerve pallor, rather than the panretinal changes seen here.
51756

Rate this question:

Next Question

Previous Question Tag Question

Feedback End Review

Difficulty: Average

Peer Responses %
Q. Answered Flagged

Q1

Q2

Q3

Q4

Q5

Q6

Q7

Q8

Q9

0:00:17/03:00:00

A 64-year-old man comes to the Emergency Department with a painful, distended abdomen.
He is a peritoneal dialysis patient due to end-stage renal failure related to type 1 diabetes. He
tells you that his overnight bag failed to drain out properly, and he noted particulate and
proteinaceous material within the drained-out fluid, which he was able to collect. His
temperature is 38.1 °C and blood pressure 115/82 mmHg, and he is generally tender on
abdominal palpation. Bowel sounds are quiet, but active.
Which of the following organisms is the most likely cause of his symptoms?

A Candida albicans

B Corynebacterium striatum

C Enterococcus faecalis

D Staphylococcus aureus

E Staphylococcus epidermidis (SE)

Explanation 

E Staphylococcus epidermidis (SE)

Coagulase-negative staphylococci, of which SE is a prominent member, are responsible for


39% of cases of peritoneal dialysis (PD)-related peritonitis. The incidence of SE peritonitis has
fallen over recent years, because of the introduction of no-touch techniques for PD bag
changes. Vancomycin is the usual intervention of choice.

A Candida albicans

Fungal infection is a rare cause of peritonitis associated with PD. It is an indication for
catheter removal, as it is impossible to eradicate with a catheter in place. Amphotericin B
delivered via the catheter into the intraperitoneal space is the usual initial intervention.

B Corynebacterium striatum
This is a diphtheroid; as a bacterial subgroup, Corynebacteriae are responsible for around
2.5% of cases of continuous ambulatory PD (CAPD) peritonitis. Corynebacteriae are
susceptible to a wide range of antibiotics, including penicillins, tetracyclines and macrolides.

C Enterococcus faecalis

Enterococcus infections occur due to either touch contamination or direct infection via the
gastrointestinal (GI) tract. Vancomycin-resistant Enterococcus (VRE) isolates have increased
in frequency over the past few years, although vancomycin is still the initial intervention of
choice. Nitrofurantoin usually retains efficacy in patients with VRE.

D Staphylococcus aureus

S. aureus is only responsible for around 6% of cases of peritonitis associated with PD. As for
SE, vancomycin is the usual initial intervention of choice.
51750

Rate this question:

Next Question

Previous Question Tag Question

Feedback End Review

Difficulty: Average

Peer Responses %

Q. Answered Flagged

Q1

Q2
Q. Answered Flagged

Q3

Q4

Q5

Q6

Q7

Q8
0:00:17/03:00:00

A 59-year-old woman who has received a renal transplant some five months earlier presents
to the Emergency Department with a severe sore throat. Her immunosuppression regime
includes tacrolimus. She has recently begun a course of oral fluconazole for recurrent genital
candidiasis. Other medications include amlodipine, ramipril, indapamide and linagliptin. Her
temperature is 38.2 °C, pulse 99 bpm and regular and blood pressure 95/70 mmHg. There is
obvious pharyngitis.
Investigations:

Haemoglobin (Hb) 9.8 g/dl (13.5–17.5 g/dl)

White cell count (WCC) 4.1 × 10 9/l Neutrophil count 0.2 (4–11 × 10 9/l)

Platelets (PLT) 122 × 10 9/l (150–400 × 10 9/l)

Sodium (Na +) 144 mmol/l (135–145 mmol/l)

Potassium (K +) 5.7 mmol/l (3.5–5.0 mmol/l)

Creatinine (Cr) 170 µmol/l (50–120 µmol/l)

Electrocardiogram (ECG) QT interval 480 ms

Which of the following medications is the most likely cause of her investigation findings?

A Amlodipine

B Fluconazole

C Indapamide

D Linagliptin

E Ramipril

Explanation 

B Fluconazole
Tacrolimus is extensively metabolised by CYP3A4, which means that potent CYP3A4
inhibitors, such as fluconazole, can lead to a significant increase in tacrolimus levels. Markers
of tacrolimus toxicity include hyperkalaemia, pancytopenia and QT interval prolongation, all
of which are seen here.

A Amlodipine

Although amlodipine is metabolised by CYP3A4, it is not a potent inhibitor. As such, it is


much less likely than fluconazole to have had a significant impact on tacrolimus levels. It is
also likely to have been chronically prescribed for control of hypertension in this case.

C Indapamide

Indapamide has no P450 interaction potential with tacrolimus. It is also a thiazide-like


diuretic, with no potential to worsen hyperkalaemia. Agents such as spironolactone, which do
have the potential to increase potassium levels, should not be used concomitantly with
tacrolimus.

D Linagliptin

Linagliptin, in contrast to fluconazole, is a weak inhibitor of CYP3A4. As such, it is less likely


than fluconazole to result in tacrolimus toxicity when used concurrently. Around 80% of
linagliptin dose is eliminated in the faeces within four days of dosing.

E Ramipril

Ramipril is converted after absorption to ramiprilat. It has no CYP3A4 interaction potential.


Angiotensin-converting enzyme (ACE) inhibitors may, however, lead to hyperkalaemia, which
is a concern with concomitant use of tacrolimus.
51697

Rate this question:

Next Question

Previous Question Tag Question

Feedback End Review

Difficulty: Average

Peer Responses %
Q. Answered Flagged

Q1

Q2

Q3

Q4

Q5

Q6

Q7

Q8

Q9

0:00:17/03:00:00

You are investigating the effects of blood loss in trauma patients and need to provide an
estimate of blood volume.
Which of the following is the expected blood volume in ml/kg for a healthy 25-year-old
man?

A 55

B 65

C 75

D 85

E 96

Explanation 

C 75

Blood volume is determined by muscle mass, fat mass and total surface area. It is also
increased in patients with cardiac failure related to salt and water retention and may, of
course, be reduced where there are losses from the renal and gastrointestinal tract. In a 25-
year-old man with normal lean muscle and fat mass, a total volume of 75 ml/kg would be
expected.

A 55

This represents significant volume depletion. A level of 55 ml/kg would be consistent with
massive fluid loss.

B 65

A volume of 65 ml/kg is the expected blood volume for a healthy woman. It is lower than the
value seen in men because of differences between women and men in muscle mass and fat
percentage.
D 85

A volume of 85 ml/kg is the expected blood volume for a neonate. This is driven by obvious
differences in the distribution of body fat in a neonate vs an adult, and obvious differences in
surface area.

E 96

This is the blood volume per kilogram expected in a premature neonate.


51703

Rate this question:

Next Question

Previous Question Tag Question

Feedback End Review

Difficulty: Average

Peer Responses %

Q. Answered Flagged

Q1

Q2

Q3

Q4

Q5

Q6
Q. Answered Flagged

Q7

Q8

0:00:17/03:00:00

A 32-year-old woman is referred to the Endocrine Clinic as an emergency by her general


practitioner (GP). Over the past two to three weeks, she has begun suffering from flu-like
symptoms, palpitations and dull pain over the anterior neck. She tells you that she cannot
sleep and has lost 3 kg in weight.
Examination reveals a blood pressure of 112/82 mmHg and a pulse of 95 bpm and regular.
She has a fine tremor at rest. Palpation of her neck reveals tenderness over the thyroid gland.
Investigations:

Thyroid-stimulating hormone (TSH) < 0.05 (0.17–3.2 µu/l)

Free thyroxine (T4) 29 pmol/l (11–22 pmol/l)

Free tri-iodothyronine (T3) 8.5 pmol (3.5–5.0 pmol/l)

Which of the following investigations is likely to be the most useful in her further workup?

A Autoimmune profile

B Fine-needle aspiration of the thyroid

C Magnetic resonance imaging (MRI) of the neck

D Thyroid isotope scan

E Thyroid ultrasound scan

Explanation 

D Thyroid isotope scan

The most likely diagnosis is subacute thyroiditis, with a relatively short history of neck pain
and flu-like symptoms, coupled with a thyrotoxic picture on biochemical testing. Because the
disease is characterised by destructive thyroiditis, in contrast to Graves’ disease, uptake of
isotope is actually diminished, rather than increased. Subacute thyroiditis is managed with
non-steroidal anti-inflammatory drugs (NSAIDs) to control pain over the thyroid gland.
Occasionally, a short course of corticosteroids may be required.
A Autoimmune profile

Thyroid peroxidase autoantibodies are present in around 75% of cases of Graves’ disease and
can help differentiate cases of Graves’ disease from a single toxic nodule or toxic
multinodular goitre. They are likely to be of limited value here.

B Fine-needle aspiration of the thyroid

This is most useful in examination of thyroid masses where a suspicious area has been
identified on ultrasound ± thyroid uptake scan.

C Magnetic resonance imaging (MRI) of the neck

MRI of the neck can be useful in evaluating patients before thyroidectomy or where
ultrasound results are equivocal with respect to identifying a suspicious lesion.

E Thyroid ultrasound scan

Ultrasound scanning is of most use in evaluating the size of thyroid nodules and in guiding
fine-needle aspiration biopsy. It may be of value in evaluating the size of a multinodular
goitre.
51715

Rate this question:

Next Question

Previous Question Tag Question

Feedback End Review

Difficulty: Average

Peer Responses %
Q. Answered Flagged

Q1

Q2

Q3

Q4

Q5

Q6

Q7

Q8

Q9

0:00:17/03:00:00

A 76-year-old man has been admitted to the Elderly Care Ward with acute confusion. He has
been diagnosed with a urinary tract infection, for which he is being treated with intravenous
(IV) co-amoxiclav. You are asked to see him because he has become agitated and aggressive.
He has hypertension and ischaemic heart disease and takes a range of medications. He drinks
alcohol only on special occasions.
When you examine him at the request of nursing staff, he is agitated and aggressive, trying to
get out of bed, and he attempts to bite you.
Which of the following is the most appropriate intervention?

A Diazepam

B Donepezil

C Galantamine

D Haloperidol

E Risperidone

Explanation 

D Haloperidol

The National Institute for Health and Care Excellence (NICE) guidelines recommend
haloperidol used at the lowest effective dose for management of delirium, the cause of this
patient’s behaviour on the ward. He should also, of course, be managed with 1 : 1 nursing, and
aids such as cot sides, which may actually increase the risk of injury, should be avoided.

A Diazepam

Benzodiazepines are only considered where agents such as haloperidol are unsuitable
because of a pre-existing movement disorder such as Parkinson’s disease. Lorazepam is the
preferred agent, and it, like haloperidol, should be used at the lowest effective dose.

B Donepezil
Donepezil is a cholinesterase inhibitor used in the treatment of dementia. It has no role in an
acute situation and may actually worsen confusion when there is underlying delirium.

C Galantamine

Like donepezil, galantamine is a chronic therapy for dementia and has no value in treating
delirium. Significant adverse effects of galantamine treatment include bradycardia and
atrioventricular block and bladder outflow obstruction.

E Risperidone

Risperidone is an atypical antipsychotic which is recommended as an initial option for the


treatment of schizophrenia. It is not recommended for the treatment of delirium where
haloperidol or lorazepam are preferred.
51724

Rate this question:

Next Question

Previous Question Tag Question

Feedback End Review

Difficulty: Average

Peer Responses %

Q. Answered Flagged

Q1

Q2

Q3
Q. Answered Flagged

Q4

Q5

Q6

Q7

Q8 

 External Links

National Institute for Health and Care Excellence. 2010. Treating delirium.
nice.org.uk/guidance/cg103/chapter/1-Guidance#treating-delirium
(https://www.nice.org.uk/guidance/cg103/chapter/1-Guidance#treating-delirium)
0:00:17/03:00:00

A 19-year-old man is referred to the Renal Clinic for review. He has had microscopic
haematuria on three occasions over the last six months, and his general practitioner (GP) is
wondering about the need for further investigations. He has no significant medical illnesses
and takes no regular medications. His relatives are all well, although he does report that some
of them, including his father, have had traces of blood in their urine diagnosed at the GP.
His blood pressure is 102/80 mmHg, and physical examination is entirely normal.
Investigations:

Haemoglobin (Hb) 13.2 g/dl (13.5–17.5 g/dl)

White cell count (WCC) 7.0 × 10 9/l (4–11 × 10 9/l)

Platelets (PLT) 210 × 10 9/l (150–400 × 10 9/l)

Sodium (Na +) 144 mmol/l (135–145 mmol/l)

Potassium (K +) 4.5 mmol/l (3.5–5.0 mmol/l)

Creatinine (Cr) 95 µmol/l (50–120 µmol/l)

Urine 1+ blood, protein negative

Which of the following is the most likely diagnosis?

A Alport syndrome

B Immunoglobulin A (IgA) nephropathy

C Medullary sponge kidney

D Polycystic kidney disease

E Thin basement membrane nephropathy (TBMN)

Explanation 

E Thin basement membrane nephropathy (TBMN)


TBMN is an autosomal dominant disorder resulting from similar defects in collagen formation
to Alport syndrome. Unlike Alport syndrome, however, it is not associated with progressive
proteinuria, renal failure and hearing loss, and the disease is not usually more severe in men.
It does not require intervention, and there is only rarely a need to perform renal biopsy to
confirm the diagnosis.

A Alport syndrome

Alport syndrome is usually transmitted as an X-linked disorder (although it may also follow
an autosomal dominant inheritance pattern). What distinguishes this patient from one with
Alport syndrome is that father-to-son transmission has occurred and there is no evidence of
renal failure or hearing loss.

B Immunoglobulin A (IgA) nephropathy

IgA nephropathy is more likely to present with episodes of more marked proteinuria
occurring after a respiratory tract infection. Proteinuria, which would be expected in IgA
nephropathy, is also absent here.

C Medullary sponge kidney

As well as being associated with microscopic haematuria, medullary sponge kidney is also
associated with episodes of urinary tract infection and recurrent renal stones, neither of
which have been reported here.

D Polycystic kidney disease

By age 20, it is expected that patients with autosomal dominant polycystic kidney disease
will have some evidence of renal cyst formation and at least mild renal impairment. Given this
patient has a normal abdominal examination, with no evidence of kidney enlargement, and
normal renal function and no history of renal impairment in the family, this is unlikely to be
the underlying diagnosis.
51754

Rate this question:

Next Question

Previous Question Tag Question

Feedback End Review

Difficulty: Average
Peer Responses %

Q. Answered Flagged

Q1

Q2

Q3

Q4

Q5

Q6

Q7

Q8

Q9

0:00:17/03:00:00

A 23-year-old man comes to the Emergency Department some three weeks after returning
from a trip to South East Asia. He initially developed a shallow, painless ulcer on the glans
penis and is now very concerned as he has begun to develop very swollen inguinal lymph
nodes.
Examination confirms a shallow ulcer on the glans penis with exudate and grossly enlarged,
fluctuant inguinal lymph nodes.
Which of the following is the most appropriate intervention?

A Ciprofloxacin twice daily (bd) for 14 days

B Doxycycline bd for 21 days

C Erythromycin bd for 14 days

D Single-dose ceftriaxone

E Single-dose azithromycin

Explanation 

B Doxycycline bd for 21 days

This patient has lymphogranuloma venereum (LV), acquired as a sexually transmitted


infection. It is caused by the L1, L2 or L3 serovars of Chlamydia trachomatis, which infect
mononuclear phagocytes, rather than squamocolumnar epithelial cells. The disease is largely
confined to tropical regions of the world, although outbreaks do occur in major cities in the
United States and Western Europe. Treatment is with three weeks of doxycycline. Drainage of
lymph node abscesses is best avoided because of the risk of sinus formation.

A Ciprofloxacin twice daily (bd) for 14 days

Quinolones are not a first-line choice for treatment of LV. Use is discouraged in the United
Kingdom because of the risk of tendon rupture associated with their use, and the optimum
treatment duration and dosing regimen for quinolones in LV has not been established.

C Erythromycin bd for 14 days


Macrolides are the main alternative to doxycycline where the latter is contraindicated, with
the best evidence in the class existing for erythromycin.

D Single-dose ceftriaxone

Intramuscular (im) single-dose ceftriaxone is part of the recommended treatment regimen for
gonorrhoea infection, rather than for LV.

E Single-dose azithromycin

Azithromycin 1 g is given in conjunction with im ceftriaxone for the treatment of gonorrhoea.


It is given at a higher dose when ceftriaxone is unsuitable or contraindicated for the
treatment of gonorrhoea.
51739

Rate this question:

Next Question

Previous Question Tag Question

Feedback End Review

Difficulty: Average

Peer Responses %

Q. Answered Flagged

Q1

Q2

Q3
Q. Answered Flagged

Q4

Q5

Q6

Q7

Q8 
0:00:17/03:00:00

A 67-year-old man presents to the clinic with chronic fatigue, night sweats and weight loss
over the past few months. Apart from these symptoms, he has been started on allopurinol
some four months earlier, following an attack of gout.
On examination, he is pale. His blood pressure is 115/82 mmHg, and his pulse is 90 bpm and
regular. He has obvious hepatosplenomegaly on examination of the abdomen, and you note
bruising affecting his arms and legs.
Investigations:

Haemoglobin (Hb) 8.1 g/dl (13.5–17.5 g/dl)

42 × 10 9/l, film shows granulocytes


White cell count (WCC) (4–11 × 10 9/l)
at all stages of development

Platelets (PLT) 81 × 10 9/l (150–400 × 10 9/l)

Sodium (Na +) 139 mmol/l (135–145 mmol/l)

Potassium (K +) 5.3 mmol/l (3.5–5.0 mmol/l)

Creatinine (Cr) 139 µmol/l (50–120 µmol/l)

Cytogenetics Philadelphia chromosome positive

Which of the following is the most likely diagnosis?

A Chronic eosinophilic leukaemia (CEL)

B Chronic lymphocytic leukaemia (CLL)

C Chronic myeloid leukaemia (CML)

D Chronic neutrophilic leukaemia (CNL)

E Primary polycythaemia

Explanation 

C Chronic myeloid leukaemia (CML)


The scenario here of anaemia, low platelets and a marked increase in white blood cell count
driven by myelocytes is consistent with a diagnosis of CML. The presence of the Philadelphia
chromosome further supports the diagnosis. Bone marrow aspiration and biopsy are used to
quantify the percentage of blasts and assess the degree of bone marrow fibrosis. They can
also provide further material for detailed cytogenetic studies.

A Chronic eosinophilic leukaemia (CEL)

CEL is a rare clonal myeloproliferative disorder, characterised by massive overproduction of


normal-looking eosinophils. They cause end-organ damage by infiltration, and a small
percentage progress to CML. Patients with CEL are Philadelphia chromosome-negative.

B Chronic lymphocytic leukaemia (CLL)

CLL presents with a marked increase in peripheral blood B-lymphocytes, and it is not
associated with the Philadelphia chromosome. Patients do present with non-specific
symptoms, including night sweats and lethargy, and splenomegaly as seen here.

D Chronic neutrophilic leukaemia (CNL)

CNL is associated with clonal proliferation of neutrophils alone, and patients are Philadelphia
chromosome-negative. They do not usually progress to CML, although they have limited
survival of up to two years. Responses to interferon and Janus kinase (JAK) inhibitors have
been seen.

E Primary polycythaemia

Primary polycythaemia is associated with a primary increase in Hb, not the anaemia with an
increase in white blood cell count seen here. Primary polycythaemia is associated with the
JAK-2 mutation, and some patients progress to CML.
51683

Rate this question:

Next Question

Previous Question Tag Question

Feedback End Review

Difficulty: Average

Peer Responses %
Q. Answered Flagged

Q1

Q2

Q3

Q4

Q5

Q6

Q7

Q8

Q9

0:00:17/03:00:00

A 52-year-old woman has recovered from a bowel resection for Duke’s C colon cancer. She
works as a local biomedical researcher and asks if any biochemical markers are useful in
monitoring for recurrence of disease.
Monitoring of which of the following has a role in colon cancer surveillance?

A Alpha-fetoprotein (AFP)

B CA 19-9

C CA 72-4

D CA 125

E Carcino-embryonic antigen (CEA)

Explanation 

E Carcino-embryonic antigen (CEA)

Elevated preoperative CEA levels that do not normalise following surgical resection imply the
presence of persistent colon cancer and the need for further evaluation. Cancers of the
pancreas, stomach, breast and lung, medullary carcinoma of the thyroid and ovarian cancer
can also lead to elevations in CEA levels. Patients with a preoperative CEA level of > 5 ng/ml
have a much worse outlook, such that preoperative CEA may be useful in guiding post-
surgical intervention.

A Alpha-fetoprotein (AFP)

Very elevated levels of AFP are seen in hepatocellular carcinoma (HCC). In patients with a
diagnosis of HCC, a level above around 40 ng/ml indicate poor outcome, both overall and in
patients given either local palliation or in whom a cure is attempted. The normal range is 10–
20 ng/ml for AFP levels, and its sensitivity as a screening test is only around 60%. Up to 20%
of HCCs do not produce AFP, meaning it has limited utility as a screening test on its own.

B CA 19-9
CA 19-9 is a tumour marker used in pancreatic cancer, although elevated levels are also seen
in other gastrointestinal neoplasms such as gastric cancer. This means it has limited utility as
an initial screening tool, but in patients who undergo curative surgery who were positive for
CA 19-9 preoperatively, it may have a role in screening for secondary disease.

C CA 72-4

CA 72-4 is a marker of gastric cancer. Along with CA 19-9 and CEA, it has been investigated
as a preoperative marker to determine prognosis. Out of the three, it appears that CA 72-4
most closely approximates with advanced gastric cancer at presentation.

D CA 125

This is a marker of ovarian carcinoma. Like many markers, however, it has limited utility as a
screening tool because it is only positive in around 50% of stage 1 epithelial ovarian cancers.
False positives are also seen in association with benign ovarian disease and in non-ovarian
gynaecological cancers.
51706

Rate this question:

Next Question

Previous Question Tag Question

Feedback End Review

Difficulty: Average

Peer Responses %

Q. Answered Flagged

Q1
Q. Answered Flagged

Q2

Q3

Q4

Q5

Q6

Q7

Q8
0:00:17/03:00:00

A 53-year-old man with type 2 diabetes comes to the Endocrine Clinic for review. His type 2
diabetes was diagnosed some five years ago, and current therapy is metformin 1 g twice daily
(bd). He has recently suffered an inferior myocardial infarction and has been left with residual
shortness of breath. His blood pressure is 139/85 mmHg, and pulse 78 bpm and regular.
There is pitting oedema of both ankles. His HbA1c is measured at 66 mmol/mol. A decision is
made to commence dapagliflozin therapy.
Which of the following is the mode of action of dapagliflozin?

A Alpha-glucosidase inhibition

B Dipeptidyl peptidase-IV (DPPIV) inhibition

C Glucagon-like peptide-1 (GLP-1) agonism

D Glucokinase activation

E Sodium–glucose co-transporter-2 (SGLT-2) inhibition

Explanation 

E Sodium–glucose co-transporter-2 (SGLT-2) inhibition

SGLT-2 inhibitors promote glucose excretion by blocking the action of the SGLT-2
transporter. This leads to increased excretion of blood glucose into the urine, and a
consequent reduction in HBA1c of around 5–6 mmol/mol. SGLT-2 inhibitors also promote mild
diuresis and reduce uric acid levels and have a positive impact on major adverse
cardiovascular events, heart failure and progression of renal disease in type 2 diabetes.

A Alpha-glucosidase inhibition

Glucosidase inhibition reduces the breakdown of complex starches, leading to reduced


absorption of glucose following a meal. The effect on blood glucose levels is modest (5–6
mmol/mol), although the class (including acarbose) in the United Kingdom is associated with
positive cardiovascular outcomes and the prevention of type 2 diabetes (the STOP-NIDDM
trial).
B Dipeptidyl peptidase-IV (DPPIV) inhibition

DPPIV inhibitors, such as sitagliptin, reduce degradation of endogenous incretin hormones


such as GLP-1. They lower HbA1c without increasing the risk of hypoglycaemia, unless they
are combined with insulin or a sulfonylurea. They have a neutral effect on cardiovascular
outcomes.

C Glucagon-like peptide-1 (GLP-1) agonism

GLP-1 agonists, such as liraglutide and dulaglutide, increase the incretin effect and potentiate
insulin release in response to a meal challenge. As well as potentiating insulin release in
response to a meal, they also slow gastric emptying, which has a positive effect by increasing
satiety.

D Glucokinase activation

Glucokinase activators increase insulin release from beta cells and can drive an increased risk
of hypoglycaemia. Trials of these agents have shown tachyphylaxis to their effects over time,
possibly due to accumulation of liver glycogen.
51722

Rate this question:

Next Question

Previous Question Tag Question

Feedback End Review

Difficulty: Average

Peer Responses %

Q. Answered Flagged
Q. Answered Flagged

Q1

Q2

Q3

Q4

Q5

Q6

Q7

Q8

0:00:17/03:00:00

A 72-year-old man is reviewed on the Acute Medical Ward the morning after admission for a
suspected ischaemic stroke. He appears to be neglecting his left-hand side, and when you
ask him to copy an object on a piece of paper, he only copies the right-hand side of the
object.
Where is the most likely site of his infarct?

A Left frontal lobe

B Left occipital lobe

C Left parietal lobe

D Right occipital lobe

E Right parietal lobe

Explanation 

E Right parietal lobe

The right parietal lobe is primarily responsible for visuo-spatial awareness, and injuries to it, in
particular the temporo-parietal junction and the posterior parietal cortex, can lead to neglect.
This may be manifest by visual, auditory, proprioceptive or olfactory symptoms. The left
parietal lobe is mainly thought to deal with language. A middle cerebral artery infarct is the
most likely cause of his stroke.

A Left frontal lobe

Frontal lobe injury can lead to increased risk-taking behaviour, emotional lability and deficits
in executive function, including anticipation, goal selection, planning, initiation, sequencing,
detecting errors and self-correction.

B Left occipital lobe


Occipital lobe injury is associated with cortical blindness, which is different to neglect. Injury
to the left occipital lobe leads to right homonymous hemianopia. It does not lead to the left-
sided neglect seen here.

C Left parietal lobe

Left parietal lobe damage is more likely to lead to defects in mathematical processing,
reading, writing and processing of symbols.

D Right occipital lobe

Occipital lobe defects lead to cortical blindness, and a right occipital lobe defect would
therefore lead to left homonymous hemianopia. Where bilateral occipital lobe damage has
occurred, patients may actually deny blindness, although they are unable to process any
images.
51741

Rate this question:

Next Question

Previous Question Tag Question

Feedback End Review

Difficulty: Average

Peer Responses %

Q. Answered Flagged

Q1

Q2
Q. Answered Flagged

Q3

Q4

Q5

Q6

Q7

Q8
0:00:17/03:00:00

A 22-year-old man is reviewed in the Nephrology Clinic, having presented to the Emergency
Department with left-sided renal colic. Other medical history of note includes recurrent
microscopic haematuria, which has not been investigated by his general practitioner (GP). His
blood pressure is 135/82 mmHg, and pulse 70 bpm and regular. Abdomen is soft and non-
tender; body mass index is 22 kg/m 2.
Investigations:

Haemoglobin (Hb) 12.1 g/dl (13.5–17.5 g/dl)

White cell count (WCC) 7.2 × 10 9/l (4–11 × 10 9/l)

Platelets (PLT) 287 × 10 9/l (150–400 × 10 9/l)

Sodium (Na +) 144 mmol/l (135–145 mmol/l)

Potassium (K +) 3.8 mmol/l (3.5–5.0 mmol/l)

Creatinine (Cr) 137 µmol/l (50–120 µmol/l)

Nephrocalcinosis with several areas


Plain abdominal film of pyramidal medullary calcification
in both kidneys

Which of the following is the most appropriate initial intervention to reduce urinary stone
formation in this patient?

A Bendroflumethiazide

B Citrate supplementation

C Increased fluid intake

D Magnesium supplementation

E Naproxen

Explanation 

C Increased fluid intake


This patient is likely to have medullary sponge kidney, as evidenced by recurrent microscopic
haematuria, renal impairment, the presentation with renal colic and presence of multiple
areas of medullary calcification on plain abdominal film. Oxalate, phosphate and uric acid
stones are all seen in patients with medullary sponge kidney. For this reason, increasing fluid
intake to achieve a urine output of at least 2–3 litres/day is desirable.

A Bendroflumethiazide

Bendroflumethiazide is utilised to reduce calcium excretion. It is considered an option for


patients with recurrent calcium stones where other interventions are ineffective.

B Citrate supplementation

Citrate supplementation is valuable; however, it is a next step on top of increasing fluid input,
rather than an initial intervention. Trials show it is effective in reducing future stone formation
in patients with medullary sponge kidney and recurrent stone disease despite other
measures. Increasing cranberry juice intake is preferred by many patients to taking citrate
tablets.

D Magnesium supplementation

Magnesium and vitamin B6 supplementation is used to reduce oxalate stone formation. It has
been trialed against a low-oxalate diet in patients with idiopathic hyperoxaluria where it was
found to be inferior to dietary modification.

E Naproxen

Non-steroidals may be effective in pain relief, although they do not reduce the risk of stone
formation in patients with medullary sponge kidney. Over the longer term, they may worsen
renal function.
51752

Rate this question:

Next Question

Previous Question Tag Question

Feedback End Review

Difficulty: Average

Peer Responses %
Q. Answered Flagged

Q1

Q2

Q3

Q4

Q5

Q6

Q7

Q8

Q9

0:00:17/03:00:00

A 34-year-old woman comes to the Emergency Department for review. She is extremely
concerned, as over the past 48 hours, she has suffered from left posterior ear pain and now
has paralysis of the left side of her face.
Examination reveals evidence of a left 7th nerve palsy without brow sparing.
Which of the following is the most important intervention?

A Aciclovir

B Aspirin

C Clopidogrel

D Prednisolone

E Valaciclovir

Explanation 

D Prednisolone

A number of trials have supported the use of prednisolone within 48 hours of presentation
with Bell’s palsy (lower motor neurone 7th nerve palsy), the diagnosis here. Prednisolone has
been shown to shorten the time to recovery and improve functional recovery rates by 20% or
more vs placebo. Eye care with topical ocular lubrication is also essential to prevent corneal
damage. The usual dosing regimens are 1 mg/kg of prednisolone or 60 mg/day.

A Aciclovir

Herpes zoster infection is recognised as a cause of Bell’s palsy, although evidence to support
the use of antivirals is weak, with trials showing little added benefit on top of prednisolone
alone. Ramsay Hunt syndrome is specifically used to describe facial nerve palsy in
conjunction with herpes zoster infection affecting the skin of the external ear canal and the
auricle.

B Aspirin
There is no evidence that Bell’s palsy is ischaemic in origin; there is therefore no role for the
use of aspirin. Aspirin is the antiplatelet intervention of choice for primary prevention of
ischaemic cardiovascular events.

C Clopidogrel

Clopidogrel is the intervention of choice for secondary stroke prevention and has no role in
the treatment of Bell’s palsy.

E Valaciclovir

Valaciclovir offers no advantage vs aciclovir, apart from a more convenient dosing regimen.
Like aciclovir, there is limited evidence to support its efficacy on top of prednisolone alone.
51748

Rate this question:

Next Question

Previous Question Tag Question

Feedback End Review

Difficulty: Average

Peer Responses %

Q. Answered Flagged

Q1

Q2

Q3
Q. Answered Flagged

Q4

Q5

Q6

Q7

Q8
0:00:17/03:00:00

A 70-year-old man is brought to the Neurology Clinic by his wife. She is very concerned, as
over the last six months, he has developed problems with short-term memory loss and
slowness of thought. In addition, he has been incontinent of urine on a number of occasions
and has developed problems walking with a shuffling, broad-based gait. His blood pressure in
the clinic is 135/85 mmHg, and pulse is 70 bpm and regular. Neurological exam reveals no
evidence of papilloedema. Reflexes appear slightly brisk. His body mass index is 24 kg/m 2.
Investigations:

Haemoglobin (Hb) 12.2 g/dl (13.5–17.5 g/dl)

White cell count (WCC) 6.7 × 10 9/l (4–11 × 10 9/l)

Platelets (PLT) 271 × 10 9/l (150–400 × 10 9/l)

Sodium (Na +) 144 mmol/l (135–145 mmol/l)

Potassium (K +) 4.5 mmol/l (3.5–5.0 mmol/l)

Creatinine (Cr) 100 µmol/l (50–120 µmol/l)

Computed tomography (CT) Ventricular enlargement

Normal opening pressure,


Lumbar puncture protein 0.4 g/l, glucose 4.5
mmol/l (blood glucose 6.1 mmol/l)

Which of the following is the most likely diagnosis?

A Alzheimer’s disease

B Idiopathic intracranial hypertension (IIH)

C Multi-infarct dementia

D Normal pressure hydrocephalus (NPH)

E Parkinson’s disease

Explanation 
D Normal pressure hydrocephalus (NPH)

NPH is characterised by the triad of gait disturbance, memory impairment and urinary
incontinence, as seen here. The ventricular enlargement and normal opening pressure on
lumbar puncture further support the diagnosis. Magnetic resonance imaging (MRI) scanning
to confirm that ventricular enlargement is out of proportion to sulcal atrophy and
periventricular lucency and cerebrospinal fluid (CSF) dynamics may further support the
diagnosis. Carbonic anhydrase, lumbar puncture and CSF shunt insertion are all potential
interventions.

A Alzheimer’s disease

Alzheimer’s disease is more likely to present first with memory loss, in advance of motor
disturbance and incontinence, and evidence of atrophy is expected to be proportionate to
ventricular enlargement.

B Idiopathic intracranial hypertension (IIH)

IIH is associated with headaches which are usually worse in the morning and on coughing or
straining. The condition is associated with papilloedema, and CSF pressure is elevated on
lumbar puncture.

C Multi-infarct dementia

Multi-infarct dementia is associated with a stepwise deterioration in cognitive function where


personality and affect are usually preserved, until relatively late in the condition. It is
associated with vascular risk factors, which appear largely absent here.

E Parkinson’s disease

Parkinson’s disease is predominantly a motor disorder in the early stages of the disease, with
increased rigidity and tremor, which are not reported here. The relatively early deterioration
in cognitive function does not fit with idiopathic Parkinson’s disease.
51743

Rate this question:

Next Question

Previous Question Tag Question

Feedback End Review


Difficulty: Average

Peer Responses %

Q. Answered Flagged

Q1

Q2

Q3

Q4

Q5

Q6

Q7

Q8

Q9

0:00:17/03:00:00

A 72-year-old man is reviewed urgently, following admission to the Emergency Department


with two transient ischaemic attacks (TIAs) in the past month. He has hypertension managed
with ramipril and amlodipine, and smokes two cigars per week.
His blood pressure is 155/94 mmHg, and his pulse is 70 bpm and regular. Heart sounds are
normal, and there are no bruits. A computed tomography (CT) head and carotid duplex have
been reported as normal.
Investigations:

Haemoglobin (Hb) 19.2 g/dl (13.5–17.5 g/dl)

White cell count (WCC) 12.2 × 10 9/l (4–11 × 10 9/l)

Platelets (PLT) 512 × 10 9/l (150–400 × 10 9/l)

Sodium (Na +) 144 mmol/l (135–145 mmol/l)

Potassium (K +) 4.9 mmol/l (3.5–5.0 mmol/l)

Creatinine (Cr) 105 µmol/l (50–120 µmol/l)

You suspect primary polycythaemia.

Which of the following other findings would most support the diagnosis?

A Elevated serum erythropoietin

B Leukocyte alkaline phosphatase > 100 iu

C Oxygen (O 2) saturation 92% on air

D Presence of JAK-2 mutation

E Splenomegaly

Explanation 

D Presence of JAK-2 mutation


Apart from Hb > 18.5 g/dl, the presence of the JAK-2 mutation is the other major criterion for
making a diagnosis of primary polycythaemia. Other minor criteria for making the diagnosis
include hypercellularity on bone marrow biopsy, serum erythropoietin (EPO) below the
normal range, and endogenous erythroid colony formation in vitro.

A Elevated serum erythropoietin

Elevated serum EPO suggests another driver for the elevated Hb seen here such as chronic
hypoxia. A reduced level of EPO is expected when making a diagnosis of primary
polycythaemia.

B Leukocyte alkaline phosphatase > 100 iu

Measurement of leukocyte alkaline phosphatase has been supplanted by more specific tests
for primary polycythaemia such as JAK-2 testing.

C Oxygen (O 2) saturation 92% on air

Although normal O 2 saturation on air makes the diagnosis of primary polycythaemia less
likely, it does not rule it out completely, meaning that other investigations, such as JAK-2
testing and serum EPO assay, are preferred. Resting O 2 saturations of 92% or below may
drive you towards suspecting secondary polycythaemia due to chronic obstructive
pulmonary disease.

E Splenomegaly

Splenomegaly is found across a range of clonal haematological disorders and is not found
specifically in association with primary polycythaemia.
51686

Rate this question:

Next Question

Previous Question Tag Question

Feedback End Review

Difficulty: Average

Peer Responses %
Q. Answered Flagged

Q1

Q2

Q3

Q4

Q5

Q6

Q7

Q8

Q9

0:00:17/03:00:00

An 18-year-old man is admitted to the Emergency Department having suffered a second


episode of angio-oedema in the past three months. He has been diagnosed with inherited C1-
inhibitor deficiency.
Which of the following is thought to be a cause of the episodes of angio-oedema?

A Decreased activated factor XII

B Deficiency of C3

C Deficiency of C4

D Elevated bradykinin

E Elevated peripheral blood eosinophils

Explanation 

D Elevated bradykinin

Bradykinin is the primary driver for angio-oedema in C1-inhibitor deficiency. It leads to


vasodilatation, increased tissue permeability, bronchoconstriction and cough. Elevated
bradykinin levels occur because of increased production of activated factor XII, which, in turn,
drives greater production of kallikrein from pre-kallikrein, and kallikrein then drives the
production of bradykinin from the breakdown of high-molecular weight kininogen. A number
of triggers for attacks of angio-oedema have been described, including stress, certain drug
therapies and bacterial infection.

A Decreased activated factor XII

C1 inhibitor blocks the conversion of factor XII to activated factor XII. As such, in C1-inhibitor
deficiency, levels of activated factor XII actually rise.

B Deficiency of C3
C3 levels are usually normal in C1-inhibitor deficiency, and changes in C3 levels are not
responsible for the symptoms of angio-oedema. Complement is usually consumed in
vasculitis because of tissue inflammation.

C Deficiency of C4

Although levels of C4 are low in patients with C1-inhibitor deficiency, they are not directly
related to episodes of angio-oedema. Congenital C4 deficiency is highly associated with the
development of systemic lupus erythematosus.

E Elevated peripheral blood eosinophils

Elevated peripheral blood eosinophils are seen in patients with eosinophilic granulomatosis,
chronic parasitic infection and rare clonal haematological disorders driving eosinophilia. They
are not the driver for episodes of angio-oedema.
51707

Rate this question:

Next Question

Previous Question Tag Question

Feedback End Review

Difficulty: Average

Peer Responses %

Q. Answered Flagged

Q1

Q2
Q. Answered Flagged

Q3

Q4

Q5

Q6

Q7

Q8
0:00:17/03:00:00

A 35-year-old woman is referred to the clinic with symptomatic thyrotoxicosis. She has lost 6
kg in weight over the last three months and suffered from intermittent palpitations, and her
periods have almost completely stopped. The general practitioner (GP) confirmed
biochemical thyrotoxicosis and thought the patient had tenderness over the right-hand side
of the thyroid gland. She has completed her family, and her children are 10 and 8 years old.
On examination in the clinic, her blood pressure is 105/80 mmHg, and pulse 90 bpm and
regular. She has a fine tremor at rest.
Investigations:

Thyroid ultrasound – solitary 1-cm thyroid nodule in the right lobe of the thyroid gland.
Isotope uptake scan – uptake concentrated in the right-sided thyroid nodule.

Thyroid autoantibodies are negative.


Which of the following is the most appropriate intervention?

A Long-term carbimazole monotherapy

B Long-term carbimazole and thyroxine in combination

C Partial thyroidectomy

D Total thyroidectomy

E Treatment-dose radioiodine

Explanation 

E Treatment-dose radioiodine

In a patient with a solitary toxic thyroid nodule who has completed her family, with older
children, radioiodine therapy is the default option for treatment. It is administered as a single
capsule of I-131 or as an oral solution, and in the majority of cases, thyrotoxicosis is controlled
within three months. Up to 10% of patients may become hypothyroid over a 3-year period.

A Long-term carbimazole monotherapy


Long-term therapy with thionamides (either carbimazole or propylthiouracil) is not
recommended in this situation and is only really used when patients refuse both surgery and
radioiodine treatment. Thionamide monotherapy is appropriate for the treatment of
thyrotoxicosis in pregnancy.

B Long-term carbimazole and thyroxine in combination

High-dose thionamides, in combination with thyroxine, are considered in patients with


Graves’ disease, particularly when there is very active thyroid eye disease. The reason for this
is the risk of inducing a worsening of eye disease by causing inadvertent hypothyroidism,
including at around the time radioiodine is introduced.

C Partial thyroidectomy

Partial thyroidectomy to remove the nodule is only considered in the event that the patient
refuses radioiodine or radioiodine is unsuitable, eg where a woman has very young children
and cannot remove herself from contact with them. Morbidity is higher for surgery vs
radioiodine, and this drives most patients to radioiodine as the therapy of choice.

D Total thyroidectomy

Total thyroidectomy is not indicated here. It may be indicated where there is a large toxic
multinodular goitre or when a thyroid carcinoma is identified. Where there is a toxic nodule
and other non-functioning nodules are identified, total thyroidectomy may be considered,
depending upon risk assessment by fine-needle aspiration biopsy.
51716

Rate this question:

Next Question

Previous Question Tag Question

Feedback End Review

Difficulty: Average

Peer Responses %
Q. Answered Flagged

Q1

Q2

Q3

Q4

Q5

Q6

Q7

Q8

Q9

0:00:17/03:00:00

A 72-year-old man is reviewed in the clinic following an upper gastrointestinal (GI)


endoscopy for progressive symptoms of vomiting, undigested food and difficulty swallowing.
His endoscopy has demonstrated achalasia. Biopsies from the lower third of the oesophagus
have proved negative for dysplasia or underlying malignancy.
Which of the following interventions is most likely to resolve his symptoms?

A Amlodipine

B Isosorbide mononitrate

C Local botulinum toxin injection

D Omeprazole

E Ranitidine

Explanation 

C Local botulinum toxin injection

Local botulinum toxin injection is effective in 85% of patients with initial intervention,
although the effects wane after a few months. At the 2-year timepoint, only 30% of patients
gain symptomatic benefit from the therapy. Other interventions such as dilatation and
myotomy may be considered.

A Amlodipine

Calcium channel antagonists are only effective in around 10% of patients with achalasia. Due
to the low rate of effectiveness, they are usually reserved for patients in whom other options
for treatment are ineffective.

B Isosorbide mononitrate

Although some patients receive a trial of nitrates for achalasia, a Cochrane review concluded
there is not enough evidence from clinical trials to support their effectiveness.
D Omeprazole

Symptoms associated with achalasia are not associated with excess gastric acid production,
but because of increased pressure at the lower oesophageal sphincter. Once treatment is
instigated, there may be a worsening of reflux, for which proton pump inhibitors may be
needed.

E Ranitidine

Ranitidine is an H2 receptor antagonist, which is available over-the-counter for the treatment


of reflux symptoms. It is less effective than proton pump inhibitors. As achalasia is not caused
by excess acid production, it is not effective here.
51731

Rate this question:

Next Question

Previous Question Tag Question

Feedback End Review

Difficulty: Average

Peer Responses %

Q. Answered Flagged

Q1

Q2

Q3

Q4
Q. Answered Flagged

Q5

Q6

Q7

Q8 
0:00:17/03:00:00

A 76-year-old man comes to the Neurology Clinic following an ischaemic stroke.


He has a residual visual field defect, which is identified as a right superior homonymous
quadrantanopia on formal visual field testing.
Where is the most likely site of the underlying lesion?

A Left occipital lobe

B Left parietal lobe

C Left temporal lobe

D Right temporal lobe

E Right parietal lobe

Explanation 

C Left temporal lobe

The temporal lobe portion of the optic radiation (inferior optic radiation) carries fibres
representing the superior quadrant of the visual field from the opposite side. So, in this case,
a left temporal lobe lesion leads to a right superior quadrantanopia.

A Left occipital lobe

Occipital lobe lesions lead to homonymous hemianopia, rather than the quadrantanopia seen
here. A left lobe lesion would lead to loss of the right side of the visual field in both eyes.

B Left parietal lobe

A parietal lobe lesion affects the superior portion of the optic radiation, leading to an inferior
quadrantanopia on the opposite side of the visual field.

D Right temporal lobe


Because temporal lobe lesions lead to visual loss on the contralateral side, a right temporal
lobe lesion would lead to a left superior homonymous quadrantanopia.

E Right parietal lobe

A right parietal lobe lesion leads to a left inferior quadrantanopia.


51746

Rate this question:

Next Question

Previous Question Tag Question

Feedback End Review

Difficulty: Average

Peer Responses %

Q. Answered Flagged

Q1

Q2

Q3

Q4

Q5

Q6

Q7

Q8
Q. Answered Flagged 
0:00:17/03:00:00

A 29-year-old woman presents to the Emergency Department with polyuria and polydipsia.
She has recently been diagnosed with a multi-drug resistant urinary tract infection (UTI), for
which she was prescribed ofloxacin 200 mg twice daily (bd) by her general practitioner (GP).
She claims to be going to pass urine some 30 times per day and is drinking up to 8 litres of
water per day to keep up.
Examination reveals a blood pressure (BP) of 105/80 mmHg, with a postural drop of 20
mmHg. She is apyrexial. She is admitted overnight for observation.
Investigations:

Haemoglobin (Hb) 13.5 g/dl (11.5–15.5 g/dl)

White cell count (WCC) 7.8 × 10 9/l (4–11 × 10 9/l)

Platelets (PLT) 209 × 10 9/l (150–400 × 10 9/l)

Sodium (Na +) 147 mmol/l (135–145 mmol/l)

Potassium (K +) 4.5 mmol/l (3.5–5.0 mmol/l)

Bicarbonate (HCO 3) 24 mmol/l (24–30 mmol/l)

Creatinine (Cr) 135 µmol/l (50–120 µmol/l)

Urine osmolality 264 mOsm/kg (50–1200 mOsm/kg)

Urine sodium excretion 286 mmol/day (40–220 mmol/day)

Which of the following is the most likely diagnosis?

A Cranial diabetes insipidus (DI)

B Drug-induced nephrogenic DI

C Drug-induced syndrome of inappropriate antidiuretic hormone (SIADH)

D Primary polydipsia

E Renal tubular acidosis (RTA) type 1

Explanation 
B Drug-induced nephrogenic DI

Clues to a diagnosis of DI include polyuria and polydipsia, postural drop in BP on standing,


consistent with relative dehydration and sodium and creatinine levels above the normal
range, coupled with low urine osmolality and elevated urine sodium excretion. Ofloxacin is a
recognised cause of nephrogenic DI, and the mechanism is postulated to be via accumulation
of free fluoride, via metabolism of ofloxacin. This fluoride is thought to lead to activation of
G/q11, involved in mediation of vasopressin receptor activity.

A Cranial diabetes insipidus (DI)

Acquired cranial DI is unlikely here, given there is no history of head injury, infection or
inflammatory process which could drive intracranial pathology. Cranial DI is seen after
encephalitis, meningitis, surgery, radiotherapy and intracranial granulomatous disease such as
neurosarcoidosis.

C Drug-induced syndrome of inappropriate antidiuretic hormone (SIADH)

Drug-induced SIADH leads to hyponatraemia and relative salt and water overload, rather than
the scenario here of polyuria and polydipsia and evidence of inability to keep up with fluid
losses.

D Primary polydipsia

This is a possible differential, although given ofloxacin is a well-recognised cause of


nephrogenic DI, this should be excluded first. Other drugs known to lead to nephrogenic DI
include orlistat and lithium.

E Renal tubular acidosis (RTA) type 1

This leads to hypokalaemic metabolic acidosis. Both potassium and serum bicarbonate levels
are in the normal range, ruling this out. Lithium, topiramate and non-steroidal anti-
inflammatory drugs (NSAIDs) are all recognised causes of RTA type 1.
51719

Rate this question:

Next Question

Previous Question Tag Question

Feedback End Review


Difficulty: Average

Peer Responses %

Q. Answered Flagged

Q1

Q2

Q3

Q4

Q5

Q6

Q7

Q8

Q9

0:00:17/03:00:00

A 51-year-old woman is referred to the Gastroenterology Department by her general


practitioner for investigation of abnormal liver function tests (LFTs). She is obese and tells
you that she drinks a few glasses of wine each evening, although she is able to maintain a job
as a legal assistant, and exercises at the gym each lunchtime. Her blood pressure is 121/82
mmHg, and pulse 67 bpm and regular. She is obese with a body mass index (BMI) of 31
kg/m 2. You can feel a liver edge on palpation of the abdomen, and there are a number of
spider naevi over the upper body.
Investigations:

Haemoglobin (Hb) 11.0 g/dl (11.5–15.5 g/dl)

White cell count (WCC) 6.1 × 10 9/l (4–11 × 10 9/l)

Platelets (PLT) 102 × 10 9/l (150–400 × 10 9/l)

Sodium (Na +) 138 mmol/l (135–145 mmol/l)

Potassium (K +) 3.9 mmol/l (3.5–5.0 mmol/l)

Creatinine (Cr) 89 µmol/l (50–120 µmol/l)

Alanine aminotransferase (ALT) 95 iu/l (5–30 iu/l)

Aspartate aminotransferase (AST) 225 iu/l (10–40 iu/l)

Alkaline phosphatase 182 iu/l (30–130 iu/l)

Gamma glutamyl transpeptidase (GGT) 289 iu/l (5–30 iu/l)

Autoimmune profile Negative

Ultrasound abdomen Evidence of fatty infiltration

Which of the following is the most likely diagnosis?

A Alcoholic steatohepatitis (ASH)

B Decompensated cirrhosis

C Hepatic steatosis

D Non-alcoholic steatohepatitis (NASH)


E Primary biliary cirrhosis (PBC)

Explanation 

A Alcoholic steatohepatitis (ASH)

The clinical picture with fatty infiltration of the liver could fit with both ASH and NASH as the
underlying diagnosis. The > 2 : 1 ratio of AST to ALT is suggestive of ASH, as is the large rise
in GGT seen here. The relative difference in elevation of AST to ALT may be due to hepatic
deficiency of pyridoxal 5'-phosphate in alcoholics, which is a co-factor for ALT activity.
Alcohol cessation can reverse hepatic steatosis associated with ASH. Evidence is better in
patients without advanced cirrhosis where cessation of alcohol may reduce the speed of
progression of disease but not stop it altogether.

B Decompensated cirrhosis

Decompensated cirrhosis is associated with the development of portal hypertension, which is


characterised by ascites and the development of oesophageal varices. There is no evidence
of either here.

C Hepatic steatosis

The marked elevation in transaminases, GGT and alkaline phosphatase is consistent with at
least some degree of hepatic fibrosis, which fits with the diagnosis of ASH, rather than
steatosis alone.

D Non-alcoholic steatohepatitis (NASH)

Given this patient is obese, NASH is a reasonable alternative diagnosis, although the relatively
greater elevation in AST vs ALT fits better with ASH, as does the marked elevation in GGT.
Although this patient reports her alcohol consumption as relatively modest, there is likely to
be underreporting here.

E Primary biliary cirrhosis (PBC)

PBC usually presents with progressive symptoms of lethargy and itching, rather than the
asymptomatic abnormal LFTs seen here. An obstructive picture is seen on liver function
testing, and anti-mitochondrial antibodies are almost always positive.
51728

Rate this question:


Next Question

Previous Question Tag Question

Feedback End Review

Difficulty: Average

Peer Responses %

Q. Answered Flagged

Q1

Q2

Q3

Q4

Q5

Q6

Q7

Q8

Q9

0:00:17/03:00:00

A 72-year-old man is reviewed in the Rheumatology Clinic. He has suffered from pain
affecting his right hip over the past year and progressive loss of movement, so that he is now
able to walk only a few metres without pain. He is otherwise well and takes only amlodipine 5
mg for treatment of hypertension.
His blood pressure is 125/82 mmHg in the clinic. His body mass index is 23 kg/m 2. Movement
of the right hip is severely limited, with reduction in flexion and external rotation due to pain.
Investigations:

Haemoglobin (Hb) 13.1 g/dl (13.5–17.5 g/dl)

White cell count (WCC) 5.9 × 10 9/l (4–11 × 10 9/l)

Platelets (PLT) 301 × 10 9/l (150–400 × 10 9/l)

Sodium (Na +) 144 mmol/l (135–145 mmol/l)

Potassium (K +) 4.5 mmol/l (3.5–5.0 mmol/l)

Bicarbonate (HCO 3) 24 mmol/l (24–30 mmol/l)

Creatinine (Cr) 95 µmol/l (50–120 µmol/l)

Alkaline phosphatase 415 iu/l (30–130 iu/l)

Calcium (Ca) 2.3 mmol/l (2.2–2.6 mmol/l)

Albumin 38 g/l (35–55 g/l)

Gross deformity of the right hip


X-ray hips and pelvis joint with joint destruction
and new bone formation

Which of the following is the most appropriate intervention?

A Intravenous (IV) zoledronate

B Oral raloxifene

C Oral risedronate

D Oral strontium ranelate


E Subcutaneous teriparatide

Explanation 

A Intravenous (IV) zoledronate

Bisphosphonates are the mainstay of management of Paget’s disease, the most likely
diagnosis here, given symptoms and signs affecting one joint only, with a marked elevation in
alkaline phosphatase levels and normal serum calcium levels. One randomised controlled trial
has compared a single dose of IV zoledronate to oral risedronate. It showed more rapid
resolution of symptoms in those given IV zoledronate, a greater percentage of resolution of
symptoms and reduced relapse vs oral risedronate. After review of this evidence on
zoledronate, it is now recommended as the first-line option in the 2019 Paget’s guideline.

B Oral raloxifene

Oral raloxifene is a selective oestrogen receptor modulator (SERM). It is used as a treatment


for osteoporosis when other options, such as bisphosphonates, are not tolerated or are
contraindicated. It does increase the risk of venous thromboembolism and has been shown in
long-term trials to reduce the risk of breast cancer in women who take it.

C Oral risedronate

Although oral bisphosphonates are an effective treatment for Paget’s disease, it has been
shown that IV therapy leads to faster resolution of symptoms and a reduced risk of relapse vs
oral options.

D Oral strontium ranelate

Oral strontium ranelate is a treatment for osteoporosis and is used when other options are
unsuitable or contraindicated, because studies have demonstrated an increase in both
cardiovascular events and venous thromboembolism when used in patients with
osteoporosis.

E Subcutaneous teriparatide

This is a synthetic analogue of parathyroid hormone, which is used in patients with severe
osteoporosis where it has been shown to increase bone mineral density.
51720

Rate this question:


Next Question

Previous Question Tag Question

Feedback End Review

Difficulty: Average

Peer Responses %

Q. Answered Flagged

Q1

Q2

Q3

Q4

Q5

Q6

Q7

Q8

Q9

 External Links

Paget’s Association. New Paget’s guidelines. 2019.


paget.org.uk/news/archive/new-pagets-guideline/
(https://www.paget.org.uk/news/archive/new-pagets-guideline/)
0:00:17/03:00:00

A 45-year-old man who is normally fit and well presents to the Emergency Department with
profuse vomiting in the early hours of the morning. He bought a fried rice Chinese takeaway
on returning from work around 2100 h the previous evening. He has no diarrhoea.
Examination reveals a blood pressure of 110/80 mmHg, with a postural drop of 15 mmHg on
standing. His abdomen is soft but is generally tender, and there are active bowel sounds.
Which of the following is the most likely diagnosis?

A Bacillus cereus (BC) food poisoning

B Giardia lamblia (GL) infection

C Norovirus infection

D Salmonella typhi (ST) food poisoning

E Staphylococcus aureus (SA) food poisoning

Explanation 

A Bacillus cereus (BC) food poisoning

The sudden onset of profuse vomiting suggests this patient’s symptoms are due to a toxin.
Reheated fried rice (the likely cause here) is known to harbour BC bacteria. Two types of BC
toxin are identified, one which causes profuse vomiting and another which causes diarrhoea,
with rapid onset a few hours after ingesting contaminated food. Symptoms settle after a few
hours, and supportive therapy with fluids is all that is required.

B Giardia lamblia (GL) infection

GL infection occurs via the faeco-oral transmission route and is more common in subtropical
countries. It leads to symptoms akin to irritable bowel syndrome with diarrhoea and bloating,
with gradual weight loss. It is managed with a single dose of tinidazole or a short course of
metronidazole.

C Norovirus infection
Norovirus infection leads to sudden-onset vomiting, diarrhoea and abdominal pain, and
would be an alternative diagnosis here, were diarrhoea to be present, in addition to vomiting.

D Salmonella typhi (ST) food poisoning

ST food poisoning is more likely to be associated with a gradual onset of symptoms, with
abdominal pain, diarrhoea (which may contain blood), nausea and vomiting. Patients are also
often pyrexial.

E Staphylococcus aureus (SA) food poisoning

SA, like BC, food poisoning is caused by a toxin and leads to rapid-onset vomiting a few
hours after ingesting contaminated food. Cream and dairy products are the usual suspect
where they have been left out on a warm day, for example as part of a buffet display.
51727

Rate this question:

Next Question

Previous Question Tag Question

Feedback End Review

Difficulty: Average

Peer Responses %

Q. Answered Flagged

Q1

Q2

Q3
Q. Answered Flagged

Q4

Q5

Q6

Q7

Q8 
0:00:17/03:00:00

A 74-year-old woman who is a resident in a nursing home because of cognitive impairment is


admitted to the Emergency Department with projectile vomiting. On arrival in the
department, she has also soiled herself with watery diarrhoea. Apparently, seven other
residents in the nursing home and some staff members have been unwell over the past week.
Her blood pressure is 90/60 mmHg, with a pulse of 100 bpm. The abdomen is soft and
generally tender, with very active bowel sounds.
Investigations:

Haemoglobin (Hb) 11.0 g/dl (11.5–15.5 g/dl)

White cell count (WCC) 10.9 × 10 9/l (4–11 × 10 9/l)

Platelets (PLT) 299 × 10 9/l (150–400 × 10 9/l)

Sodium (Na +) 145 mmol/l (135–145 mmol/l)

Potassium (K +) 3.4 mmol/l (3.5–5.0 mmol/l)

Creatinine (Cr) 189 µmol/l (50–120 µmol/l)

Which of the following is the most likely diagnosis?

A Bacillus cereus gastroenteritis

B Ischaemic colitis

C Norovirus infection

D Shigella sonnei infection

E Staphylococcus aureus gastroenteritis

Explanation 

C Norovirus infection

The acute onset of symptoms with both projectile vomiting and watery diarrhoea in a nursing
home resident, where others in the home are ill, is very typical of norovirus infection.
Supportive management with intravenous (IV) fluid rehydration is the mainstay of therapy.
Patients who are infected with norovirus should be isolated from others, if possible, and
barrier-nursed to prevent spread of infection via healthcare staff.

A Bacillus cereus gastroenteritis

Ingestion of B. cereus toxin is associated with either acute-onset vomiting or acute-onset


diarrhoea, and reheated fried rice is a common route of infection. Symptoms resolve within a
few hours, and supportive management is all that is required.

B Ischaemic colitis

Mesenteric angina presents with recurrent colicky abdominal pain and intermittent diarrhoea.
Significant ischaemia with bowel necrosis presents with abdominal pain, metabolic acidosis
and a significant rise in serum lactate levels, which is not seen here.

D Shigella sonnei infection

Shigella infection does occur in institutions, including in nursing homes, although the main
symptom is bloody diarrhoea, alongside abdominal pain, rather than the vomiting and watery
diarrhoea seen here.

E Staphylococcus aureus gastroenteritis

S. aureus gastroenteritis occurs because of ingestion of S. aureus toxin, which leads to


vomiting. Contaminated dairy products are the most common cause of ingestion.
51730

Rate this question:

End Session

Previous Question Tag Question

Feedback End Review

Difficulty: Average

Peer Responses %
Q. Answered Flagged

Q1

Q2

Q3

Q4

Q5

Q6

Q7

Q8

Q9

You might also like